Está en la página 1de 531

HEMATOLOGIA

Cifras normales de hemoglobina y hematocrito


al nivel del mar (Wintrobe)

Hemoglobina Hematocrito

Recién nacido
19.5 ± 5.0 54 ± 10
Mujeres
14.0 ± 2.0 42 ± 5

Hombres 16.0 ± 2.0 47 ± 7


• BANDAS
• 1-3%
QUIMICA Y ELECTROLITOS
• GLUCOSA • HDL
• 70- 105 • 40- 90

• UREA • LDL
• 10- 50 • 0 – 120

• ACIDO URICO • VLDL


• 2.5- 7.7 • 0- 50

• NITROGENO UREICO • ALBUMINA


• 6- 20 • 3.5- 5.5

• CREATININA SERICA • SODIO


• 0.5- 1.2 • 135- 145

• COLESTEROL • POTASIO
• 120- 200 • 4- 5.3

• TRIGLICERIDOS • CLORO
• 10- 160 • 98- 106
PFH Y ENZIMAS CARDIACAS
• BILIRRUBINA TOTAL • ALT/TGP
• 0 – 1.5 • 0 – 35

• BILIRRUBINA INDIRECTA • DHL


• 0 – 1.50 • 100 – 190

• BILIRRUBINA DIRECTA • GGT


• 0 – 0.5 • 7 – 50

• AST/TGO • CPK
• 6 – 38 • 0 – 226
GASES ARTERIALES
• PH
• 7.35--7.45
• PCO2
• 33-45mmHg
• PO2
• 75-105mmHg
OSMOLARIDAD SERICA Y LCR
• OSMOLALITY SERUM
• 275-295mOsmol/Kg
• LCR
• CELLCOUNT
• 0.5cells/mm3
• CHLORIDE
• 118--132mEq/L
• GAMMA GLOBULIN
• 3--12% total Proteins
• GLUCOSE
• 40--70mg/dL
• PRESSURE
• 70--180mm h2O
• PROTEINS,TOTAL
• < 40mg/dL
ANEMIA POR DEFICIT DE HIERRO
ANEMIA POR DEFICIT DE HIERRO
ANEMIA MEGALOBLÁSTICA
# de Eritrocitos Otros
Hemoglobina estudios
Hematocrito
V.G.M. Aspirado de
H.G. M. médula ósea:
C.M.H.G. N HIPERCELULAR
Reticulocitos N o lig.
Leucocitos bilirrubina
Diferencial L= indirecta = lig.
Frotis N=
Plaquetas DHL =
VITAMINA B12
A que se refiere el termino POLICITEMIA ESPURIA?
R = AUMENTO DEL HTO por aumento de la concentración del VOLUMEN PLASMÁTICO NO POR EL > DE LA
MASA ERITROCITARIA

Cual es el cc de la TROMBOSIS ESENCIAL?


R = El primer signo puede ser trombosis mesentérica, hepática o portal. Eritromelalgia.

Cuales son los datos de laboratorio de la trombosis esencial?


1) PLAQUETAS >2, 000,000
2) FSP CON > DE PLAQUETAS GRANDES
3) MO CON > DE MEGACARIOCITOS
4) GEN FILADELFIA AUSENTE que descarta leucemia mieloide crónica

Que otras enfermedades pueden causar > plaquetario sin ser TE?
R = AR, CUCI e infección crónica

Cual es el manejo de la TE?


R = HIDROXIUREA y ASA hasta reducir a < 500, 000

Que es la MIELOFIBROSIS?
1) Trastorno mieloproliferativo caracterizado por FIBROSIS DE LA MO, esplenomegalia y cuadro leuco
eritroblastico en FSP con POIQUILOCITOS EN LAGRIMA.
2) HEMATOPOYESIS EXTRAMEDULAR
Cual es la etiología de la mielofibrosis?
R = Se desarrolla en respuesta a un > EN LA SECRECIÓN DE FACTOR DE CRECIMIENTO DERIVADO DE LAS PLAQUETAS

Cual es el cc de la mielofibrosis?
1) Fatiga, distensión abdominal por esplenomegalia, DOLOR OSEO PRINCIPALMENTE RETROESTERNAL,
hepatomegalia.
2) Hemorragias por secuestro de plaquetas
3) HIPERTENSIÓN PORTAL Y VARICES ESOFÁGICAS POR HEMATOPOYESIS HEPÁTICA

Cuales son los datos de laboratorio y punción obtenidos en la mielofibrosis?


1) Anemia
2) FSP con POIQUILOCITOS EN LAGRIMA, formas inmaduras mieloides y PLAQUETAS GIGANTES
desgranuladas
3) PUNCIÓN SECA por aumento de fibras reticulares

Cual es la triada clásica de la mielofibrosis?


1) POIQUILOCITOS EN LAGRIMA
2) SANGRE LEUCO-ERITROBLASTICA
3) PLAQUETAS GIGANTES
4) Supervivencia de 3-5ª

Cual es el manejo de la mielofibrosis?


R = Transfusiones continuas, TETOSTERONA. TMO + talidomida

Cuando se considera LEUCEMIA según la OMS?


R = Cuando hay al menos 20% DE BLASTOS EN SANGRE O MO.
LEUCEMIA AGUDA LINFOBLASTICA
LEUCEMIA AGUDA LINFOBLASTICA
Cuales son las bases para el dx de LLC?
R = Es una malignidad ORIGINADA EN LOS LINFOCITOS B, en la cual cursan asintomáticos, CON
INFILTRACIÓN DE LB EN LOS ÓRGANOS

Cual es el cc de LLC?
R = Fatiga, LINFADENOPATIA Y HEPATO-ESPLENOMEGALIA

Cual es el hallazgo de laboratorio de la LLC?


1) LINFOCITOSIS AISLADA > 5000
2) MARCADOR CD 19 EN LB Y CD 5 EN LT

Cual es el manejo de la LLC?


R = Clorambucil

Cual es la estirpe histológica de la que proviene la LEUCEMIA DE CÉLULAS VELLOSAS?


R = LINFOCITOS B

Cual es el cc de LCV?
R = Fatiga, ESPLENOMEGALIA o asintomático
PUNCION SECA
I. MIELOFIBROSIS
• Cual es la etiología de la mielofibrosis?
R = Se desarrolla en respuesta a un > EN LA SECRECIÓN DE FACTOR DE CRECIMIENTO DERIVADO DE LAS
PLAQUETAS

• Cual es el cc de la mielofibrosis?
1) Fatiga, distensión abdominal por esplenomegalia, DOLOR OSEO PRINCIPALMENTE RETROESTERNAL,
hepatomegalia.
2) Hemorragias por secuestro de plaquetas
3) HIPERTENSIÓN PORTAL Y VARICES ESOFÁGICAS POR HEMATOPOYESIS HEPÁTICA

II. LEUCEMIA DE CELULAS VELLOSAS


• Cual es la estirpe histológica de la que proviene la leucemia de células vellosas?
R = LINFOCITOS B

• Cual es el cc de LCV?
R = Fatiga, ESPLENOMEGALIA o asintomático
Que es el MIELOMA MÚLTIPLE?
1) Es una neoplasia de células plasmáticas que sustituye la MO.
2) Hay DESTRUCCIÓN ÓSEA con formación de PARAPROTEÍNAS plasmáticas IgG o IgA
3) Hay HIPERVISCOCIDAD debido a paraproteínas
4) Puede provocar IRA por las paraproteínas

Cual es el cc del mieloma múltiple?


1) DOLOR ÓSEO 70% que aumenta con los movimientos, principalmente en la espalda CON EVIDENCIAS DE FRACTURAS
VERTEBRALES
2) Edad > 65ª , ANEMIA, dolor óseo, infección
3) Amiloidosis manifiesta: macroglosia, neutropatia, ICC o hepatomegalia

Cuales son los datos de laboratorio del MM?


1) HIPERCALCEMIA, Sx anémico con ANEMIA NORMO-NORMO por efecto supresor de eritropoyetina por IL6
2) NEFRITIS intersticial con pérdida DE CADENAS LIGERAS
3) Biopsia de MO con infiltración normal o difusa

Cual es el manejo de mieloma múltiple?


1) Vincristina, doxorrubicina y dexametazona
2) TMO autologo
3) TALIDOMIDA + DEXAMETAZONA PARA RECAÍDAS
4) Dx dif. Con plasmocitoma solitario que responde bien a la radioterapia que no presenta la clínica típica del mieloma

Como estadificas la carga tumoral en el MM?


1) BAJA: con espiga IgG <5 gr/dl sin hipercalcemia ni IR
2) ALTA: con espiga IgG > 7 gr, Hto <25% y calcio > 12

Que es la MACROGLOBULINEMIA DE WALDESTROM?


R = Enfermedad MALIGNA DE LB, con hibrido de LB célula plasmática con PARAPROTEINA IgM
Cuales son las causas secundarias más comunes de PTI?
1) Fármacos como la heparina
2) LES y LLC

Cual es el mecanismo por el cual la heparina causa trombocitopenia?


R = Auto anticuerpos IgG vs factor 4 plaquetario

Cual es el manejo de la PTI?


1) PREDNISONA 1-2 mg/kg/dia con respuesta en una semana
2) INMUNOGLOBULINA iv para aumento de plaquetas
3) ESPLENECTOMÍA

Que es la PURPURA TROMBOCITOPENICA TROMBOTICA?


R = Es una enfermedad caracterizada por ANEMIA HEMOLÍTICA MICROANGIOHEPATICA,
TROMBOCITPENIA y AUMENTO NOTABLE DE DHL SÉRICA (a diferencia de la PTI donde no hay
elevación de la DHL)

Cual es la etiología de la PTT?


R = Deficiencia de “PROTEASA” que divide al factor de VON WILLEBRAND (VIII), por lo tanto se acumula
dicho factor originando aglutinación de plaquetas en endotelio vascular. TAMBIÉN LO PUEDE
DESENCADENAR EL EMBARAZO, LOS ESTRÓGENOS ELEVADOS Y LAS INFECCIONES.

Cual es el cc de la PTT?
1) PENTADA CLASICA: Anemia hemolítica microangiopatica, Trombocitopenia, Afección neurológica, Falla
Renal y Fiebre. ATARF
2) Fiebre, palidez, PETEQUIAS
3) Anemia, hemorragia.
4) Síntomas neurales como HEMIPARESIA QUE DURA MINUTOS
DEFICIT DE ADHESION PLAQUETARIO

I. Que es la tromboastenia de GLANZMAN?


A. Es un trastorno AR que produce hemorragias ( trastorno
plaquetario). LAS PLAQUETAS NO PUEDEN AGREGARSE por que no
hay RECEPTORES (GLUCOPROTEINAS IIB Y IIIA) para el fibrinógeno.
B. Se maneja con desmopresina

I. Que es el síndrome de BERNARD-SOULTIER?


A. Es un trastorno AR que OCASIONA HEMORRAGIAS. Las PLAQUETAS
NO SE ADHIEREN por FALTA DEL RECEPTOR IIB para el factor de vWF
B. Se maneja con transfusion de plaquetas
A que se debe la HEMOFILIA TIPO C y cual es su principal cc?
R = Déficit del FACTOR XI y se manifiesta con HEMORRAGIAS LEVES.

Que es la HEMOFILIA TIPO A?


R = Es un trastorno hereditario AR LIGADO AL X con déficit del FACTOR VIII y SOLO
AFECTA A VARONES

Cual es el cc de la hemofilia tipo A?


R = HEMORRAGIA EN ARTICULACIONES de rodilla, tobillo, codo, músculos y aparato GI

Cuales son los datos de lab en la hemofilia tipo A?


1) TPT SE PROLONGA únicamente
2) FACTOR VIII DISMINUIDO Y VWF NORMAL

Cual es el manejo de la hemofilia tipo A?


R = Concentrados de FACTOR VIII 4000 UI para 70 kg

Como puede transmitirse la enfermedad de anticuerpos vs el factor VIII?


R = Puede desarrollarse POST PARTO o sin una causa, hemofilia A que han recibido
concentrados plasmáticos.
Cual es el cc de la enfermedad de anticuerpos vs el factor VIII?
R = Hemorragia grave

Cual es el manejo de elección en la enfermedad de anticuerpos vs el factor VIII?


R = CICLOFOSFAMIDA

Que es la HEMOFILIA TIPO B o enfermedad de Christmas?

R = Trastorno AR LIGADO AL X con déficit de FACTOR IX en el cual se eleva el TPT


Cual es el manejo de la hemofilia tipo B o enfermedad de Christmas?
R = Concentrados de FACTOR IX 6000 para 70 kg

Que factores de coagulación se ven afectados en COAGULOPATIA POR ENFERMEDAD


HEPÁTICA?
1) Se fabrican todos los factores de coagulación en el hígado EXCEPTO EL FACTOR VIII
2) Primero se ven afectados los vitamina K dependientes II, VII, IX y X y el factor V
3) Aumenta la fibrinólisis por la disminución de plasmina

Cual es el manejo de la coagulopatia por enfermedad hepática?


R = PLASMA FRESCO CONGELADO
Cual es el cc de la coagulación intravascular diseminada?
1) HEMORRAGIAS Y TROMBOSIS
2) Hemorragia +++ por punción, heridas o espontanea
3) Trombosis + como isquemia digital o gangrena

Que es el SÍNDROME DE TROSEAU en la coagulación intravascular diseminada?


R = Coagulación intravascular diseminada manifiesta como TROMBOSIS VENOSA SUPERFICIAL Y PROFUNDA POR CÁNCER
principalmente de manera recurrente.

Cuales son los datos de lab para la coagulación intravascular diseminada?


1) HIPOFIBRINOLEMIA
2) Aumento de productos de degradación de la fibrina, TROMBOCITOPENIA y AUMENTO DEL TP Y TPT
3) DÍMERO D como producto de degradación de la fibrina que aumenta si hay fallo hepático.

Cual es el manejo para la coagulación intravascular diseminada?


1) TRATAR EL TRASTORNO SUBYACENTE
2) Heparina, S- AMINOCAPROICO (AUMENTA EL FIBRINÓGENO)
3) El ÉXITO del tx consiste en que AUMENTE EL FIBRINÓGENO
4) CRIOPRECIPITADOS PARA FIBRINÓGENO

Como se diagnostica la disfibrinolemia?


R = Aumento del TIEMPO DE REPTILASA

Que medicamento se utiliza de elección en los estados de hipercoagubilidad?


R = WARFARINA
PATOLOGIA CELULA PATOGNOMONICA
ANEMIA DE CELULAS FALCIFORMES Cuerpos de Howell-Jolly:Estructuras únicas o
dobles, pequeñas y redondas. Se observan
como gránulos densos y de color azul rojizo o
violeta. Localizados excéntricamente

DEFICIT DE 6 GPDH Cuerpos de Heinz: Citoplasma de los glóbulos


rojos , aparecen como pequeños puntos
oscuros bajo el microscopio

LINFOMA DE HODKING Reed Stemberg cells: Celula B modificada por


lo general es muy grande dando la pariencia de
ojos de Buho con nucléolos prominentes en
forma de inclusiones, la variedad mononuclear
tiene un solo nucleo y un nucléolo prominente.

LEUCEMIA AGUDA Bastones de Auer: Son estructuras filiformes


dentro de los blastos
ANEMIA SIDEROBLASTICA Cuerpos de Papenheimer: Son acúmulos
de hemosiderina unida a proteínas. Consisten
en gránulos basófilos, con las tinciones
habituales, que además, se tiñen también de
azul con el colorante de Perls (azul de Prusia).
PATOLOGIA CELULA PATOGNOMONICA
INTOXICACION POR PLOMO, LEUCEMIA Y Punteado basofilo: Pueden ser agregados
TALASEMIA. ribosómicos originados por una degeneración
vacuolar del citoplasma o precipitados de
cadenas globínicas libres. Consiste en puntitos
basófilos, con las tinciones habituales, de
tamaño variable y dispersos por toda la
superficie del hematíe.

ANEMIA MEGALOBLASTICA Anillos de cabot: Están formados por restos de


la membrana nuclear o de microtúbulos.
Consisten en una especie de hilos basófilos,
con las tinciones habituales, que adoptan una
forma de anillo o de ocho y que pueden
ocupar toda la periferia celular.

CÉLULA WARTHIN-FINKELDEY Sarampión: Célula gigante multinucleótica con


citoplasma eosinofílico e inclusiones nucleares
PATOLOGIA
• De que patología son característicos los CUERPOS DE HEINZ?
R = Déficit de glucosa 6PDH

• De que patología son patognomónicos los cuerpos de HOWELL-JOLLY?


R = Anemia de células falciformes

• De que patología son patognomónicos los BASTONES DE AUER?


R = Leucemia mieloide aguda en la que además la distingue la presencia de “MIELOPEROXIDASA”

• Que es la LEUCEMIA MIELOGENA CRÓNICA?


R = Se caracteriza por SOBREPRODUCCIÓN DE CÉLULAS MIELOIDES (principalmente LEUCOCITOS), presenta EL COMOSOMA FILADELFIA
(translocacion de los brazos 9 y 22) BCR/ABL que produce “TIROCINA CINASA”.

• Que diferencia la LEUCEMIA MIELOIDE AGUDA de la leucemia linfoblastica aguda?


R = Mieloperoxidasa que no se produce en la LLA

• Cual es el hallazgo patognomónico de linfoma de Hodking?


R = RED STEMBERG CELLS (CÉLULA B MODIFICADA por lo general es muy grande dando la apariencia de ojos de Búho con nucléolos prominentes
en forma de inclusiones, la variedad mononuclear tiene un solo núcleo y un nucléolo prominente).

• Que patología te da trombos hialinos en la biopsia?


R = PTT y SUH

• Citogenetica reconocible con T (18:14) de que enfermedad es patognomónico?


R = Linfoma no Hodking
ENF. DE HODKING
• Característica de la Enfermedad de Hodgkin?
• Linfadenopatia indolora, Síntomas constitucionales, no se conoce su origen, diseminación
ordenada

• Cuadro clínico?
• MASA INDOLORA (CUELLO), mas frecuente en varones, diseminación ordenada, fiebre de larga
evolución, DISMINUCIÓN DE PESO, PRURITO INTENSO, sudación nocturna, DOLOR GANGLIONAR
(ALCOHOL)

• Diagnostico definitivo?
• Biopsia del ganglio linfático afectado, CÉLULAS DE REED-STENBERG, no hay anormalidades
cariotipicas

• El virus Epstein Barr con que enfermedad tumoral se ha relacionado?


• Enfermedad de Hodgking

• En la Enfermedad de Hodgking cual es la variedad histológica mas frecuente?


• ESCLEROSIS NODULAR
• En la Enfermedad de Hodgking cual es la variedad de mejor pronostico?
• VARIEDAD LINFOCITICA

• En la Enfermedad de Hodgking cual es la variedad menos frecuente?


• Depleción linfocitaria

• En la Enfermedad de Hodgking que se relaciona con VIH?


• DEPLECIÓN LINFOCITARIA

• Tratamiento de la de la Enfermedad de Hodgking?


1) MOPP (mecloretamina, Vincristina, procarbacina, prednisona)
2) ABVD (adriamicina, bleomicina, Vincristina, Dacarbacina)

• Clasificación de ann-arbor?
• Etapa 1 una sola cadena ganglionar
• Etapa 2 2 regiones ganglionares (mismo lado del diafragma)
• Etapa 3 regiones ganglionares en ambos lados del diafragma
• Etapa 4 metástasis
• (A asintomático, B síntomas constitucionales)
ENF. NO HODKING
• Característica de la Enfermedad no Hodgking?
• No tiene cél de Reed Stemberg, si hay citogenética reconocible T (8:14), correlación con proto-
oncogen C-myc

• Cuadro clínico de la E no H?
• Linfadenopatia desordenada, a menudo diseminada al momento del diagnostico, síntomas
constitucionales

• En que consiste el termino de ganglio de Richter?


1) Es un ganglio generalmente SUPRACLAVICULAR .
2) GANGLIO que comparte una LLC q posteriormente se convertirá (EVOLUCIONARA) en LINFOMA NO
HODGKIN

• Tratamiento de los linfomas no Hodgkin?


1) 1 solo ganglio afectado: radioterapia local … si esta asintomático valorar conducta expectante
2) Tx: clorambucilo o CVP+fludarabina, CHOP

• Tratamiento para un MALTOMA?


• Terapia para HELICOBACTER PYLORI
LEUCEMIA AGUDA
• Cáncer mas frecuente en niños?
 Leucemia siendo la aguda la mas frecuente (80%)

• El cáncer de los ganglios linfáticos se llama?


 Linfoma

• El cáncer de medula ósea?


 Leucemia

• Característica de la leucemia Linfoblastica Aguda?


 Tienen blastos (20% en MO y 90% en sangre periférica) y su caract.
Es la capacidad de infiltrar órganos (encías, pericardio, testículos, MO,
SNC etc), 2da. Capacidad de producir Citopenias o Pancitopenias
• Cuadro Clínico de LLA?
 Por procesos de infiltración hay: Hemorragias (petequias, equimosis) Adenopatías,
Hepato-esplenomegalias, puede llegar a una CID

• Diferencia entre LLA y la hipoplasia medular y purpura trombocitopenica


idiopática

 En que puede haber trombocitopenia pero TANTO EN LA HIPOPLASIA COMO EN


LA PURPURA NUNCA habrá ganglios linfáticos palpables o hepatoesplenomegalias

• Características de la CID?
 Tiempo de protrombina alargado, dímero D presente, trombocitopenia.

• Que tipo de leucemia mielocitica aguda tiene la característica de presentarse


como una CID
 La promielocitica y la monocitica aguda (habitualmente son de buen pronostico
mientras que no se presente con CID)
• Citogeneticas desfavorables?
 Monosomia 5 y 7, Cromosoma Ph t(4;11) (Encontrar Cromosoma ph en LMA(2%) es
de muy mal pronostico habitualmente se observa en LMC (98%)

• Con respecto a marcadores de superficie:


 Determinación de una Enzima desoxinucleotidil transferasa terminal….. es una LLA (95%)

• Si es MIELOPEROXIDASA POSITIVA…. LMA


 Marcador CD19, CD10…. Es una LLA de células B (mas frecuente)
 Marcador CD2, CD5, CD7…. Es una LLA de células T (menos frecuente)

• Tratamiento de Leucemias Mielocitica Agudas?


 1ra etapa: Inducción de la Remisión Daunorrubicina o Idarrubicina mas citarabina
 2da etapa: Quimioterapia intensa de repetición mas transplante de MO autologa

• Tratamiento de la Leucemia Linfoblastica aguda?


 1ra etapa: Inducción de la Remisión Vincristina, Prednisona, Daunorrubicina, L
Asparaginasa (4 medicamentos) Metrotexate (profilaxis del SNC)
 2da etapa: Quimioterapia a dosis altas mas transplante de MO
• Leucocitosis con Desviación a la DERECHA:
 Cuando los LINFOCITOS son los que están elevados (arriba del 20%)

• Leucocitosis con Desviación a la IZQUIERDA:


 Cuando los NEUTRÓFILOS están elevados

• Característica Histológicas de LMA?


 Mieloperoxidasas positivas, cuerpos de AUER en citoplasma

• Características Histológicas de LLA?


 Antígeno CALLA (Antígeno común de LLA) y PAS Acido periódico de Shift
positivo

• Citogenética Favorables?
 En px con LMA t(8:21), t(15:17),inv (cr16), Inv (p13; q22) buen
pronostico
LEUCEMIA CRONICA
• Característica de una LLC?
1) Linfocitos mayores de 5000, linfocitos de apariencia normal (madura), con expresión de CD19, y CD5
2) Origen: malignidad de linfocitos B (95%) inmunoincopetentes

• Cuadro clínico de una LLC?


 Se presenta en px MAYORES DE 50 AÑOS (90%), linfocitosis impresionante, linfadenopatia y
Hepatoesplenomegalia

• Clasificación de la LLC (CLASIFICACION RAI)


 ETAPA O Linfocitosis mas de 5000
 ETAPA 1 Linfocitosis MAS LINFADENOPATIA (revisar al px)
 ETAPA 2 Linfocitosis MAS ORGANOMEGALIA HEPATOESPLENOMEGALIA
 ETAPA 3 Linfocitosis mas anemia
 Etapa 4 Linfocitosis , lo anterior mas trombocitopenia (mal pronostico)

• Laboratorios de una LLC?


 LINFOCITOSIS AISLADA (mas de 5000), Leucos mayor de 20000, inmunogenotipo: CD19 (LLC de Celulas
B), CD5 (LLC de Celulas T)
• Tratamiento de LLC con sintomatología?
 CLORAMBUCILO alternativa fludarabina en px jóvenes

• Característica de LMC?
1) Leucocitosis, Cromosoma Ph t(9:22) presentes en un 95%,
2) Leucocitosis con desviación a la izquierda (aumento delos
neutrófilos)

• Tx de LMC?
 EL TX no es urgente Mesilato de Imatinib, alternativa Hidroxiurea
o interferon alfa

• Cuadro clínico de LMC?


 Crecimiento impresionante de bazo, mas leucocitosis con desviación
a la izq con serie mieloide, sensibilidad esternal, cromosoma Ph (95%)
SINDROME PARANEOPLASICO/ AC
• Frecuentemente presentan pleocitosis linfocitaria en LCR
con aumento de proteínas y de IgG. Algunos pacientes
presentan anticuerpos característicos en suero y LCR que
proveen una fuerte evidencia de que el cuadro
neurológico es paraneoplásico y que además sugieren el
tipo de cáncer asociado, por ejemplo el anticuerpo anti-
Yo se asocia con degeneración cerebelosa y cáncer
ginecológico, el anticuerpo anti-Hu con encéfalomielitis
paraneoplásica y cáncer pulmonar de células pequeñas y
el anticuerpo anti-Ki con opsocionus y cáncer de mama
MICELANEAS
• En un paciente con anemia ferropénica que
esperaría encontrar en su estudio de
laboratorio?
• Ferritina, ferremia y saturación de transferrina
bajas y transferrina alta.
• Por cuanto tiempo se realiza el tratamiento
de la anemia ferropenica?
• R = 6 meses
CARDIOLOGIA

ENARM
EKG
EKG
 INTERVALOS
 PR:
Se denomina así, al espacio que va del inicio de la Onda P al comienzo de la Onda R.
Valor normal: 120 a 200 milisegundos ( 0.12 – 0.20 segundos).

 COMPLEJO QRS:
Corresponde a la activación del miocardio ventricular.
Valor Normal: 60 a 100 milisegundos ( 0.06 – 0.10 segundos).
 QT:
Se mide desde el inicio del Complejo QRS hasta el final de la Onda T y corresponde a la duración total de la Sístole
Ventricular.
Valor normal: 240 a 480 milisegundos ( 0.24 – 0.48 segundos ).
( Varia de acuerdo a la frecuencia cardiaca )

 SEGMENTO
 ST:
Es el intervalo normalmente iso-electrico entre el final del Complejo QRS y el inicio de la Onda T.
Valor normal: 60 a 160 milisegundos ( 0.06 a 0.16 segundos )
EKG
• INTERVALO QT
 P r o l o n g a d o:
 C a r d i o p a t í a I s q u é m i c a.
 I. C. C.
 M i o c a r d i t i s.
 Drogas: Quinidina, Amiodarona, Antidepresivos triciclicos...
 H i p o m a g n e s e m i a.
 H i p o c a l c e m i a.
 H i p o k a l e m i a ?.

 A c o r t a d o:
 R e p o l a r i z a c i ó n p r e c o z.
 D r o g a s: Digital...
 H i p e r c a l c e m i a.
 H i p e r k a l e m i a.
HIPOKALEMIA

• < 3 mEq/L: onda T


plana, depresión ST,
ondas U

• < 2,5 mEq/L: onda U


prominente,
inversión onda T, PR
y QT prolongado,
QRS ensanchado
HIPERKALEMIA : POTASIO SÉRICO > 5 mEq/L

LEVE 5.0 - 5.5 mEq/L

MODERADA 5.5 - 6.0 mEq/L

SEVERA > 6.0 mEq/L


HIPERPOTASEMIA Y EKG

K+ 5.0 - 6.5 mEq/L Cambios mínimos

K+ 6.5 - 8.0 mEq/L Onda T picuda


Aplanamiento y desaparición de la onda P
Depresión del segmento ST

K+ 8.0 a más Ensanchamiento del QRS


BAV
Arritmias cardíacas
Ritmo idioventricular
Paro cardíaco en asistolia.
EJES ELECTRICOS EKG
PULSO VENOSO YUGULAR
AXVAYC
• Se distinguen fundamentalmente dos ondas, la "a" y la "v". La primera, la onda
"a", ocurre justo antes del sístole, y se debe a la CONTRACCIÓN DE LA AURÍCULA
DERECHA (al final del diástole, cuando se termina de vaciar al ventrículo derecho).
El colapso de la vena después de la onda "a", es el descenso "x" y se debe a
la RELAJACIÓN DE LA AURÍCULA. AX

v
• La onda " " se debe al LLENE PASIVO DE LA AURÍCULA DERECHA debido al
retorno venoso normal, mientras la VÁLVULA TRICÚSPIDE PERMANECE CERRADA
durante el sístole. Por lo tanto, es una onda que ocurre al mismo tiempo del sístole
y que se vería sobre el vena yugular. El colapso que se observa después de la onda
y
"v", se denomina el descenso " ", que corresponde al PASO DE LA SANGRE
DE LA AURÍCULA AL VENTRÍCULO DURANTE EL DIÁSTOLE, después que se abre la
válvula tricúspide. VAY
• Con registros muy finos, se describe una pequeña muesca ubicada en el descenso
c
de la onda "a", que se ha llamado la onda " ", ATRIBUIDA AL CIERRE DE LA
VÁLVULA TRICÚSPIDE, después que se ha terminado de contraer la aurícula
derecha y está comenzando el sístole, pero no es posible de ver a simple vista.
FARMACOLOGIA
ANGINA DE PECHO
ANGINA ESTABLE
• Es la más frecuente, aparece con el esfuerzo y
remite espontáneamente con el reposo y/o la
medicación. Posee una duración de pocos
minutos y presenta un patrón regular, por lo que
el paciente puede ser capaz de identificarla e
incluso predecir su aparición. Su origen se halla
primordialmente en una arteriopatía
aterosclerótica que causa la progresiva reducción
de la luz vascular, de uno o varios vasos
coronarios, en porcentajes del orden del 70% o
superior.
ANGINA INESTABLE
• La angina inestable no se relaciona con un mayor trabajo cardíaco,
es decir no deriva de un mayor consumo miocárdico de oxígeno. Su
causa debe buscarse en una disminución aguda del flujo cardíaco
coronario, que puede deberse a la complicación de una placa
aterosclerótica coronaria por erosión, fisura o rotura y trombosis
sobreañadida que cause una interrupción súbita del flujo coronario
o por causas extrínsecas al árbol coronario que produzcan
inestabilización. Su sintomatología clínica es muy similar a la que
registra el infarto agudo de miocardio, sin embargo, en la angina
inestable no se produce necrosis miocárdica.

• La angina inestable incluye diversos tipos de anginas caracterizadas


por su evolución imprevisible, aunque no siempre fatal y que se
apartan claramente del patrón típico de angina estable: angina de
reciente comienzo, angina progresiva, angina de reposo, angina
prolongada, angina vasoespástica o de Prinzmetal y angina
postinfarto, todas ellas consideradas urgencias médicas.
IAM
IAM
IAM
SICA
CORONARIAS INVOLUCRADAS EN IAM
ARTERIA
DERIVACION CON
TIPO DE INFARTO CORONARIA COMPLICACIONES
ELEVACION DEL ST
RESPONSIBLE
Descendente
V1-V2 Septal anterior (ramos Bloqueos de rama
septales)
Descendente
Disfuncion VI, IC,
V3-V4 Pared anterior anterior (ramos
Bloqueo de rama
diagonales)
Hipotension (evitar
V5-V6 Lateral alto Circunfleja nitroglicerina o
morfina)
Posteroinferior con
Derecha (ramos
DII, DIII y aVF extension al Hipotension
proximales)
ventriculo derecho
Circunfleja o
V1-V4 Posterior descendente Disfuncion VI
posterior
PREGUNTAS
Cual es la triada clásica del infarto del ventrículo
derecho?
 Hipotensión, campos pulmonares limpios y
elevación de la PVY

Cual es el tratamiento de la taquicardia auricular


multifocal?
 Suele asociarse a enfermedad pulmonar grave,
mejora con la ventilación mecánica y la
oxigenación
TIPOS DE SHOCK
VALVULOPATIAS
RESUMEN SOPLOS
• ESTENOSIS MITRAL; SINTOMAS Y SOPLO
 DISNEA de esfuerzo, ORTOPNEA, EAP, disfonía por compresión del NLR.
Imagen de 4 arcos en Rx por congestion venocapilar.
 Soplo diastólico
• INSUFICIENCIA MITRAL; SINTOMAS Y SOPLO
 DISNEA de esfuerzo y fatiga, ORTOPNEA, DISNEA PAROXÍSTICA
NOCTURNA. Por prolapso valvular dolor torácico (signo de Barlow) dx dif con
SICA.
 Soplo sistólico
• ESTENOSIS AORTICA; SINTOMAS Y SOPLO
 Triada clásica, ANGINA DE PECHO - IC – SINCOPE. Palpitaciones, visión
borrosa. HV.
 Soplo sistólico
• INSUFICIENCIA AORTICA; SINTOMAS Y SOPLO
 Disnea que va de esfuerzo a DISNEA PAROXÍSTICA NOCTURNA, estertores.
 Soplo diastólico.
BRI
BRD
B AV I
• Prolongacion del intervalo PR en forma
continua
• >20” adultos
• >16” en infantes
B AV II/ M1
B AV II/M2
B AV C
RESUMEN BLOQUEO AV I-III
BLOQUEOS AV MANEJO
FA
QT LARGO
TRAZOS EKG PATOLOGICOS
SX WPW
SX BRUGADA
INSUFICIENCIA CARDIACA
INSUFICIENCIA CARDIACA
ENDOCARDITIS INFECCIOSA
Endocarditis infecciosa
Profilaxis antibiótica en procedimientos dentales,
cavidad oral, respiratorio y esófago

A. Amoxicilina 2gr 1 hora antes


B. Ampicilina 2gr IM 30 minutos antes
(en intolerancia oral)
Alergia a penicilina
1) Claritromicina 500 mg 1 hora antes
2) Clindamicina 600 mg 1 hora antes
3) Cefalexina 2gr 1 hora antes
Endocarditis infecciosa
Profilaxis antibiótica en procedimientos genitourinarios
y gastrointestinales

• Ampicilina 2 gr IM o IV + Gentamicina
1,5mg/Kg/ 30 minutos antes, 6 h después
Ampicilina 1gr IM/IV o Amoxicilina oral 1gr

Alérgicos a penicilina

• Vancomicina 1gr IV en 1-2 h + Gentamicina


1,5 mg/Kg IV/IM terminando la perfusión 30
minutos después del procedimiento
LESIONES DE JANEWAY
MANCHAS DE ROTH
NODULOS DE OSLER
TAMPONADE
JOVEN + DOLOR PRECORDIAL, FIEBRE Y ST (LESIÓN)
EXTENSA = PERICARDITIS AGUDA. LO EXACERBA LA RESPIRACIÓN.
ES FRECUENTE ANTECEDENTE DE IRAS VIRALES
DE NO CONOCERSE LA ETIOLOGÍA SE DAN AINES (AAS)
PERICARDITIS
• El supradesnivel del ST en la pericarditis se diferencia del infarto
agudo del miocardio porque en éste el supradesnivel es convexo y
más localizado, pueden coexistir ondas T negativas al mismo tiempo
del supradesnivel, presencia de ondas Q cuando es un infarto Q y el
EKG no normaliza antes de hacerse negativa la onda T.

• De la angina de Prinzmetal se distingue porque el supradesnivel es


transitorio y sólo durante el dolor; además, compromete
derivaciones más localizadas.

• La imagen de la repolarización precoz es parecida pero nunca hay


depresión del PR, no evoluciona y no hay síntomas.
MIOCARDIOPATIAS
Adulto + anasarca + complejo bajos =
pericarditis constrictiva
Rx de mujer , imagen de hiperflujo pulmonar, ECG con
BRD y eje a la derecha= Comunicación Interauricular
(OS)
CARDIOPATIAS CIANOGENAS Y
ACIANOGENAS
CIV
CIA
COARTACION AORTICA
ANEURISMAS
CARDIOPATIAS CONGENITAS
DISECCION AORTICA
EMBOLIA VS TROMBOSIS
OCLUSION ARTERIAL CRONICA
HAS
HAS
HAS
HAS
• Cual es el signo electrocardiografico mas común en tromboembolismo pulmonar?
• R = Taquicardia sinusal.

• -A 42-year-old man develops shortness of breath (SOB) and chest pain 7 days after an open cholecystectomy.
His blood pressure is 145/86 mm Hg, pulse is 120/min, respirations 24/min, and oxygen saturation of 97%.
Pulmonary embolism is clinically suspected. Which of the following is the most common ECG finding of
pulmonary embolism?

• (A) a deep S wave in lead I


• (B) depressed ST segments in leads I and II
• (C) prominent Q wave in lead I, and inversion of T wave in lead III
• (D) sinus tachycardia
• (E) clockwise rotation in the precordial leads
• Correcta D

• Cuales son los síntomas de estenosis mitral?


• R = DISNEA, ortopnea y disnea paroxística nocturna

• Como es el soplo en insuficiencia mitral?


• R = PanSISTOLICO en vértice e irradiado a axila.

• -A 25-year-old woman is found to have a midsystolic murmur on routine evaluation. The murmur does not
radiate but it does increase with standing. She otherwise feels well and the rest of the examination is normal.
• R = Mitral valve prolapsed.

• -A 65-year-old man with a previous history of an anterior MI comes for follow-up. On examination, he has a
systolic murmur heard best at the apex and radiating to the axilla. Transient external compression of both arms
with blood pressure cuffs 20 mm Hg over peak systolic pressure increases the murmur.
• R = Regurgitación mitral
• Que caracteriza al signo de MUSSET y en que patología se presenta?
• R = Hay movimientos de la cabeza en cada pulsación debido a INSUFICIENCIA DE LA VÁLVULA AORTICA.

• -A 75-year-old man is bought to the hospital because of a syncopal episode. There was no incontinence or post-event confusion. On examination, his blood
pressure is 140/80 mm Hg, pulse 72/min with no postural changes. His second heart sound is diminished and there is a systolic ejection murmur that radiates
to the carotids. With the Valsalva maneuver, the murmur decreases in length and intensity.
• R = Aortic stenosis.

• En que patología se escucha el soplo de AUSTIN FLINT?


• Insuficiencia aortica

• Que sucede cuando hay regurgitación aortica en forma aguda?


• Insuficiencia ventricular izquierda manifestada como EDEMA AGUDO PULMONAR.

• -A 32-year-old asymptomatic woman has a rapidly rising, forceful pulse that collapses quickly. Which of the following is the most likely diagnosis?
• (A) mitral stenosis
• (B) mitral regurgitation
• (C) aortic stenosis
• (D) aortic regurgitation
• Respuesta correcta D

• Que medicamento es de elección en el síndrome de Marfan con regurgitación aortica?


• R = BETABLOQUEADORES

• Que medicamentos pueden disminuir la dosis de homocisteina?


• R = Acido fólico, B6 y B12.

• Cuál es el fenómeno de hibernación y aturdimiento miocardico?


• R = Son las áreas del miocardio que se encuentran subperfundidas que se adaptan para ser viables con disfunción contráctil sostenida.

• Que es el síndrome de TIETZE?


• R = Inflamación de uniones condrocostales tumefactas, rojas que causa dolor retroesternal

• Que antianginosos han demostrado prolongar la vida?


• R = Los BETABLOQUEADORES a excepción de el pindolol que exacerba la angina
RESUMEN SOPLOS
• ESTENOSIS MITRAL; SINTOMAS Y SOPLO
 Disnea de esfuerzo, ortopnea, EAP, disfonía por compresión del NLR.
Imagen de 4 arcos en Rx por congestión venocapilar.
 Soplo diastólico
• INSUFICIENCIA MITRAL; SINTOMAS Y SOPLO
 Disnea de esfuerzo y fatiga, ortopnea, disnea paroxística nocturna. Por
prolapso valvular dolor torácico (signo de Barlow) dx dif con SICA.
 Soplo sistólico
• ESTENOSIS AORTICA; SINTOMAS Y SOPLO
 Triada clásica, angina de pecho - IC – Sincope. Palpitaciones, visión borrosa.
HV. Signo de Musset.
 Soplo sistólico
• INSUFICIENCIA AORTICA; SINTOMAS Y SOPLO
• Disnea que va de esfuerzo a disnea paroxística nocturna, estertores. PULSO
DE CORRIGAN , QUINQUE Y DUROZEIZ . Soplo de Austin Flint.
 Soplo diastólico.
• En cuanto tiempo tiene que estar el TTP cuando se da tratamiento anticoagulante?
• R = 50-70 segundos.

• En que pacientes se utiliza la profilaxis antiarrítmica?


• R = LATIDOS ECTÓPICOS FRECUENTES y taquicardia ventricular.

• Como se trata una fibrilación ventricular que no responde a una cardioversión?


• R = Amiodarona y RCP

• Como se dx infarto del ventrículo derecho?


• R = Con la ELEVACIÓN DEL ST DEL HEMITORAX DERECHO

• -A 69-year-old woman complains of some atypical chest pain 2 days prior to presentation. On examination, the
JVP is at 8 cm, positive Kussmaul’s sign, and normal heart sounds. The lungs are clear. The ECG is abnormal, and
the CXR shows a normal cardiac silhouette.

• Que tan frecuente se presenta un AUNERISMA VENTRICULAR IZQUIERDO POST IAM?


• R = 20% y se da el dx por medio de la ELEVACIÓN PERSISTENTE DEL ST DURANTE 4-8 SEMANAS.

• -Three months after an anterior MI, a 73-yearold man has a follow-up ECG. He is clinically feeling well with no
further angina symptoms. His ECG shows Q waves in the anterior leads with persistant ST-segment elevation.
The current ECG is most compatible with which of the following diagnosis?

• (A) ventricular aneurysm


• (B) hibernating myocardium
• (C) acute infarction
• (D) silent infarction
• (E) early repolarization
• Que caracteriza al SÍNDROME DE DRESSLER?
• R = SE PRESENTA POST IAM, debido a un FENÓMENO AUTO INMUNITARIO CON PERICARDITIS, FIEBRE,
LEUCOCITOSIS y ocasionalmente derrame pericardico o pleural.

• -A 67-year-old man presents with an anterior myocardial infarction (MI) and receives thrombolytic therapy.
Three days later, he develops chest pain that is exacerbated by lying down, and his physical findings are normal
except for a friction rub. His ECG shows evolving changes from the anterior infarction but new PR-segment
depression and 1-mm ST-segment elevation in all the limb leads. Which of the following is the most likely
diagnosis?

• (A) reinfarction
• (B) pulmonary embolus
• (C) viral infection
• (D) post-MI pericariditis
• (E) dissecting aneurysm
• Respuesta correcta D

• Cual es el manejo del infarto agudo del VD?


• R = CARGA LIQUIDA para mejorar el llenado ventricular derecho e INOTRÓPICOS.

• Cual es el mecanismo de acción de los antiarrítmicos clase Ia y cuales son?


• R = BLOQUEADORES DE LOS CANALES DE SODIO. Procainamida, quinidina y disopiramida.

• Cual es el mecanismo de acción de los antiarrítmicos clase Ib y cuales son?


• R = BLOQUEADORES DE LOS CANALES DE SODIO. Lidocaina y difenilhidantoina.

• Cual es el mecanismo de acción de los antiarrítmicos clase II y cuales son?


• R = BETABLOQUEADORES, retardando la conducción AV. Propanolol, metoprolol y esmolol.
• Cual es el mecanismo de acción de los antiarrítmicos clase III y cuales son?
• R = PROLONGAN EL POTENCIAL DE ACCIÓN. Amiodarona, sotalol, dofelitida.

• Cual es el mecanismo de acción de los antiarrítmicos clase IV y cuales son?


• R = BLOQUEADORES DE LOS CANALES LENTOS DE CALCIO. Verapamil, diltiazem, digoxina y adenosina.

• Con que fenómenos se relaciona la taquicardia paroxística supraventricular, la cual suele cursar asintomática.
• R = Intoxicación por digitalicos, bloqueos AV y fenómeno de reentrada.

• Que tratamiento se utiliza en taquicardia paroxística supraventricular en caso de estar contraindicados los
antiarrítmicos clase IV y que ha demostrado 100% de éxito?
• R = Cardioversion.

• Cual es el fármaco de elección en caso de prevención de ataques de taquicardia paroxística supraventricular?


• R = DIGOXINA

• Que caracteriza al síndrome de LOWN-GANONG-LEVINE?


• R = INTERVALO PR CORTO Y MORFOLOGÍA NORMAL DE QRS

• En que casos no necesita tratamiento las taquicardias supraventriculares causadas por vías accesorias o síndrome de
preexitacion?
• R = En casos de no presentar palpitaciones, mareos o sincope.

• Cual es el procedimiento de elección en pacientes con síndrome de preexitacion?


• R = ABLACIÓN POR RADIOFRECUENCIA

• Que agentes farmacológicos deben evitarse en el síndrome de preexitacion?


• R = Digoxina, betabloqueador y bloqueador de los canales de calcio.

• Cual es el tratamiento a largo plazo de el síndrome de preexitacion?


• R = Procainamida, verapamil y digoxina.
• Cuales son los fármacos de elección en extrasístoles ventriculares sintomáticas?
• R = BETABLOQUEADORES.

• Que medida se utiliza cuando la taquicardia ventricular es recurrente?


• R = Marcapasos

• - A22-year-old woman complains of palpitations and has a regular heartbeat at a rate of 170/min, with a blood pressure of 110/70 mm Hg. The rate
abruptly changes to 75/min after applying carotid sinus pressure. Which of the following is the most likely diagnosis?
• (A) sinus tachycardia
• (B) paroxysmal atrial fibrillation
• (C) paroxysmal atrial flutter
• (D) paroxysmal supraventricular tachycardia (PSVT)
• Respuesta correcta D

• Cual es el mecanismo de elección para la prevención de muerte súbita en un paciente con factores de riesgo?
• R = Desfibrilador implantado

• Que caracteriza al síndrome QT PROLONGADO?


• R = SINCOPE RECURRENTE, arritmias ventriculares y muerte súbita. Es una ANOMALÍA CONGÉNITA hereditaria que afecta los CANALES DE Na y K.

• Cual es la terapéutica mas eficaz en el síndrome de QT prolongado?


• R = BETABLOQUEADORES Y DESFIBRILADOR IMPLANTADO.

• Que hallazgo EKG caracteriza al bloqueo AV tipo I?


• R = PROLONGACIÓN DEL PR > .20” EN ADULTOS Y > .16”. LA ALTERACIÓN ES NODAL.

• Que hallazgo EKG caracteriza al bloqueo AV tipo II?


1) - 1er grado o fenómeno de WENCKEBACH, MOBITZ I: prolongación progresiva del PR hasta que 1 onda P no conduce.
2) - 2do grado o MOBITZ II: intervalo PR, sin embrago hay onda P que no conduce con un patrón 2:1 o 3:1

• En que casos se diagnostica el síndrome de SENO ENFERMO?


• R = PARO SINUSAL, bloqueo sinoauricular, BRADICARDIA SINUSAL PERSISTENTE o bradiarritmias.
• Cual es el manejo del síndrome de seno enfermo?
• R = TEOFILINA ORAL Y MARCAPASOS.

• Cual es la indicación terapéutica del Mobitz III?


• R = MARCAPASOS

• Cuales son los síntomas inadvertidos de ICC?


• R = Tos crónica, nicturia

• Que medicamento se utiliza en caso de intoxicación por digoxina?


• R = Anticuerpos FAP

• Cual es el cuadro clínico característico de alguien con ICC con edema agudo pulmonar?
• R = DISNEA, tos, ESPUTO ROSADO, diaforesis, cianosis.

• En que consiste el fenómeno de reentrada?


• R = Involucra un circuito que conduce de forma anterograda al ventrículo y de forma retrograda a la aurícula
utilizando vías accesorias.

• Que tratamiento se utiliza en ICC después de que los diuréticos y nitratos no mejoran la sintomatología?
• R = NISERITIDA, forma recombinante de péptido atrial natriuretico del cerebro humano.

• Que agente antihipertensivo se debe evitar en ICC?


• R = CALCIOANTAGONISTAS

• Como es típicamente la Rx de torax en edema agudo pulmonar con ICC?


• R = PATRÓN EN MARIPOSA en la distribución del edema alveolar.
• En el EAP la presión capilar en cuna se encuentra siempre elevada, a que cantidad aproximadamente?
• R = > 25 mm Hg

• La morfina es eficaz en el manejo de derrame pleural cardiogenico, debido a que mecanismo?


• R = AUMENTA LA CAPACITANCIA VENOSA, disminuye la presión auricular izquierda y disminuye la ansiedad.

• Cuál es la diferencia entre cardioversión y desfibrilación?


• R = La primera se encuentra sincronizada con complejo QRS y la 2da no hay sincronización.

• Cuáles son los agentes más comunes para miocarditis aguda?


• R = VIRAL (COXACKIE) bacteriano, riketsias, espiroquetas, micoticos y parasitarios.

• -A 23-year-old man develops sharp left-sided chest pain, fever, and a friction rub heard at the lower left sternal
border, unaffected by respiration. The pain is also aggevated by lying down and relieved by sitting up. He is
otherwise well with no other symptoms and the remaining physical examination is normal. Which of the
following is the most likely cause for his symptoms?

• (A) rheumatic fever


• (B) tuberculosis (TB)
• (C) herpes simplex virus
• (D) MI
• (E) coxsackievirus
• Respuesta correcta E

• La miocarditis Riketsial , por que enfermedad es originada?


• R = TIFO, fiebre de las montanas rocallosas y fiebre Q.
• Cual es el agente causal y cual es el periodo de incubación de la ENFERMEDAD DE CHAGAS?
• R = Es causado por el TROFOSZOITO TRIPANOSOMA CRUZI y su principal manifestación clínica aparece
en 10 años.

• Que afección sistémica es la regla para un individuo que padece Chagas?


• R = MEGAESOFAGO O MEGACOLON.

• Entre las afecciones parasitarias cual es la mas frecuente de afección cardiaca?


• R = La triquinosis.

• Cual es la fisiología de la maniobra de valsalva?


• R = Se produce un aumento del tono parasimpático por liberación de acetilcolina, lo que genera un
retardo en la conducción AV causando un bloqueo AV transitorio.

• Que dato puede ser único o inicial en el EKG en MIOCARDITIS INFECCIOSA?


• R = Ectopia ventricular

• Que medicamento esta indicado en el espasmo coronario inducido por la COCAÍNA?


• R = BETABLOQUEADORES Y CALCIOANTAGONISTAS

• Cuales son los síntomas de MIOCARDIOPATIA DILATADA?


• R = Disnea, dolor torácico .

• Cual es el manejo del paciente con CARDIOPATÍA HIPERTRÓFICA?


• R = BETABLOQUEADORES, CALCIOANTAGONISTAS Y MARCAPASOS.
• -A 45-year-old woman has developed increasing SOB on exertion and fatigue. She has a loud systolic ejection murmur heard best at the left
sternal border, and the murmur increases with standing. A double apical impulse is also felt.
• R = Cardiomiopatia hipertrofica.

• 66. Cuales son las causas principales de MIOCARDIOPATIA RESTRICTIVA?


• R = AMILOIDOSIS, RADIACIÓN, fibrosis por cirugía, sarcoidiosis, hemocromatosis y síndrome carcinoide.

• -57 A 63-year-old man develops edema, and dyspnea on exertion. He has no prior cardiac or renal conditions, and his examination is
significant for macroglossia, elevated jugular venous pressure (JVP), hepatomegaly, and 3+ pedal edema. His investigations reveal 3.5 g/d of
protein in the urine, anemia, normal fasting glucose, and serum immunoelectrophoresis is positive for a monoclonal immunoglobulin.
Which of the following is the most characteristic neurologic finding associated with this condition?

• -Peripheral motor and sensory neuropathy: In addition to peripheral motor and sensory neuropathy, cardiac involvement, tongue
enlargement, gastrointestinal (GI) manifestations, and carpal tunnel syndrome are also seen in amyloidosis. The specific diagnosis requires
tissue biopsy with presence of amyloid with specific stains. In primary amyloidosis and myeloma, the amyloid protein is of the ALtype. In
reactive amyloidosis, the protein is of the amyloid Aprotein (AA) type.

• Cuales son los criterios mayores de Jones de CARDIOPATÍA REUMÁTICA?


1) CARDITIS: pericarditis, ICC, cardiomegalia y soplos.
2) ERITEMA MARGINADO
3) NÓDULOS SUBCUTÁNEOS: los nódulos subcutáneos SON INDOLOROS y menores de 2 cm
4) COREA DE SHYDENHAM: Movimientos coreoatetoides
5) ARTRITIS: dura de 1-5 semanas y responde a AINES

• Cuales son los criterios menores de cardiopatía reumática?


• R = Fiebre, VSG elevada, prolongación del PR.

• Cual es el manejo de cardiopatía reumática?


• R = PENICILINA BENZATINICA 1.2 millones DU o PEN-PRO 600, 000 x 10 dias.

• Como se previene la fiebre reumática?


• R = PEN-BEN 1.2 cada 4 semanas

• En los pacientes con Wolf-Parkinson-Withe. Que medicamentos están contraindicados?


• R = ADENOSINA, DIGOXINA, CALCIOANTAGONISTAS Y BETABLOQUEADORES YA QUE BLOQUEAN EL NODO AV.
• En los pacientes con Wolf-Parkinson-Withe. Cual es el manejo indicado?
• R = Cardioversión eléctrica y ablación por radiofrecuencia de vías accesorias.

• Cuales son los síndromes de preexitacion?


• R = Síndrome de WPW y Lown-Ganong-Levine

• Cual es el microorganismo causal de la ENFERMEDAD DE LYME?


• R = Borrelia Burgdorjeri

• Que características tiene la pericarditis secundaria a síndrome urémico?


• R = Pericardio afelpado, derrame hemorrágico y exudativo.

• Que caracteriza al síndrome de Dressler?


• R = Pericarditis post IAM o cirugía del corazón abierto y de etiología autoinmune.

• Como se define la hipertensión pulmonar primaria?


• R = Como aumento de la resistencia vascular pulmonar sin enfermedad subyacente.

• -Auscultation of the heart of a 17-year-old boy reveals an increased intensity of the pulmonary component of the
second heart sound. He complains of dyspnea on exertion but no other cardiac or pulmonary symptoms. Which of the
explanations is the most likely cause of his dyspnea?
• (A) pulmonary stenosis
• (B) aortic stenosis
• (C) MI
• (D) pulmonary hypertension
• (E) systemic hypertension
• Respuesta correcta D

• Cual es el manejo de hipertensión pulmonar primaria?


• R = CALCIOANTAGONISTAS, anticoagulantes, prostaciclina (potente vasodilatador pulmonar)
• Cual es el tratamiento de elección de la hipertensión primaria?
• R = Endotelina

• A que hace alusión el termino Cor-pulmonale?


• R = Hipertrofia ventricular derecha e insuficiencia por enfermedad pulmonar, mas frecuente en EPOC

• Cual es el dato de laboratorio mas frecuente en el Cor-pulmonale?


• R = Policitemia

• Cuales son los tumores que afectan al corazón?


• R = Carcinoma broncogenico, carcinoma mamario, melanoma maligno, linfomas, carcinoma de células renales y sarcoma de kaposi.

• Cuales son los tumores primarios del corazón mas frecuentes?


• R = MIXOMA AURICULAR el cual es un tumor benigno que puede embolizar sistémicamente y se ubica comúnmente en la aurícula derecha.

• -A 47-year-old woman has new-onset transient right arm weakness and word finding difficulty symptoms lasting 3 hours. She is also
experiencing exertional dyspnea, and had a syncopal event 1 month ago. Her echocardiogram reveals a cardiac tumor in the left atrium, it
is pendunculated and attached to the endocardium. Which of the following is the most likely cause of this lesion?
• (A) myxoma
• (B) sarcoma
• (C) rhabdomyoma
• (D) fibroma
• (E) lipoma
• Respuesta correcta A

• En que consiste la ENFERMEDAD DE BURGUER?


• R = Es un PROCESO INFLAMATORIO Y TROMBOTICO de las arterias y venas periféricas PRODUCIENDO CLAUDICACIÓN, DOLOR Y NECROSIS.

• Cuales son los datos clínicos mas frecuentes de la enfermedad de Takayasu?


• R = Soplos vasculares, PULSOS PERIFÉRICOS DISMINUIDOS y ASIMETRÍA DE LA PA DE LAS EXTREMIDADES.

• Cual es el tratamiento de la enfermedad de Takayasu?


• R = ESTEROIDES para la inflamación y después COLOCAR STENT O PUENTES.
• Cual es el tratamiento para pacientes con displasia fibromuscular de la intima?
• R = ASA

• Que fenómenos deben descartarse en una persona que haya cursado con enfermedad de Raynaud?
• R = 80% ESCLERODERMIA, MIOSITIS, LES, AR.

• Cual es el síndrome doloroso regional o complejo tipo 1 distrofia simpática refleja?


• R = DOLOR ARDOROSO O QUEMANTE de mas duración de lo esperado POR TRAUMATISMO EN EXTREMIDAD
secundario a aplastamiento o quemadura.

• Cuales son los datos clínicos de la distrofia simpática refleja?


• R = DOLOR LOCAL, hiperestesia, calor, RIGIDEZ MUSCULAR, RIGIDEZ ARTICULAR, con consiguiente DESUSO Y
OSTEOPENIA RADIOLÓGICA

• Que tratamientos se utilizan en distrofia simpática refleja?


• R = OPIOIDES Y GABAPENTINA

• Cual es el tipo mas común de síndrome de salida torácica?


• R = NEUROGENICO 90%, venoso y arterial.

• En el síndrome de salida torácica , que es el síndrome de PAGET-SCHROETTER O TROMBOSIS DE ESFUERZO?


• R = Se presenta como EDEMA UNILATERAL AGUDO DEL BRAZO, pesantez de axila, CIANOSIS DE LA MANO E
INGURGITACIÓN DE LAS VENAS DEL HOMBRO Y TÓRAX.

• Qué medidas se toman en un sujeto con choque que no responde a estímulos?


• R = DEXTROSA AL 50%, naloxona 2 ml iv. Manteniendo la diuresis horaria mayor de .5

• Que te indica la PVC?


• R = MENOR DE 5 mm hg indica HIPOVOLEMIA, MAYOR DE 18 SOBRECARGA VOLUMEN.
• Cuáles son los datos clínicos de taponamiento cardiaco?
• R = Taquicardia, HIPOTENSIÓN, pulso paradójico, AUMENTO DE LA PVY. Excepto en pacientes urémicos o con
hipotiroidismo.

• -A 25-year-old man complains of left precordial chest pain that radiates to the left shoulder but not down the
left arm. The pain is accentuated by inspiration and relieved by sitting up. The pain is accompanied by fever and
chills. His blood pressure is 105/75 mm Hg, pulse 110/min and regular, and temperature 37.5°C. Aside from the
tachycardia, there are no abnormal physical findings in the heart or lungs. The ECG shows STsegment elevation
in all leads except aVR and VI. On the third hospital day, the patient’s blood pressure falls, JVP rises, and he
goes into CHF. Which of the following is the most likely diagnosis?
• (A) a second pulmonary embolus
• (B) extension of a myocardial infarct
• (C) cardiac tamponade
• (D) secondary bacterial infection
• (E) rupture of a chordae tendineae
• Respuesta correcta C

• -A 56-year-old man presents with SOB, fatigue, and edema. He has also noticed weight gain, abdominal
discomfort, and distension. He has a prior history of lung cancer treated with radiotherapy to the chest. There
is no history of liver or cardiac disease in the past. On examination, he has an elvated JVP, prominent y descent
of neck veins, and positive Kussmaul’s sign. The heart sounds are normal. The CXR shows a normal cardiac
silhouette and the ECG has low voltages.

• -A 55-year-old woman with metastatic lung cancer presents with dyspnea and pedal edema. On examination,
the JVP is at 10 cm, with a negative Kussmaul’s sign. The heart sounds are diminished and the lungs have
bibasilar crackles. The ECG shows QRS complexes of variable height

• Como afecta el cliostazol en la enfermedad arterial periférica?


• R = Es un INHIBIDOR DE LA FOSFODIESTERASA 3 que disminuye la agregación plaquetaria, activa la lipoproteína
lipasa y causa vasodilatación.
• Cuál es el objetivo del tratamiento en un paciente con choque?
• R = PVC 8-12, PA ½ 65-90, índice cardiaco 2-4 lts/min, y O2 venoso > 70%.

• Cuáles son los efectos de la dopamina?


• R = 2-3 ug/kg/min estimula RECEPTORES DOPA y B AGONISTAS con AUMENTO DEL FG, FC Y IONOTROPISMO. >5 ug/kg/min
estimula receptores ALFA adrenergicos produciendo VASOCONSTRICCIÓN PERIFÉRICA.

• Cual es el medicamento de elección para choque cardiogenico?


• R = DOBUTAMINA con aumento del ionotropismo y disminución de la poscarga

• Que efecto tiene la desmopresina utilizada en choque distributivo?


1) Vasoconstricción periférica, disminución de la FC, vasodilatación coronaria, pulmonar y cerebral.
2) La vasoconstricción es mediada por oxido nítrico.

• Cual es la causa de la miocardiopatía hipertrófica?


• R = DEFECTO CONGÉNITO en los sarcomeros.

• Cual es la causa de miocardiopatía dilatada?


• R = Isquemia, cardiotoxicos. EMBARAZO.

• Cual es la causa de la miocardiopatía restrictiva?


• R = Primaria: fiebre endomiocardica, endocarditis de coffler o ideopatica, Secundaria. RADIACION.

• -A 55-year-old woman is recently diagnosed with amyloidosis. She is now noticing increasing SOB, fatigue, and edema. On
examination, the JVP is at 10 cm with a negative Kussmaul’s sign but prominent x and y descent. The blood pressure is 90/70
mm Hg, no pulsus paradoxus, pulse 100/min with low volume, and normal heart sounds.

• -A 60-year-old man presents with SOB, increasing abdominal distention, and lower leg edema. He has no prior history of
cardiac, renal, or liver disease. On examination, the JVP is at 8 cm with a negative Kussmaul’s sign but prominent x and y
descent. The blood pressure is 95/75 mm Hg, no pulsus paradoxus, pulse 100/min with low volume, and normal heart sounds.
There is shifting dullness of the abdomen and pedal edema. His blood glucose and hemoglobin A1C are elevated.
• Que medicamentos están contraindicados en miocardiopatía restrictiva?
• Digitalicos y agonistas B adrenérgicos.

• Cuales son los criterios para ENDOCARDITIS BACTERIANA DE DUKE.

• CRITERIOS MAYORES:
• 1. Hemocultivos positivos para EI
1.1. Microorganismos típicos de EI en dos hemocultivos separados
1.1.1 ESTREPTOCOCO VIRIDANS
S. BOVIS
HACEK
1.1.2. S. Aureus o Enterococus adquiridos en la comunidad en ausencia de foco primario
1.2. Hemocultivos persistentes positivos
1.2.1. Hemocultivos extraidos con más de 12 horas de separación
1.2.2. La totalidad de tres, o la mayoría de cuatro o más hemocultivos separados
siempre que entre el primero y el último haya al menos una hora

• 2. Evidencia de afectación miocárdica


2.1. Ecocardiograma positivo
2.1.1. VEGETACIÓN EN VÁLVULA O ESTRUCTURAS ADYACENTES o en el choque del jet, o
sobre dispositivos protésicos en ausencia de otra explicación anatómica
2.1.2. Absceso
2.1.3. Nueva dehiscencia parcial de una válvula protésica
2.2. Nueva regurgitación valvular (incremento o cambio en un soplo preexistente no
es suficiente)
• CRITERIOS MENORES
1. Predisposición. Una cardiopatía predisponente o ser ADVP.
2. FIEBRE > 38ºC
3. Fenómenos vasculares: émbolos en arterias mayores, infartos pulmonares, sépticos, aneurismas micóticos,
hemorragia intracraneal, hemorragia conjuntival y LESIONES DE JANEWAY
4. Fenómenos inmunológicos (glomerulonefritis, NÓDULOS DE OSLER, MANCHAS DE ROTH Y factor
reumatoide).
5. Ecocardiograma: sugestivos de eI sin alcanzar los criterios mayores antes mencionados.
6. Evidencia microbiológica (hemocultivos positivos que no cumplen los criterios mayores) o evidencia
serológica de infección activa con un microorganismo que produce EI.
• -A 28-year-old man develops viridans group streptococci septicemia. Which of the following cardiac lesions has
the highest risk of developing endocarditis?
• (A) ventricular septal defect
• (B) atrial septal defect, secundum type
• (C) mitral valve prolapse with regurgitation
• (D) pure mitral stenosis
• (E) asymmetric septal hypertrophy
• Respuesta correcta A

• En que consiste la PERICARDITIS CONTSTRICTIVA?


• R = Constrictive pericarditis is characterized by a prominent y descent of the neck veins and LOW VOLTAGE ON
ECG. THE PRESENCE OF A POSITIVE KUSSMAUL’S SIGN HELPS DIFFERENTIATE THE SYNDROME FROM COR
PULMONALE AND RESTRICTIVE CARDIOMYOPATHIES. Apericardial knock is characteristic of constrictive
pericarditis. It is in fact an early S3, occurring 0.06–0.12 seconds after aortic closure. S1 and S2 are frequently
distant.

• -A 56-year-old man presents with SOB, fatigue, and edema. He has also noticed weight gain, abdominal
discomfort, and distension. He has a prior history of lung cancer treated with radiotherapy to the chest. There
is no history of liver or cardiac disease in the past. On examination, he has an elvated JVP, prominent y descent
of neck veins, and positive Kussmaul’s sign. The heart sounds are normal. The CXR shows a normal cardiac
silhouette and the ECG has low voltages.
• -A 64-year-old presents with dyspnea and edema. He had previous coronary
bypass surgery 5 years ago, which was uncomplicated. Since then he has
had no further chest pain. On examination, his JVP is at 8 cm, with
prominent Kussmaul’s sign. The heart sounds are easily heard but there is
an early diastolic filling sound (pericardial knock).

• Cuál es el diagnostico diferencial entre miocardiopatia constrictiva y


pericarditis restrictiva?
1) PERICARDITIS RESTRICTIVA: Puede ser causada por tuberculosis, Rx del
tórax y cirugía cardiaca.
2) PERICARDITIS CONSTRICTIVA: Causada por un tamponade.
3) MIOCARDIOPATIA CONSTRICTIVA: Lo causa la amiloidosis,
hemocromatosis, sarcoidiosis, esclerodermia, carcinoide.
REUMATOLOGIA

ENARM
ARTRITIS CRONICA JUVENIL
ESPONDILITIS ANQUILOSANTE
• Que espondiloartropatias se asocian al HLA-B27?
 R = ESPONDILITIS ALQUILOSANTE, ARTRITIS REACTIVA, ESPONDILOARTROSIS
PSORIASICA, uveítis anterior aguda.

• Cual es el síntoma inicial de la espondilitis alquilosante (EA) y que hallazgos son frecuentes en la ex fis?
 R = DOLOR en regiones SACROILIACAS Y LUMBARES. En la ex fis HAY DISMINUCIÓN DE LA
MOVILIDAD DE LA COLUMNA LUMBAR, dolor al presionar las regiones sacroiliacas y disminución de la
expansión del tórax.

• Cual es la manifestación extrarticular mas frecuente de la EA?


 R = UVEITIS ANTERIOR, 2/3 de px tiene CAMBIOS EN LA MUCOSA Y SUBMUCOSA DEL COLON E ÍLEON
TERMINAL

• Cuales son los hallazgos más comunes en imagen obtenidas en la espondilitis anquilosante?
 R = Erosiones y esclerosis sacroiliacas, IMAGEN EN COLUMNA DE BAMBU

• Cual es el manejo de la EA?


 R = AINES, COX-2, Infliximab, Etanercept mas terapia física. Indometacina, INFLIXIMAB que disminuye el FNTa.
ENFERMEDAD DE REITER
• Cuales son los datos clinicos más comunes de ARTRITIS REACTIVA
o enfermedad de Reiter y con que HLA se relaciona?
 R = Ligado al HLA B27, oligoartritis, CONJUNTIVITIS, URETRITIS,
CERVICITIS, ULCERAS ORALES x un CUADRO INFECCIOSO PREVIO.

• Que situación precede a una manifestación de ARTRITIS


REACTIVA o síndrome de Reiter y cual es el patógeno > común?
 R = Alguna INFECCIÓN por SALMONELLA, SHIGELLA, YERSINIA,
CLAMYDIA TRACHOMATIS +++ o
CAMPYLOBACTER que PRECEDE a la MANIFESTACIÓN ARTICULAR
3-4 semanas después.

• Cual es el estudio de laboratorio o gabinete de elección del


síndrome de Reiter?
 R = CULTIVO CON ANTIBIOGRAMA, se puede encontrar también Ac
anti bacteriano en el suero o liquido sinovial o DNA bacteriano.
ENFERMEDAD DE STILL
• Que caracteriza clínicamente a la enfermedad de Still y que la precede?
 R = VARIANTE DE LA ARTRITIS REUMATOIDE en la cual hay FIEBRE DE
MAS DE 40 y con precipitación posterior varios grados debajo,
ODINOFAGIA Y POLIADENOPATIAS. LO PRECEDE
FARINGITIS.
• Cual es una manifestación clínica característica de la enfermedad de Still?
 R = EXANTEMA NO PRURIGINOSO, maculopapular de COLOR SALMÓN
en TÓRAX Y ABDOMEN. (EL CUAL NO HAY EN REITER)

• Cuales son los laboratorios distintivos de la enfermedad de Still, que


inmunoglobulina se eleva y como afecta al FR?
 R = LEUCOCITOSIS > 40, VSG AUMENTADA, hipergamaglobulinemia IgG y anemia.
NO se detectan AUTOANTICUERPOS NI FR.

• Cual es el manejo de la enfermedad de Still?


 R = AINES
ARTRITIS SORIASICA
• Que es la artritis soriasica?
 R = Ligado al HLA B27. Se define como una ARTRITIS usualmente
SERONEGATIVA que se ASOCIA A SORIASIS.

• Cual es el cuadro clínico de la artritis soriasica?


 R = PRECEDE PSORIASIS A ARTRITIS, ARTRITIS ASIMÉTRICA con
DEDOS EN APARIENCIA DE SALCHICHA (DACTILITIS) de los dedos de manos
y pies.

• Como diagnosticas artritis soriasica?


 R = HC, y RX con evidencia de SACROILEITIS ASIMÉTRICA, alteraciones de la
conformación de hueso nuevo con IMAGEN DE PUNTA DE LÁPIZ EN UNA
COPA.

• Cual es el manejo de la artritis soriasica?


 R = AINES E INFLIXIMAB. Tratamiento de soriasis.
RESUMEN ESPONDILOARTROPATIAS
SERONEGATIVAS
LUPUS ERITEMATOSO SISTEMICO
• Que HLA tienen relación con el LES?
1) HLA B8, HLA DR2 y DR3.
2) Los alelos de HLA están en el brazo corto del CROMOSOMA 6.

• Que fármacos inducen la producción de anticuerpos antinucleares y LES?


 R = HIDRALAZINA, procainamida, clorpromazina, ISONIAZIDA y diversos anticonvulsivantes.

• Cual es el cc del LES en cada aparato – sistema y con que padecimientos se relaciona?
 1. Se relaciona con: PTI, ANEMIA HEMOLITICA Y TIROIDITIS.
 2. Constitucionales: la FATIGA es lo mas común
 3. Mucocutaneas: 80%, LESIONES DISCOIDES observándose en cara, pabellones auriculares y el
AREA DE V DEL ESCOTE. Eritema malar o en ALAS DE MARIPOSA QUE ES FOTOSENSIBLE. EL
neonatal QUE SE PRESENTA EN LOS HIJOS DE PX CON LES POR EL PASO DE IgG
CARACTERIZÁNDOSE POR LESIONES CUTÁNEAS Y BLOQUEO CARDIACO CONGÉNITO EN
LOS PRIMEROS 6 MESES DE EDAD. ALOPECIA.
 4. Musculoesqueletico: ARTRALGIAS con artritis. Osteonecrosis en la cabeza femoral, humeral,
cóndilos femorales, carpo, tarso manifestándose clínicamente con dolor que mejora con el reposo.
 5. Renal: nefritis lupica siendo la mas común la TIPO IV ¨GLOMERULONEFRITIS PROLIFERATIVA
DIFUSA¨.
 6. SNC: Meningitis aséptica, EVC, síndromes desmielinizantes y cefaleas.
 7. Cardiopulmonar: En corazón con VEGETACIONES ENDOCARDICAS de LIBMAN-SACKS, en
pleura lo mas común es pleuritis con o sin derrame pleural con niveles altos de proteínas DHL,
complejos inmunes, ANA y células LE. LA VALVULA MAS AFECTADA ES LA MITRAL.
 8. Gastrointestinal: Se puede presentar con SEROSITIS, disfagia y dispepsia muy a menudo.
 9. Hematológico: PANCITOPENIA Y >TTP. Anemia por deficiencia de hierro MICROCITICA-
HIPOCROMICA.
ESCLERODERMIA
SX SJOGREN
POLIARTERITIS NODOSA
• Cuales son los signos y síntomas MAS COMUNES de poliarteritis nodosa?
 R = DOLOR EN EXTREMIDAD POR ARTRALGIA, MIALGIA QUE AFECTA
PANTORRILLAS, puede afectar riñón causando HAS, IRA O HEMORRAGIA
POR MICROANEURISMAS. MONONEURITIS MÚLTIPLE
(PIE CAÍDO), corazón con IAM. Livides reticular, nódulos subcutáneos, y ulceras en
piel. Son necesarios 3 de 10 criterios para su diagnostico.

• Cuales son las pruebas de laboratorio en la poliarteritis nodosa y con que otro
virus se relaciona habitualmente?
 R = DESCARTAR VHB QUE SE ASOCIA EN 20-30%. ANCA (-) Y ANA (-) .Anemia,
leucocitosis y trombocitosis. Se confirma diagnostico con toma de BIOPSIA DE LA
LESIÓN.

• Que datos se obtienen al tomar la biopsia en la poliarteritis nodosa?


 R = BIOPSIA DE ARTERIA de mediano o pequeño calibre con PRESENCIA DE
INFILTRADO GRANULOCITICO, PMN Y CÉLULAS MONONUCLEARES EN LA PARED DEL
VASO.

• Cual es el manejo de la poliarteritis nodosa?


 R = ESTEROIDES, pero SI SE ASOCIA CON VHB SE AGREGAN RETROVIRALES.
ARTERITIS DE LA TEMPORAL
VASCULITIS
ARTRITIS SEPTICA PIOGENA
• Cuales son los factores de riesgo para la artritis séptica piógena (gonocócica o no)?
 R = Abuso de drogas IV, jóvenes sexualmente activos, enfermedades concomitante como DM, AR o lupus,
antecedentes de cirugía o prótesis.

• Cual es el cc de la artritis séptica piógena gonocócica o no?


 R = Es una entidad de inicio agudo y progresivo caracterizado por dolor e inflamación local (salvo en el primer
año de vida donde es similar a la sepsis). El dolor es moderado a intenso con limitación funcional, aumento de
la temperatura local y rubor es monoarticular, cuando es poliarticular es en paciente inmunocomprometido
siendo la rodilla la mas afectada.

• Cuales son los hallazgos radiológicos en la artritis séptica piógena?


 R = Edema de tejidos blandos y derrame articular. EN AFECCIÓN POR ANAEROBIOS SE VE GAS EN TEJIDOS
BLANDOS. El gamagrama óseo con tecnecio (valora flujo) o con galio (valora actividad inflamatoria) pueden
utilizarse de manera temprana.

• Cual es el manejo de la artritis séptica piógena?


1) Consiste en la administración de antibióticos y drenaje de la articulación afectada. En caso de ARTRITIS
PIÓGENA debe administrarse de 3 A 4 SEMANAS y en caso de ARTRITIS GONOCÓCICA DE 7 A 14 DÍAS.
2) En menores de 2M: DICLOXACILINA mas CEFOTAXIMA
3) En LACTANTES: DICLOXACILINA mas CEFOTAXIMA
4) En niños MAYORES DE 3ª: DICLOXACILINA
5) Artritis séptica GONOCÓCICA: 1g por via IM o IV cada 24 hrs.
6) Artritis séptica en DROGADICTOS por via IV: DICLOXACILINA mas GENTAMICINA
7) Artritis séptica en INMUNOCOMPROMETIDOS: CEFTAZIDIMA mas AMIKACINA
8) Artritis séptica POSTOPERATORIA: VANCOMICINA mas AMIKACINA
9) Artitis séptica en FRACTURA ABIERTA: AMOXICILINA con ACIDO CLAVULANICA
ARTRITIS SEPTICA PIOGENA
RESUMEN DE LAS ENFERMEDADES
METABOLICAS OSEAS
CPK
• Las siguientes patologías cursan con aumento
de la CPK
1) HIPOTIROIDISMO
2) RAZA NEGRA
3) POLIMIOSITIS Y
4) DERMATOMIOSITIS
GASTROENTEROLOGIA

ENARM
ACALASIA
• Que es la acalasia?
 R = Es un trastorno por motilidad inadecuada que se caracteriza por una RELAJACIÓN INCOMPLETA y
AUMENTO DE LA PRESIÓN BASAL DEL EEI, además de presentar APERISTALSIS DEL CUERPO.

• Que causa la acalasia?


 R = Es IDIOPÁTICO, aunque una de las hipótesis indica que es de origen central con DESTRUCCIÓN
PARCIAL DEL NÚCLEO DORSAL DEL VAGO o sus fibras nerviosas a nivel del esófago con la siguiente
alteración de la función.

• Cuales son los síntomas más comunes de la acalasia?


 R = DISFAGIA EN EL 90% DE LOS CASOS, INICIALMENTE A SÓLIDOS Y DESPUÉS A LÍQUIDOS. El segundo
síntoma mas frecuente es la REGURGITACIÓN EN 75% mas acentuada en decúbito, EL DOLOR 20-
30% ES MENOS COMÚN, DE LOCALIZACIÓN EPIGÁSTRICA, RETROESTERNAL, E IRRADIADO HACIA
CUELLO, DORSO Y EXTREMIDADES SUPERIORES.

• Cuales son los estudios de gabinete necesarios para el diagnostico de acalasia?


 R = La MANOMETRIA ES DE ELECCIÓN la cual nos muestra una PRESIÓN DEL EEI AUMENTADA (>69
MM/HG), SIN PRESENCIA DE ONDAS PERISTÁLTICAS A LA DEGLUCIÓN. La ESOFAGOGRAFIA CON BARIO
MUESTRA ALTERACIONES RELACIONADAS CON LAS ONDAS PERISTÁLTICAS Y LA AUSENCIA DE
RELAJACIÓN DEL EEI DANDO UNA IMAGEN EN PICO DE PÁJARO O EN PUNTA DE LÁPIZ característica.

• Cual es el manejo de la acalasia?


1) MEDICO: A base de DILATACIONES HIDROSTÁTICAS O NEUMÁTICA, además de LA INYECCIÓN CON
TOXINA BOTULINICA.
2) Quirúrgico: MIOTOMIA DE HELLER con una tasa de curación del 95%.
ESPASMO ESOFAGICO DIFUSO
• Que es el espasmo esofágico difuso?
 R = Se trata de un trastorno motor esofágico que afecta fundamentalmente a la musculatura lisa y que se caracteriza
por LA APARICIÓN DE ACTIVIDAD CONTRÁCTIL NO PROPULSIVA que alterna con episodios de peristalsis normal.

• Cual es el cuadro clínico del espasmo esofágico difuso?


 R = DOLOR TORAXICO CON LOCALIZACIÓN fundamentalmente RETROESTERNAL, aunque a veces SE IRRADIA A
CUELLO Y A LOS HOMBROS SIMULANDO UNA ENFERMEDAD CARDIACA aumentando cuando están bajo estrés
sicológico o emocional. La DISFAGIA es tan frecuente o mas que el dolor torácico. También es intermitente SE
DESENCADENA CON FRECUENCIA CON LA TOMA DE SÓLIDOS O BEBIDAS FRÍAS.
• Cuales son las alteraciones histológicas que esperas encontrar en el espasmo esofágico difuso?
 R = HIPERPLASIA DE LAS CÉLULAS musculares y ocasionalmente mínimas ALTERACIONES a nivel de las fibras nerviosas
terminales del PLEXO DE AUERBACH, no existiendo alteraciones significativas en las células ganglionares y nervios o en
todo caso estas son mínimas.

• Cuales son los hallazgos al gabinete en el espasmo esofágico difuso?


 R = ESOFAGOGRAMA: Que nos muestra hallazgos característicos con APARICIÓN DE FRECUENTES ONDAS NO
PROPULSIVAS en el musculo liso esofágico QUE INDENTAN LA COLUMNA DE BARIO y retrasan su evacuación, lo
cual se describe como ESÓFAGO EN SACACORCHOS. MANOMETRIA: Alteraciones segmentarias, apareciendo
generalmente en los DOS TERCIOS INFERIORES DEL CUERPO ESOFÁGICO. PHmetria obligatoria para descartar ERGE
como causa del espasmo.

• Cual es el manejo MEDICO Y QUIRURGICO del espasmo esofágico difuso?


• MEDICO: NIFEDIPINO, ANTIDEPRESIVOS Y ANSIOLÍTICOS.
• QUIRURGICO: La ESOFAGOMIOTOMIA AMPLIADA hasta el cayado aórtico puede ser una alternativa muy acertada EN
CASOS GRAVES REFRACTARIOS a cualquier otro tratamiento.
PERISTALSIS ESOFÁGICA SINTOMÁTICA O ESÓFAGO EN CASCANUECES
• Que es la peristalsis esofágica sintomática o esófago en cascanueces y con que se relaciona?
 R = Trastorno de la motilidad esofágica que se caracteriza por la PRESENCIA DE CONTRACCIONES DE
GRAN AMPLITUD, SIN ALTERACIONES EN EL PERISTALTISMO del cuerpo esofágico y relajación del
esfínter esofágico inferior. Se RELACIONA CON ERGE y a la histología no presenta alteraciones.

• Cual es el cuadro clínico de la peristalsis esofágica sintomática o esófago en cascanueces?


 R = DOLOR TORÁCICO RECURRENTE y con menor frecuencia disfagia.

• Cuales son los hallazgos de gabinete en la peristalsis esofágica sintomática o esófago en cascanueces
en la manometria o en el esofagograma?
 R = El diagnostico se establece por la MANOMETRIA y se caracteriza por la aparición de ONDAS DE
GRAN AMPLITUD (180 MM/HG), QUE OCASIONALMENTE TAMBIÉN SON DE LARGA DURACIÓN
(6SEG), siendo condición indispensable que EL PERISTALTISMO ESOFÁGICO SEA NORMAL.
Siempre se debe descartar ERGE realizando una PHmetria. NO HALLAZGOS CON ESOFAGOGRAMA.

• Cual es el manejo de la peristalsis esofágica sintomática o esófago en cascanueces?


1) MEDICO: Si existe ERGE se recomiendo uso de OMEPRAZOL y evitar MIORRELAJANTES (nifedipino,
diltiazem o nitratos) POR QUE EMPEORARÍAN EL ERGE. BENZODIAZEPINAS Y SOBRE TODO LOS
ANTIDEPRESIVOS TRICICLICOS parecen útiles.
2) QUIRURGICO: ESOFAGOMIOTOMIA AMPLIADA HASTA EL CAYADO AÓRTICO puede ser una alternativa
en pacientes intratables. La inyección de toxina botulinica tiene resultados muy pobres.
EEI HIPERTENSO
• Como se define el EEI hipertenso?
 R = Como la existencia DE UN TONO O PRESIÓN media máxima basal DEL
EEI IGUAL O SUPERIOR A DOS DEVIACIONES ESTÁNDAR, mas de 40-45
mm/Hg, siendo además condicionantes indispensables que existan
RELAJACIONES COMPLETAS DEL EEI y que EL PERISTALTISMO ESOFÁGICO
SEA NORMAL.

• Cual es el cuadro clínico del EEI hipertenso?


 R = DOLOR TORÁCICO O DISFAGIA. Mejorando generalmente el dolor tras el
tratamiento, siendo la disfagia la mas difícil de tratar.

• Cual es el manejo del EEI hipertenso?


 R = RELAJANTES DE LA MUSCULATURA LISA
SX DUMPING
CÁNCER GÁSTRICO
• Cual es la estirpe etiológica mas común del cáncer gástrico?
 R = ADENOCARCINOMA.

• Cual es el cuadro clínico de un paciente con cáncer gástrico?


 R = PERDIDA DE PESO, dolor abdominal, SACIEDAD TEMPRANA. Generalmente se presentan
con palidez. Entre las manifestaciones dermatológicas mas frecuentes se encuentran la
ACANTOSIS NIGRICANS y SIGNO DE LESER-TRELAT (queratosis seborreica de aparición
súbita)

• Como se manifiesta la metastasis comúnmente el cáncer gástrico?


 R = GANGLIOS EN SUPRACLAVICULAR (Virchow), GANGLIOS PERIUMBILICALES (Hermana
Maria Jose), estructuras rectales (signo del anaquel de blúmer), OVARIOS (tumor de
Krukenberg).

• Cual es el estándar de oro para el diagnostico del cáncer gástrico?


 R = PANENDOSCOPIA con toma de 7 BIOPSIAS de la lesión y después TAC PARA
ESTADIFICACION en caso de metástasis.

• Cual es el manejo del cáncer gástrico?


 R = Quirúrgico. La gastrectomía total se recomienda en caso de lesiones en el estomago
proximal. La QUIMIOTERAPIA no mejora la sobrevida sobre la cirugía, sin embargo
COMBINADA CON LA RADIOTERAPIA si la mejora.
LINFOMA GASTRICO
• 60. Cual es la estirpe etiológica mas común en el linfoma gástrico?
 R = No HODKING 95%

• 61. Cual es el cuadro clínico del linfoma gástrico?


 R = Sintomatología no especifica como DOLOR ABDOMINAL, nausea,
vomito, PERDIDA DE PESO, SUDORES NOCTURNOS y diarrea.

• 62. Cual es la apariencia del linfoma gástrico a la endoscopia?


 R = Aparecen como LESIONES FUNGOIDES, PÓLIPOS, ENGROSAMIENTO DE
PLIEGUES.

• 63. Cual es el manejo del linfoma gástrico?


 R = Quirúrgico
MALABSORCION
• Cuales son los datos clínicos de malabsorción?
 R = Diarrea, DISMINUCIÓN DE PESO y desnutrición, acompañados de
DISTENCIÓN ABDOMINAL, síntomas y signos secundaros a los déficit
específicos, ESTEATORREA.

• Que datos de laboratorio se encuentran en un paciente con malabsorción?


 R = Hipocolesterolemia, HIPOALBUMINEMIA, alteraciones electrolíticas y PH.

• Cual es la posología de la prueba con D-Xilosa en malabsorción?


 R = Se da EN AYUNAS midiendo su nivel en suero 1hr después y la excreción en
orina en 6hr, este azúcar SE ABSORBE EN FORMA PASIVA SI LA MUCOSA
INTESTINAL ESTA INTEGRA.

• Cuales son las principales causas de malabsorción?


 R = DEFICIENCIA DE LACTASA, ESPRUE celiaco, ESPRUE TROPICAL,
ENFERMEDAD DE WIPPLE, sobre crecimiento bacteriano y síndrome de
intestino corto.
ESPRUE CELIACO
• Que es el esprue celiaco?
 R = Causa malabsorción debido a lesión ocasionada por el GLUTEN a nivel intestinal, el cual
es contenido en el trigo, CEBADA, CENTENO Y AVENA.

• Cual es el cuadro clínico del esprue celiaco?


 R = Hay malabsorción de CHBTS, PROTEÍNAS Y GRASAS. Perdida progresiva de peso,
distensión abdominal, flatulencia y diarrea, evacuaciones voluminosas y de mal olor.

• Cuales estudios de diagnostico hay para el esprue celiaco y cuales son los anticuerpos
especificos?
1) La biopsia en LA UNIÓN DUODENOYUYENAL que suele REPORTAR ATROFIA O
ACORTAMIENTO DE LAS VELLOSIDADES, HIPERPLASIA DE LAS CRIPTAS y aumento en la
mitosis CON INCREMENTO DE LINFOCITOS, NEUTROFILOS, MASTOCITOS Y EOSINOFILOS EN
LAMINA PROPIA.
2) Cuantificación de grasa en heces de 24 hrs mayor a 7g/dia , la ABSORCIÓN DE D-XILOSA
SUELE SER ANORMAL, los ANTICUERPOS SÉRICOS ANTIGLIADINA SON POSITIVOS EN 90%,
el anticuerpo IgA es el mas especifico.
3) Los anticuerpos IgA ANTIENDOMISIO Y ANTIRRETICULINA son los
MAS ESPECÍFICOS.

• Cual es el manejo del esprue celiaco?


 R = Dieta libre de gluten
ESPRUE TROPICAL
• Cual es la etiología del esprue tropical?
 R = Es producido por la COLONIZACIÓN PERSISTENTE del intestino delgado por KLEBSIELLA
PNEUMONIAE, ENTEROBACTER CLOACAE o E. COLI.

• Cual es el cuadro clínico del esprue tropical?


1) CRISIS DE DIARREA ACUOSA, acompañada de DOLOR ABDOMINAL y FLATULENCIA, los
cuales disminuyen de intensidad a la semana de evolución y posteriormente se hacen
crónicos.
2) ANEMIA MEGALOBLASTICA.

• Cuales son los estudios diagnósticos del esprue tropical?


1) La biopsia de intestino delgado reporta ALARGAMIENTO DE LAS CRIPTAS, ensanchamiento
y acortamiento de las vellosidades e INFILTRACIÓN DE LA LAMINA PROPIA POR CÉLULAS
INFLAMATORIAS.
2) Se encuentra esteatorrea, ABSORCIÓN DE D-XILOSA ANORMAL.

• Cual es el manejo del esprue tropical?


 R = TETRACICLINA y ACIDO FÓLICO para mejorar ANEMIA MEGALOBLASTICA.
ENFERMEDAD DE WHIPPLE
• Cual es el agente causal de la enfermedad de Whipple y con que HLA se relaciona?
 R = Es secundaria a la infección por TROPHERYMA WHIPELLII y se relaciona con HLA B27 + en 25%.

• Cual es el cuadro clínico de la enfermedad de Whipple?


 R = Perdida de peso, diarrea o esteatorrea, artralgias, FIEBRE, dolor abdominal y HEMORRAGIA
INTESTINAL, LAS ARTRALGIAS SON MIGRATORIAS, no destructivas y afectan las grandes
articulaciones, se presenta SACROILEITIS EN 25%. Un signo característico es la MIORRITMIA
OCULOMASTICATORIA, la cual consiste en la convergencia rítmica de los ojos asociada a
contracciones sincrónicas de los músculos de la masticación. A nivel ocular se manifiesta como
UVEÍTIS, vitreitis, retinitis y papiledema. A la ex fis se encuentra LINFADENOPATIA
periférica.

• Cuales son los hallazgos de laboratorio en la enfermedad de Whipple?


 R = La biopsia de intestino delgado muestra MACRÓFAGOS POSITIVOS CON LA TINCIÓN PAS. Sin
embargo se debe demostrar el AGENTE CAUSAL EN EL LCR, tejido cerebral o GANGLIOS LINFÁTICOS.
Anemia secundaria a enfermedad crónica, deficiencia de hierro, folato o VB12, asociada a neutrofilia.
Prueba de D-Xilosa anormal, LINFADENOPATIA ABDOMINAL detectada por ultrasonido.
Radiografía de TORAX CON INFILTRADOS, FIBROSIS O DERRAME PLEURAL.

• Cual es el manejo de la enfermedad de Whipple y por cuanto tiempo se realiza?


 R = TMP/SMZ durante 1ª, si hay ALERGIA A SULFAMIDAS esta indicada la PENICILINA por vía oral
durante el mismo periodo y se deben tratar los trastornos nutricios.
SX COLON IRRITABLE
• Cuales son los criterios de ROMA II para integrar el síndrome de colon irritable?
 R = Que los síntomas hallan durado POR LO MENOS 12 SEMANAS (no necesariamente
continuas) durante los 12m previos de malestar o dolor abdominal que tiene 2 de las
siguientes características. Se dice que el síndrome de intestino irritable se diagnostica
cuando cumple al menos con 1 o mas de los criterios 2, 4 y 6 o bien 1, 3 y 5 DEPENDIENDO
DE SI ES DIARREA LA QUE PREDOMINA O CONSTIPACIÓN. Se exacerba con el estrés y es
MAS FRECUENTE EN MUJERES.

1. Menos de 3 evacuaciones por semana


2. Mas de 3 evacuaciones al día
3. Evacuaciones de consistencia dura
4. Evacuaciones sueltas o duras
5. Pujo al evacuar
6. Urgencia al evacuar
7. Sensación de evacuación completa
8. Moco durante la evacuación
9. Distención abdominal

• Cual es el manejo del síndrome de colon irritable?


 R = Debe ser INDIVIDUALIZADO a los síntomas. PInaverio con dimeticona.
POLIPOS DEL COLON
• Cual es la clasificación de los pólipos neoplásicos de colon?
 R = A)TUBULARES 75% que corresponde a ADENOMAS. B) VELLOSOS 10% de los adenomas, mas
relacionado con cáncer colonico y C) TUBULOVELLOSOS 15% de los adenomas. Requiriéndose un periodo
mínimo de 10ª para que un adenoma se convierta a cáncer.

• Cuales son las alteraciones genéticas implicadas en los adenomas y el cáncer de colon?
 R = Mutación del GEN RAS y lesiones de los cromosomas 5, 17 y 18.

• Cual es el cuadro clínico de un paciente con SÍNDROME DE PEUTZ-JEGHERS?


 R = Es un TRASTORNO AD caracterizado POR MANCHAS HIPERPIGMENTADAS en los labios, mucosa bucal, cara
y dedos, PRESENTANDO HAMARTROMAS EN TODO EL TUBO DIGESTIVO e INTUSUCEPCION CON STDB.

• Cual es el manejo del Síndrome de Peutz-Jeghers?


 R = Resección de pólipo sintomático por endoscopia

• Cual es el cuadro clínico de la poliposis juvenil difusa?


 R = Es un TRASTORNO AD caracterizado por una población homogénea de pólipos, tanto hamartromas como
adenomas, los pólipos pueden ocasionar hemorragia e intususcepción en este síndrome. 10% RIESGO DE
DESARROLLO DE CA DE COLON.

• Cual es el manejo de la poliposis juvenil difusa?


 R = RESECCIÓN del colon con y anastomosis ileorectal. SEGUIMIENTO CON PROCTOSCOPIA CADA 6M y
extirpación de cualquier pólipo nuevo.
• Cual es el cuadro clínico del SÍNDROME DE COWDEN?
1) Enfermedad AD caracterizada por HAMARTROMAS en todo el tubo digestivo, PAPULAS FACIALES Y
BUCALES, CRECIMIENTOS QUERATOSICOS EN MANOS Y PIES.
2) Estos pacientes pueden presentar cáncer de mama, tiroides o de útero.

• Cual es el cuadro clínico del SÍNDROME DE CRONKHITE-CANADA?


 R = NO ES HEREDITARIO y se caracteriza por HAMARTROMAS INTESTINALES generalizados, ALOPECIA,
PIGMENTACIÓN CUTÁNEA y ATROFIA DE NAILS. Los SÍNTOMAS INCLUYEN VOMITO, DIARREA,
MALABSORCIÓN y enteropatía con perdida de proteínas, LA MAYORÍA DE LOS PACIENTES MUERE POCO
DESPUÉS DEL DIAGNOSTICO.

• Cual es el cuadro clínico de la poliposis adenomatosa familiar?


 R = TRASTORNO AD y su defecto se localiza en el CROMOSOMA 5. Se caracteriza por la presencia de mas de
100 POLIPOS en todo el colon y recto, 100% DESARROLLA CÁNCER.

• Cual es el manejo de la poliposis adenomatosa familiar?


 R = Resección con RECTOCOLECTOMIA total con ileostomía.

• Cual es el cuadro clínico de la ENFERMEDAD DE GARDNER?


 R = POLIPOSIS ADENOMATOSA familiar que se asocia a OSTEOMIELITIS, QUISTES EPIDERMOIDES Y
FIBROMAS DE LA PIEL.

• Que es el SÍNDROME DE TURCOT?


 R = POLIPOSIS ADENOMATOSA FAMILIAR que se asocia con NEOPLASIAS DEL SNC.

• Como se manifiesta un Adenocarcinoma de colon ubicado en el lado derecho?


 R = Perdida de peso, HEMATOQUZIA, anemia por déficit de hierro y UNA MASA ABDOMINAL EN EL CID.

• Como se manifiesta un Adenocarcinoma de colon ubicado en el lado izquierdo?


 R = Perdida de peso, cambios en hábitos intestinales, RECTORRAGIA y dolor abdominal tipo cólico. Se debe
realizar una colonoscopia con biopsia.
TROMBOSIS MESENTERICA
• Que pacientes tienen el riesgo de desarrollo de trombosis venosa
mesentérica?
 R = Pacientes traumatizados, hipertensión portal, peritonitis o con
hipercoagubilidad

• Cual es el cuadro clínico de la trombosis venosa mesentérica?


 R = DOLOR ABDOMINAL DIFUSO, DISTENCIÓN ABDOMINAL, nauseas,
vómitos y signos de deshidratación

• Cual es el estándar de oro para el dx de la trombosis venosa mesentérica?


 R = TAC EN ESPIRAL con medio de contraste oral e IV

• Cual es el manejo de la TVM?


 R = ANTI COAGULACIÓN E HIDRATACIÓN.
STDB
TIPS STDA Y STDB
o Los pacientes que requieran mas de 10u de paquetes globulares deberán recibir
también PLASMA FRESCO CONGELADO, PLAQUETAS O AMBOS.

o La colocación de BANDAS ELÁSTICAS es el tratamiento de elección en la rotura de


VARICES ESOFÁGICAS.

o El LAVADO NASO GÁSTRICO se utiliza para DIFERENCIAR entre el STDA Y STDB.

o La POSITIVIDAD EN EL ASPIRADO del tubo naso gástrico, RUIDOS INTESTINALES


HIPERACTIVOS y elevación de los niveles del BUN apoyan el diagnostico de STDA.

o En la hematemesis secundaria a la ingesta de AINES se debe realizar una


esofagogastroscopia y administrar un inhibidor de la bomba.

o El hematoma retroperitoneal produce una IMAGEN DE VIDRIO DESPULIDO en la


radiografía de abdomen.
ABSCESO ANAL
• Cuales son las causas de absceso anorrectal especificas y no especificas?
 R = ESPECIFICAS: Chron, CUCI, Tb, actinomicosis, ca, leucemias. NO
ESPECIFICAS: Infección bacteriana de glándulas anales

• Cual es la complicación principal que sucede al drenar un absceso anal?


 R = Posterior formación de FISTULAS

• Cual es el cuadro clínico en un px que presenta un absceso anal?


 R = DOLOR ANAL INTENSO o una TUMORACIÓN ANAL MUY DOLOROSA que
apareció en forma reciente.

• Cual es el manejo de un absceso perianal?


 R = DRENAJE Y METRONIDAZOL O CIPROFLOXACINO.
TUMORES DEL ANO
FISTULA ANAL
• Cual es la etiología de una fistula anal?
 R = Abscesos crónicos, CHRON Y CUCI.

• Cual es el cuadro clínico de las fistulas anales?


 R = Es característico la historia de un ABSCESO PREVIAMENTE DRENADO en la
misma localización, DOLOR PERIANAL, la principal molestia es la PERMANENTE
SALIDA DE MATERIAL PURULENTO a través de un orificio que MANCHA LA ROPA
INTERIOR.

• Cual es el estudio de imagen utilizado para el diagnostico de las fistulas anales?


 R = ULTRASONIDO RECTAL

• Cual es el manejo de las fistulas anales?


 R = La forma para predecir por donde drena la fistula es la regla de GOODSALL-
SALMON y una vez identificada se realiza FISTULOTOMIA.
FISURA ANAL
• Cual es la etiología de las fisuras anales?
 R = ISQUEMIA EN MUCOSA POR ESFÍNTER HIPERTÓNICO.

• Cual es el cuadro clínico de las fisuras anales?


 R = DOLOR intenso TIPO ARDOROSO que se relaciona al inicio
de la evacuación con una SENSACIÓN DE DESGARRO
al pasar el bolo fecal. SANGRADO FRECUENTE, suele ser en
POCA CANTIDAD.

• Cual es el manejo de las fisuras anales?


 R = Baños de asiento, laxante, RELAJANTE DE MUSCULATURA
PÉLVICA COMO CREMA DE ISOSORBIDE, DILTIAZEM O
NIFEDIPINO.
HEMORROIDES
HEMORROIDES
HERNIAS ABDOMINALES
• De donde emerge la hernia inguinal directa?
 R = Emerge DENTRO DE LOS VASOS EPIGÁSTRICOS originándose en el piso del
conducto inguinal

• De donde emerge la hernia inguinal indirecta?


 R = Por FUERA DE LOS VASOS EPIGÁSTRICOS y son DE ORIGEN CONGÉNITO siendo
mas FRECUENTES DEL LADO DERECHO.

• De que manera se distingue a la ex fis una hernia inguinal directa de indirecta?


 R = Una HERNIA INDIRECTA EJERCE CIERTA FUERZA CONTRA LA PUNTA DEL DEDO,
en tanto que la DIRECTA PRESIONA LA PULPA DEL DEDO. Además cuando es
colocado el dedo en el orificio inguinal y pedirle al paciente que tosa es posible
parar la hernia indirecta y no así la directa.

• Cual es la localización de la hernia crural?


 R = DEBAJO DEL LIGAMENTO INGUINAL, medial a los vasos femorales
manifestándose CON DOLOR EN LA REGIÓN INGUINAL, progresivo e incapacitante.
CAUSAS DE ELEVACION DEL AST O ALT
HEPATITIS VIRAL
VHA-PICORNAVIRUS
• Cual es el dato de laboratorio que orienta al VHA?
 R = IgM VS VHA AGUDO E IgG CUANDO YA HABÍA SIDO INFECTADO EL
PACIENTE

• Cual es el manejo del VHA?


 R = Sintomático, Inmunización pasiva como profilaxis postexposicion.

• Cuales son las indicaciones de la VACUNA PASIVA del VHA?


 R = PERSONAS QUE PLANEAN UN VIAJE, contactos íntimos, PERSONAL DE
GUARDERÍAS, ASILOS E INSTITUCIONES. NO esta INDICADA en px que tienen
DATOS CLÍNICOS DEL VHA

• Cuales son las indicaciones de la VACUNA ACTIVA del VHA?


 R = ENFERMEDAD HEPÁTICA CRÓNICA, VARONES HOMOSEXUALES,
DROGADICTOS.
VHB- HEPADNAVIRIDAE
• Cual es el dato de laboratorio que te orienta a VHB?
1) HBs Ag, la persistencia de >6m sugiere infección crónica.
2) Anti HBs confiere inmunidad protectora ante el virus.
3) Hbe Ag es el antígeno que refiere la REPLICACIÓN ACTIVA del virus y la
CONTAGIOSIDAD, en px con infección crónica permanece constante.
4) IgM antiHBc infección aguda.
5) - 10% EVOLUCIONA A CRONICIDAD, BUEN PRONOSTICO. TX EN CRONICO

• Cual es el manejo del VHB?


1) INTERFERON ALFA
2) LAMIVUDINA (análogo de nucleosidos que inhibe la TRANSCRIPTASA REVERSA),
3) ADEFOVIR (análogo de nucleótidos sintético que inhibe a la DNA polimerasa del VHB),
4) ENTECAVIR ( análogo nucleosido deoxiguanina que inhibe la actividad del DNA
polimerasa del VHB).
VHD
• Que caracteriza al VHD?
 R = Es EL VIRUS MAS PEQUEÑO con 36 nm, NECESITA DE LA ENVOLTURA DEL VHB para
sobrevivir y su medio de CONTAGIO ES EL MISMO QUE EL DEL VHB.

• Cual es el curso del VHD en coinfección aguda con VHB?


 R = Tiende a ser LIMITADO Y ELIMINAR EL HBsAg POR LA RESPUESTA INMUNE, al mismo
tiempo que desaparece el VHD.

• Cual es el curso del VHD en súperinfección con VHB?


 R = Exposición del VHD EN INDIVIDUO PREVIAMENTE INFECTADO POR VHB. LA
REPLICACIÓN DEL VHD ES MAYOR Y MÁS RÁPIDA al encontrar previamente expresado al
HBs Ag. En 70% de pacientes de los casos la infección se vuelve crónica y se encuentran
mayores tasas de mortalidad

• Cual es el estudio de laboratorio que orienta al diagnostico del VHD?


 R = IgG anti VHD o por detección de RNA DE VHD EN SUERO.
• Cual es el manejo del VHD?
 R = No hay tratamiento especifico, se ha encontrado mejoría con INTERFERON ALFA.
VHE- CALICIVIRUS
FAMILIAS DE HEPATITIS VIRICA
• VHA
PICORNAVIRUS
• VHB
HEPADNAVIRUS
• VHC
FLAVIVIRUS
• VHE
CALICIVIRUS
HEPATITIS AUTOINMUNE
• Que predisposición genética tienen las personas que padecen hepatitis autoinmune?
 R = HLA-B8, HLA-DR3 Y DR-52.

• Que clasificaciones tiene la hepatitis autoinmune?


 TIPO I: Autoinmune clásica o Lupoide, se presenta en mujeres jóvenes donde 30% presenta trastornos como
TIROIDITIS AUTOINMUNE O CUCI. A diferencia de otros grupos se presenta HIPERGAMAGLOBULINEMIA EN
SUERO 5-6 GR/DL.
 TIPO II: Mas común en Europa y los anticuerpos que distinguen a este grupo son los ANTIMICROMOSOMA DE
HÍGADO Y RINON TIPO 1 (ANTI LKM 1), asociada a enfermedades inmunológicas como el VITÍLIGO, DM 1,
TIROIDITIS AUTOINMUNE.
 TIPO III: Se encuentran anticuerpos anti antígeno hepático soluble/hígado- páncreas (anti-SLA/LP) encontrando
mas frecuentemente la TIROIDITIS AUTOINMUNE COMO ASOCIADA.

• Cual es el cuadro clínico de las hepatitis autoinmunes?


 R = MÚLTIPLES TELANGIECTASIAS, astenia, ARTRALGIAS, ESTRIAS CUTÁNEAS, acné, hirsutismo, AMENORREA Y
HEPATOMEGALIA.

• Cuales son los CRITERIOS diagnósticos para hepatitis autoinmune?


1) MAYORES: Elevación persistente de transaminasas, Hipergamaglobulinemia 2.5 o mayor, Ac circulantes y/o
células LE positivas, marcadores serológicos para virus de hepatitis negativos.
2) MENORES: Manifestaciones sistémicas como fiebre-artralgias-erupciones cutáneas, Complicaciones con
otras enfermedades autoinmunes , incluyendo las del colágeno, eritrosedimentacion elevada.
3) HISTOLOGIA: Hepatitis crónica o cirrosis con marcada infiltración de células y necrosis de hepatocitos,
ausencia de hallazgos específicos para el diagnóstico de otras entidades.

• Cual es el manejo de la hepatitis autoinmune?


 R = De elección son los GLUCOCORTICOIDES SOLOS O COMBINADOS CON AZATRIOPINA. La remisión se
considera cuando los px demuestran mejoría clínica, niveles de transaminasas y gammaglobulinas normales en
caso de recidivas se usa tratamiento inmunosupresor de por vida.
HEPATITIS ALCOHOLICA
• Cual es el cuadro clínico de un paciente con hepatitis alcohólica?
 R = Perdida de peso, anorexia, ICTERICIA, FIEBRE Y
HEPATOMEGALIA DOLOROSA.

• Cuales son los hallazgos de laboratorio en la hepatitis alcohólica?


1) AUMENTO DE LAS ENZIMAS HEPÁTICAS
2) ANEMIA
3) TROMBOCITOPENIA
4) LEUCOCITOSIS CON PREDOMINIO DE NEUTROFILOS Y
5) SE CONFIRMA EL DIAGNOSTICO POR BIOPSIA.

• Cual es el manejo de la hepatitis alcohólica?


 R = Abstinencia, CORTICOIDES y plan nutricio.
CIRROSIS HEPATICA
• Que parámetros definen la presión portal?
 R = Flujo venoso portal y la resistencia interior del hígado

• Cuales son las causas mas frecuentes de cirrosis hepática?


 R = Alcohol, HEPATITIS B Y C, METOTREXATO, alfametildopa e
hidralazina.

• Cual es la clasificación morfológica de la cirrosis?


1) CIRROSIS MICRONODULAR: Nódulos menores de 3 mm.
2) CIRROSIS MACRONODULAR: Nódulos de mas de 3 mm.
3) CIRROSIS MIXTA: Mircro y macronodulares que ocurre comúnmente por
alcohol.

• Cual es el marcador mas útil para el diagnostico de cirrosis biliar primaria?


 R = ANTICUERPOS ANTIMITOCONDRIALES.
• Cual es el cuadro clínico de la cirrosis?
 R = Asintomatico o los siguientes:
A. SINTOMAS CONSTITUCIONALES:
B. Astenia, anorexia, perdida de peso muscular.
C. ASCITIS:
D. Por retención de sodio y agua.
E. SIGNOS CUTANEOS:
F. Ictericia, palidez por la anemia. Telangiectacias cutáneas y eritema palmar (que predomina sobre las eminencias tenar e
hipotenar y en las llemas de los dedos) son datos bastantes caracteristicos.
G. CIRCULACION COLATERAL:
H. Los sitios mas comunes de aparición con significado patológico son el esófago y el fondo gástrico.
I. ALIENTO:
J. Olor dulzón debido a la exhalación de mercaptanos, productos derivados de la metionina por defecto de su metilación.
K. ALTERACIONES ENDOCRINAS:
L. Varones con signos de feminización como ginecomastia y cambios en la distribución del vello coroporal. El hipogonadismo en
varones produce atrofia testicular, perdida de la libido, impotencia. En ambos sexos el vello axilar es escaso. En las mujeres es
frecuente la oligomenorrea, amenorrea y esterilidad.
M. ALTERACIONES HEMATOLOGICAS:
N. Alteraciones en la coagulación dado que todos los factores se sintetizan en hígado menos el factor de von Willebrand.
O. ALTERACIONES PULMONARES:
P. Ascitis torácica o derrame pleural
Q. ALTERACIONES DIGESTIVAS:
R. Hemorragia digestiva, colelitiasis.
S. ALTERACIONES RENALES:
T. Retención de sodio con ascitis
U. SX HEPATORRENAL:
V. Se trata de una insuficiencia renal funcional como respuesta a mecanismos de descompensación tales como el uso exagerado
de diuréticos, AINES, paracentesis. La histología es normal. Este síndrome se caracteriza por una insuficiencia renal funcional
con función tubular normal en un px con insuficiencia hepática.
W. INFECCIONES:
X. IVU mas comunes.
Y. ALTERACIONES NEUROLOGICAS:
Z. Encefalopatia hepática donde la primer manifestación en aparecer son los cambios en el comportamiento. El GABA es el
principal neurotransmisor inhibitorio del cerebro, se ha postulado que el GABA producido a nivel intestinal escapa del
metabolismo hepático, atraviesa la barrera hematoencefalica y penetra al cerebro desencadenando la encefalopatía hepática.
• 184. Cual es el manejo de la cirrosis hepática?
 R = Dieta con contenido de proteínas 1 a 1.2 g/gr. Prohibicion del consumo de
alcohol. Hay tratamientos utiles específicos de cirrosis, como las flebotomías en la
hemocromatosis, la d-penicilamina en la enfermedad de Wilson, los corticoides en
las cirrosis autoinmunes, los antivíricos para cirrosis por VHB y VHC y el acido
urodesoxicolico para la cirrosis biliar primaria. La cirugía se asocia a una mortalidad
operatoria del 30% pero las indicaciones para la cirugía son rotura de varices o
sangrados. A las varices tipo 2 y 3 se administra betabloqueador en caso que no
exista contraindicación. Seguimiento ecográfico cada 6m para detectar
hepatocarcinoma.

• 185. Cual es el manejo de emergencia en la cirrosis hepática?


 R = En caso de sangrado alto una endoscopia debe realizarse para dar
escleroterapia. Uso de vasopresina por via IV teniendo cuidado de la necrosis de la
piel. Anastomosis portosistemica intrahepática por via transyugular TIPS para
controlar la hemorragia digestica por varices esofágicas irreductible por otros
métodos.
HEPATOCARCINOMA
• Cuales son los factores de riesgo para desarrollar un hepatocarcinoma?
 R = Aparece en personas que ya tienen alguna enfermedad hepática como
HEPATITIS POR VIRUS B O C, CIRROSIS.

• Cual es el cuadro clínico de un px con hepatocarcinoma?


 R = Puede cursar asintomático. Cuando existen manifestaciones lo mas habitual es
que se presenten síntomas inespecíficos como baja de peso, dolor abdominal,
saciedad precoz o masa palpable.
Cuales son los estudios de gabinete utilizados para diagnosticar hepatocarcinoma?
 R = TAC DINÁMICA HELICOIDAL MULTICORTE DE ELECCIÓN. Us de seguimiento. La
ALFAFETOPROTEINA es el marcador tumoral mas utilizado para el dx y seguimiento
(puede elevarse en embarazo y tumores testiculares). Confirmación por biopsia.

• Cual es el manejo del hepatocarcinoma?


 R = RESECCIÓN del tumor. INYECCIÓN DE ETANOL en tumor lo reseca y es fácil de
realizar. QUIMIOEMBOLIZACION que frecuentemente se complica con el llamado
síndrome postembolizacion caracterizado por fiebre, elevación enzimática, dolor
abdominal y nauseas. TRANSPLANTE HEPÁTICO.
ABSCESO HEPATICO AMEBIANO
VESICULA BILIAR
• Que fármacos pueden obliterar el esfínter de Oddi?
 R = Morfina y meperidina

• Que elementos componen el síndrome de Mirizzi?


 R = COMPRESIÓN de los conductos biliares debida a INFLAMACIÓN intensa por colecistitis
secundaria a la IMPACTACIÓN de un lito en el infundíbulo de la vesícula.

• Cual es el estándar de oro para el diagnostico de coledocolitiasis?


 R = CPRE

• Cual es el cuadro clínico de la COLANGITIS?


 R = TRIADA DE CHARCOT que se compone de FIEBRE, DOLOR EN CUADRANTE DERECHO E
ICTERICIA. Cuando se le agregan cambios en el estado mental y choque se conoce como
PENTADA DE REYNOLDS.

• Cual es el estándar de oro para diagnostico de colangitis?


 R = CPRE. La presencia de leucocitosis, elevación de las bilirrubinas, FA y transaminasas
apoyan el diagnostico.
HEMOCROMATOSIS
• Que tipo de trastorno es la hemocromatosis y que intercambio de aa se involucran?
1) AR. Se produce por sustitución de CISTINA POR TIROSINA en la posición
282 (C282Y).
2) Trastorno de almacenamiento de hierro en el que el incremento inapropiado de la
absorción intestinal. Rara vez se presenta antes de los 20ª.

• Cual es el cuadro clínico de la hemocromatosis?


 R = Los síntomas iniciales consisten en debilidad, cansancio, perdida de peso, cambio de
coloración de la piel, dolor abdominal, perdida de la libido y síntomas de DM. La
HEPATOMEGALIA 95% , HIPERPIGMENTACION 90 %(color gris pizarra o
metálico/ bronceado), POLIARTRITIS 20-50%, ICC 10%, angiomas en araña,
esplenomegalia, artropatía, ascitis, arritmias cardiacas, HIPOGONADISMO con perdida
de vello, atrofia testicular y la ictericia son signos de enfermedad avanzada.

• Como diagnosticas hemocromatosis?


 R = 1) Medición de hierro sérico y porcentaje de TRANSFERRINA > 50% EN AYUNO 2)
Medición de concentración de ferritina 3) Biopsia hepática

• Cual es el manejo de la hemocromatosis?


 R = FLEBOTOMIAS de 500 ml una o 2 veces a la semana. DEFEROXAMINA cuando la anemia
o hipoproteinemia son muy graves.
ENFERMEDAD DE WILSON
• Que causa la enfermedad de Wilson?
 R = AR. Causado por mutaciones en el gen ATP 7B, cuyo producto es una ATPasa transportadora
de cobre ligada a membrana.

• Cual es el cuadro clínico de la enfermedad de Wilson?


 R = MANIFESTACIONES NEUROLÓGICAS manifestadas principalmente con trastornos del
movimiento como DISTONIA, INCOORDINACIÓN Y TEMBLORES. La disfagia y la disartria son
comunes. Anillos de KAYSER-FLEISCHER.

• Cuales son las pruebas diagnosticas para la enfermedad de Wilson?


 R = Niveles séricos de CERULOPLASMINA BAJOS. Anillos de Kayser-Fleischer 99 %. Biopsia
hepática con análisis cuantitativos de cobre con valores >3. 1mol/g.

• Cual es el manejo de la enfermedad de Wilson?


1) PENICILAMINA + PIRIDOXINA.
2) La TRIENTINA ha sustituido a la penicilamina.
3) ZINC es menos toxico.

• Cual es el pronostico de la enfermedad de Wilson?


 R = BUENO, los síntomas neurológicos desaparecen una vez iniciado el tratamiento.
PANCREATITIS AGUDA
• Cual es el cuadro clínico de la pancreatitis aguda?
 R = Aparición súbita de un dolor en la región abdominal superior acompañado de nausea y vomito. El paciente
reporta el dolor en el epigastrio y en la región periumbilical, con irradiación hacia espalda, tórax, flancos y
abdomen inferior. Tomando posición antialgica. A la EX FIS: Fiebre, hipotensión, rigidez abdominal, taquipnea y
distención abdominal. Es posible encontrar los signos de CULLEN (hematoma periumbilical) o de GREY-
TURNER (hematoma en los flancos) relacionados con hemorragias retroperitoneales.
• Cuales son los hallazgos de laboratorio en la pancreatitis aguda?
1) Elevación de la amilasa pancreática.
2) LIPASA PANCREÁTICA QUE SE ELEVA LAS PRIMERAS 4-8 HRS.
3) Niveles de AMILASA, TAC Y US TE ORIENTAN AL DIAGNOSTICO. En la pancreatitis aguda grave disminuye el
calcio.

• Cual es el hallazgo radiológico sugestivo de pancreatitis aguda?


 R = ASA EN CENTINELA.

• Cuales marcadores de gravedad de pancreatitis aguda hay?


 R = Balthazar, Ranson y APACHE II.

• Cuales son los manejos de la pancreatitis aguda?


1) TRATAMIENTO MEDICO: La pancreatitis leve se debe tratar con HIDRATACIÓN AGRESIVA hasta que el
paciente muestre balances hídricos positivos. El control del dolor se puede realizar con MORFINA. Nutrición
enteral. En los casos graves, se recomienda ELIMINAR LA INGESTIÓN POR VIA ORAL, sin descuidar la nutrición
y estado catabólico del paciente.
2) TX QUIRURGICO: En necrosis infectada
• Los pacientes que presentan hasta dos de las caracteristicas previamente
mencionadas tienen una mortalidad minima. Aquellos con tres a cinco de
estos factores, presentaran una mortalidad 10 y 20% y los que tengan mas
de 5 presentan 50% de mortalidad.
PANCREATITIS CRONICA
• Cuales son las causas de pancreatitis crónica?
 R = CONSUMO EXCESIVO DE ALCOHOL, enfermedades
autoinmunes, hipertrigliceridemia, hiperparatiroidismo,
páncreas divisum y FIBROSIS QUÍSTICA.

• Cual es el cuadro clínico de la pancreatitis crónica?


 R = ATAQUES INTERMITENTES de dolor intenso, de
localización a nivel abdominal superior con irradiación en
forma de banda hacia la espalda.

• Cuales son los estudios de imagen utilizados para el


diagnostico de pancreatitis crónica?
 R = TAC, US y CPRE.
CANCER DE PANCREAS
• Cuales son los factores de riesgo para desarrollar cáncer de páncreas?
 R = Tabaquismo, diabetes de larga evolución, pancreatitis crónica y NEM 1.

• Cual es el cuadro clínico del cáncer de páncreas?


 R = Perdida de peso, fatiga, dolor abdominal epigástrico transfictivo,
ictericia, tromboflebitis migratoria, datos de hipertensión portal
(hepatomegalia y ascitis) y VESÍCULA PALPABLE (signo de Courvoisier-
Terrier).

• Como diagnosticas cáncer de páncreas?


 R = TAC. Elevación FA y bilirrubina en sangre. Marcador tumoral CA 19-9.

• Cual es el manejo del cáncer de páncreas?


 R = Quirúrgico. Quimioterapia.
MANEJO DEL PX POLITRAUMATIZADO
TRAUMATISMO TORAXICO
NEUMOTORAX Y HEMOTORAX
TRAUMATICO
TRAUMATISMOS ABDOMINALES
TRAUMATISMOS DEL
APARATO
GENITOURINARIO
SHOCK HIPOVOLÉMICO
¿Cuándo un paciente está
en shock hipovolémico?
Pérdida estimada de sangre en un hombre de 70 Kg
CLASE I CLASE II CLASE III CLASE IV

Pérdida de
< 750 750 - 1500 1500 - 2000 > 2000
sangre (ml)
Pérdida de
< 15 % 15 – 30 % 30 – 40 % > 40 %
sangre (%)
Pulso < 100 > 100 > 120 > 140
Presión arterial Normal. Normal. Disminuida. Disminuida

Presión pulso Normal Disminuida Disminuida Disminuida

Frecuencia
14 - 20 20 - 30 30 - 40 > 35
respiratoria
Diuresis (ml/h) > 30 20 - 30 5 - 15 Mínima

Conciencia Levemente Moderada Ansioso, Confuso,


ansioso ansioso. confuso. letárgico.
NEFROLOGIA

ENARM
ANION GAP
• El Anión GAP es una ECUACIÓN que sirve PARA CALCULAR ANIONES ORGÁNICOS que están presentes a
una CONCENTRACIÓN MUY PEQUEÑA como para ser medidas en un ionograma...
Partiendo desde el concepto de electro neutralidad, pareciera que los cationes "superan" a los aniones,
pero justamente la "brecha" entre ellos esta representados por estos Aniones...

La utilidad fisiológica/clínica del Anión Gap, es para orientarte acerca de la POSIBLE ETIOLOGÍA ( causa)
DE LA ACIDOSIS METABOLICA ( y SOLO SIRVE PARA ESTO) que el paciente esta cursando...

Las AC METAB. pueden ser DE 2 "TIPOS"... por FALTA DE BASE ( el tipo tiene una DIARREA tremenda, esta
largando hasta la 1º papilla... ), o por EXCESO DE ACIDO ( esta TOMANDO ASPIRINA como si fueran tic
tacs... o CETOACIDOSIS etc.) ( esos son algunos ejemplos... )...

Si calculas el AG... SIENDO AG= [NA+]P - ([CL-]P+[HCO3-]P)

Los valores NORMALES: AG= 12 +- 2

En caso de que el Anion Gap de AUMENTADO (+ de 14) Entonces estamos viendo una ACIDOSIS
METABOLICA POR EXCESO DE ACIDO...

Si el AG DA NORMAL, entonces, el paciente tiene un AC METAB. POR FALTA DE BASE...


Este tipo de ac metab, también es conocida por ACIDOSIS HIPERCLOREMICA
• Cuales son las causas de la hipokalemia?
POTASIO
• R = Incremento en el recambio celular, disminución de la ingesta o un aumento de su excreción

• Que medida de laboratorio es útil para el diagnostico de hipokalemia de origen renal o extrarenal?
• R = EL GRADIENTE TRANSTUBULAR DE POTASIO (TTKG), el cual si es < 2 TRADUCE HIPOKALEMIA NO RENAL y UNO >
10 ES DEBIDO A PERDIDAS RENALES. EL K URINARIO <25 MEQ/DIA es NO RENAL y K URINARIO > 30
MEQ/DIA traduce perdida de ORIGEN RENAL.

• Cuales son lo síntomas de hipokalemia?


• R = DEBILIDAD MUSCULAR, fatiga, CALAMBRES. <2.5 Meq parálisis flácida, HIPORREFLEXIA, hipercapnia y rabdomiolisis.

• Que elementos participan en la captación de potasio?


• R = La insulina en presencia de glucosa, estimulación B adrenérgica

• Cual es el regulador de potasio mas importante?


• R = ALDOSTERONA facilita la excreción urinaria de K en los TCD.

• Cuales son las causas de la hiperkalemia?


• R = > 5.5 mEq. DISMINUCION DE LA EXCRESION: HIPOALDOSTERONISMO, IRA o IRC, ESPIRONOLACTONA,
IECA, B bloqueador. Desplazamiento de K al interior: Rabdomiolisis, hemolisis y ACIDOSIS METABOLICA

• Cual es el cuadro clínico de hiperkalemia y EKG?


• R = DEBILIDAD MUSCULAR, distención abdominal, DIARREA. EKG con onda T picuda, QT corto y depresión de onda T.

• Cual es el manejo de la hiperkalemia?


1) INSULINA + GLUCOSA AL 10%, SALBUTAMOL inhalado, GLUCONATO DE CALCIO IV, HCO3 que produce ingreso a la
celula de K.
2) EL TRATAMIENTO AGUDO CON GLUCONATO DE CALCIO DEBE RESERVARSE cuando se presentan manifestaciones
cardiacas de toxicidad celular por exceso de potasio (calcitosis).
NTA
• A que padecimiento se le conoce como NTA?
• R = IRA por lesiones tubulares siendo las principales causas la isquémica y por
toxinas

• Que nefrotoxinas endógenas ocasionan NTA?


• R = Productos contenedores de HEM, ACIDO ÚRICO, MIOGLOBINURIA por
RABDOMIOLISIS

• Cuales son los datos de lab de NTA?


• R = ORINA COLOR CAFÉ LODOSO por Hb y cilindros epiteliales de túbulos renales

• Cual es el manejo de la NTA?


• R = Se debe prevenir hipercalcemia, FUROSEMIDE, RESTRICCIÓN DE PROTEÍNAS
DIETÉTICAS Y DIÁLISIS EN PACIENTE CRITICO

• Cual es la complicación mas común y mas temida de la NTA?


• Falla cardiaca
NEFRITIS INSTERSTICIAL AGUDA
• Que es la nefritis intersticial aguda?
• R = Respuesta inflamatoria intersticial con edema y posible daño celular

• Cuales son las causas de nefritis intersticial?


• R = FÁRMACOS 70%. Infecciosas e inmunitarias

• Cuales son los datos clínicos de nefritis intersticial aguda?

• DETERIORO ABRUPTO DE LA FUNCIÓN RENAL


R = Se caracteriza por un
acompañado de poliuria, Nicturia, ACIDOSIS METABOLICA y glucosuria. FIEBRE 90%,
ARTRALGIAS 80%, EXANTEMA 20%. Bh: EOSINOFILIA EN FASE AGUDA. EGO con
EOSINOFILURIA, hematuria, leucocituria y cilindros leucocitarios.
• Cuales son los hallazgos en la biopsia con nefritis intersticial aguda?
• R = Infiltración de células inflamatorias en intersticio renal con edema, predominan mononucleares y LT,
GRANULOMAS NO CASEIFICANTES.

• Cual es el manejo de la nefritis intersticial aguda?


• R = RETIRAR EL FACTOR predisponente. Puede llegar a REQUERIR DIÁLISIS, MEDIDAS DE SOSTEN,
METILPREDNISOLONA en NI POR FÁRMACOS
• Que caracteriza a la necrosis intersticial crónica?
1. Ausencia de proteinuria e hipoalbuminemia
2. Piuria estéril y leucocitosis mas que hematuria
3. Poliuria y Nicturia
4. Otros defectos tubulares como ATR y osteomalacia.
IRC
• Cual es la clasificación de la IRC?
• ESTADIO FG
• I 90
• II 60 – 89
• III 30 – 59
• IV < 15 O DIALISIS

• Cual es el manejo restrictivo de IRC?


• R = Restricción de proteínas, restricción de Na y H2 O, restricción de Mg
entre otros.

• Que pasa con la capacidad de dilución y de concentración en la IRC?


• La capacidad de DILUCIÓN se deteriora antes que la de CONCENTRACIÓN.
CRIOGLOBULINEMIA
• Que caracteriza a la crioglobulinemia y con que enfermedades se relaciona?
• R = Es una vasculitis de pequeños vasos DEBIDO A INMUNOGLOBULINAS que se
PRECIPITAN CON EL FRIO relacionándose mucho con el VHC +++ mas que con el
VHB, endocarditis o LES.

• Cual es el cuadro clínico de la crioglobulinemia?


• R = PURPURA 90 %, ARTRITIS 80% , Neutropenia 70 %, DANO RENAL 50% (de este
el 80% se manifiesta como GLOMERULONEFRITIS
MEMBRANOPROLIFERATIVA Y TROMBOS EN EL INTERIOR DE LOS
CAPILARES GLOMERULARES.

• Cual es el manejo de la crioglobulinemia?


1) Proteinuria y daño renal leve dándose TRATAMIENTO SINTOMÁTICO Y ESPECIFICO
2) PARA VHC SI ES QUE ESTA ASOCIADO con origen se da INTERFERON Y RIVABIRINA,
3) Si la PROTEINURIA se encuentra en RANGO NEFRÓTICO se da
METILPREDNISOLONA, PLASMAFERESIS O RITUXIMAB.
NEFROPATÍA POR IgA O SX DE
BERGUER
• Que caracteriza a la nefropatía por IgA o sx de Berguer?

• R = Se deposita IgA en el mesangio glomerular, observándose la MISMA LESIÓN


EN PURPURA DE HENOCH- SCHONLEIN
• Cuales son los signos de nefropatía por IgA?
• R = HEMATURIA “SINFARINGITICA” con orina color COCA-COLA 100%. IVRS 50%, GI 10%

• Que datos se encuentran en la biopsia en la nefropatía por IgA ?


• R = DEPOSITOS DE IgA acompañados de C3 E IgG.

• Cual es el manejo de la nefropatía por IgA?


1) IECAS, ESTEROIDES.
2) En caso de proteinuria se combina CICLOFOSFAMIDA CON AZATRIOPINA.

• Cual es el pronóstico de nefropatía por IgA?


• R = BUENO, pero es MALO para los que desarrollan SEMILUNAS EN LA BIOPSIA
GN PAUCINMUNITARIA
• En que enfermedad se presenta GN paucinmunitaria?
1) GRANULOMATOSIS DE WEGENER,
2) ENFERMEDAD DE CHURG- STRAUSS,
3) POLIANGEITIS MICROCITICA.
• Que patogénesis se encuentra hubicada en 80% de las glomerulonefritis
paucinmunitarias?
• R = Asociada a ANCA

• Cuales son los datos de laboratorio de la GN paucinmunitaria?


• R = Patron citoplasmático c-ANCA, patrón perinuclear p-ANCA.

• Cual es el manejo de la GN paucinmunitaria?


• R = ESTEROIDES, CICLOFOSFAMIDA.
AFECCION RENAL X ENF SISTEMICA
NEFROPATIA LUPICA
NEFROPATÍA MEMBRANOSA
• Que es la nefropatía membranosa?
• R = Enfermedad glomerular por DEPOSITO DE IgG
y complemento constituyendo la forma mas COMÚN DE
SÍNDROME NEFRÓTICO EN ADULTOS, es idiopático 70% y el resto se debe a virus, neoplasias, drogas o
enfermedades autoinmunes

• Cual es el cuadro clínico de la nefropatía membranosa?


• R = SÍNDROME NEFRÓTICO.

• Que etiología tiene la NM secundaria?


• R = LES, VHC Y ENDOCARDITIS son las mas comunes. Hepatitis B, sífilis, cáncer, penicilamina y captopril.

• Cuales son los signos y síntomas de NM secundaria?


• R = Relacionados a SX NEFRÓTICO

• Que revela la biopsia e inmunohistoquimica en la GMN?


• R = HISTOLOGIA: PROLIFERACIÓN EN FORMA DE PARCHES, infiltrado leucocitario intraglomerular y necrosis
intracapilar. INMUNOHISTOQUIMICA: IgG siempre presente, IgM e IgA.

• Cual es el manejo de la nefropatía membranosa?


• R = PROTEINURIA < 4 GR DAS IECAS CON META DE TA 125/75. PROTEINURIA > 4 GR y < 8 GR SIN
DETERIORO DE LA FUNCIÓN RENAL EL TRATAMIENTO SE ENCAMINA A DISMINUIR LA PROTEINURIA CON IECAS Y
OBSERVAR POR 6M.
GLOMERULONEFRITIS FOCAL Y
SEGMENTARIA
• Cual es la etiología de la Glomerulonefritis focal y segmentaria?
• R = Primaria (idiopática) o secundaria debido a otras patologías como agenesia renal, ENFERMEDAD DE
CÉLULAS FALCIFORMES o por VIH.

• Dentro de las diferentes variedades de Glomerulonefritis focal y segmentaria cuales producen mas
proteinuria?
• R = LA VARIEDAD CELULAR Y COLAPSANTE en el 80% de los casos de síndrome nefrótico

• Cual es el cuadro clínico de la Glomerulonefritis focal y segmentaria?


• R = PROTEINURIA asintomática o síndrome nefrótico completo

• Cuales son los datos de laboratorio y biopsia de la Glomerulonefritis focal y segmentaria?


• R = La PROTEINURIA puede oscilar DESDE <1 GR HASTA 20-30 GR y en la biopsia los hallazgos de
HISTOLOGÍA COMO SON LA VARIEDAD CELULAR Y COLAPSANTE.

• Cual es el manejo de la Glomerulonefritis focal y segmentaria?


1) LOS QUE TENGAN PROTEINURIA SUBNEFROTICA 2-3 GR/ no se les dará tratamiento inmunosupresor, siendo
el principal tratamiento el control de la TA CON META DE 130/80, UTILIZANDO IECAS, hipolipemiantes,
DIURÉTICOS EN CASO DE EDEMA.
2) En los pacientes que se presentan con SÍNDROME NEFRÓTICO SE USA PREDNISONA, CONTROL DE LA TA
CON META DE 125/75, ANTICOAGULANTES (>10 GR/DIA) y en caso de RESISTENCIA SE USARA
CICLOSPORINA.

• Cual es el pronostico de la Glomerulonefritis focal y segmentaria?


• R = MALO SI LA PROTEINURIA ES > 10 GR/DIA
LIDDLE, GITELMAN Y BARTTER
• Que es la enfermedad de Liddle?
• R = AD. El síndrome de Liddle (o SEUDOHIPERALDOSTERONISMO) es un trastorno AD caracterizado por
HIPERTENSIÓN SENSIBLE A SAL con expansión de volumen, HIPOKALEMIA y ALCALOSIS
METABOLICA, con ACTIVIDAD DE RENINA Y ALDOSTERONA EXTREMADAMENTE BAJAS y FG
NORMAL.

• Que es el síndrome de Gitelman?


• R = Se caracteriza por HIPOKALEMIA e HIPOMAGNESEMIA, además de contar con HIPOCALCIURIA,
perdida de sal y alcalosis metabólica con HIPERALDOSTERONISMO, HIPERRENINEMIA, asi como
HIPERTROFIA E HIPERPLASIA DEL APARATO YUXTAGLOMERULAR.

• Cual es el cuadro clínico del síndrome de Gitelman?


• R = Debilidad muscular y DERMATITIS inespecífica. ESPASMOS DISTALES ocurren EN PERIODOS DE
FIEBRE, vomito o diarrea. Algunos sufren condrocalcinosis.

• Que es el síndrome de Bartter?


• R = AR. Se caracteriza por HIPOKALEMIA GRAVE, perdida de sal y alcalosis metabólica con
HIPERALDOSTERONISMO, HIPERRENINEMIA, así como HIPERTROFIA E HIPERPLASIA DEL APARATO
YUXTAGLOMERULAR. Casi TODOS TIENEN NEFROCALCINOSIS por la
hipercalciuria.

• Cual es el cuadro clínico del síndrome de Bartter?


• R = Tipo I y II: Las manifestaciones tempranas incluyen POLIHIDRAMNIOS, retardo en el crecimiento,
polidipsia, deshidratación, avidez por sal y debilidad muscular. En el tipo III: Se manifiesta en la
INFANCIA similar al Sx de Gitelman. El tipo IV: TODO LO ANTERIOR MAS SORDERA SENSORINEURAL E
IRC TEMPRANA.
SÍNDROME DE FANCONI
• Que es el síndrome de Fanconi?
• R = Se debe a la DISFUNCIÓN GLOBAL DEL TÚBULO PROXIMAL.

• Cuantos tipos de Sx de Fanconi hay y que los caracteriza?


1) HEREDITARIO: Cistinosis, galactosemia, tirosinemia.
2) ADQUIRIDO: INTOXICACIÓN POR PLOMO O CADMIO, TETRACICLINAS
CADUCADAS, tolueno (inhalar pegamento), agentes quimioterapéuticos
como el cisplatino.
3) FANCONI LIKE: Pacientes adultos con disproteinemias como amiloidosis,
en enfermedad de cadenas ligeras y mieloma múltiple.

• Cual es la anormalidad metabólica mas frecuente en el síndrome de


Fanconi?
• R = ACIDOSIS METABOLICA HIPERCLOREMICA.
ATR
• Cuales son las tubulopatias asociadas a alcalosis metabólica?
• R = Síndrome de CHEEK-PERRY(GITLEMAN), LIDDLE Y BARTTER

• Que es la acidosis tubular renal o ATR?


• R = Grupo de entidades patológicas caracterizadas por defectos de transporte, ya sea
REABSORCIÓN DE BICARBONATO (HCO3), EXCRECIÓN DE HIDROGENIONES O AMBAS.

• Donde se lleva a cabo la reabsorción de HCO3 y la excreción de H+?


• R = HCO3 EN EL TCP Y H+ EN TCD.

• Que caracteriza a la ATR tipo I DISTAL O CLÁSICA?


• R = ACIDOSIS METABÓLICA CON BRECHA ANIONICA NORMAL O HIPERCLOREMICA, HIPOKALEMIA
E IMPOSIBILIDAD DE ACIDIFICAR LA ORINA A < 5.5 bajo el estimulo de acidosis metabólica
intensa.

• Cuales son las bases fisiopatológicas de la ATR I?


• R = 1) Defecto secretor (DEFICIENCIA EN LA SECRECIÓN DE HIDROGENIONES), formas dominantes,
recesivas y asociadas a síndrome de Sjogren, LES, AR. 2) defecto de gradiente, en el cual hay un
reflujo de los hidrogeniones normalmente secretados a nivel distal (anfotericina B) y 3)
Incapacidad de generar o mantener una diferencia luminal negativa transepitelial distal como en la
uropatia obstructiva, enfermedad de células falciformes, hiperplasia adrenal perdedora de sal.

• Como se diferencia la ATR tipo I de la tipo II?


• R = Que estos pacientes a menudo PRESENTARAN NEFROLITIASIS Y NEFROCALCINOSIS.
• Como se maneja la ATR tipo I?
• R = ADMINISTRACIÓN DE ÁLCALI, SE PREFIERE CITRATO DE POTASIO
• Que caracteriza a la ATR TIPO II O PROXIMAL?
• R = Puede ocurrir sola o acompañada de otros defectos tubulares como
síndrome de Fanconi y se caracteriza por una DISMINUCIÓN EN EL UMBRAL
RENAL PARA EL HCO3, el cual usualmente es de 22 mml/L en infantes y 26
mmol/L en adultos.

• Que caracteriza a la ATR tipo IV hiperkalemica?


• R = Usualmente ocurre con la presencia de IR MODERADA, sin embargo la
magnitud de la HIPERKALEMIA y de la acidosis es desproporcionadamente
extensa para el nivel de IR.

• Cual es la fisiopatología de la ATR tipo IV o hiperkalemica?


• R = Defecto en la AMONIOGENESIS

• Cual es la principal enfermedad relacionada con la ATR tipo IV o


hiperkalemia?
• R = DM, además del uso de AINES, IECAS y ciclosporina.
SINDROME UREMICO HEMOLITICO

•En el SHE es común observar la prolongación del tiempo de?


• R = Sangría
INFECTOLOGIA

ENARM
MENINGITIS
• Cual es la presión de apertura a la toma de LCR en meningitis bacteriana?
• R = Normal <180 o elevada > 180, siendo común 200-500

• Cuales son los datos del LCR de la meningitis bacteriana aguda?


I. Presión de abertura: Normal o > 180 siendo común observar valores entre 2 –
500
II. Apariencia: Leucocitos y bacterias turban el LCR
III. Glucorraquia: Concentración < 40 (NORMAL > 45)
IV. Proteinorraquia: Valores arriba de 50 (NORMAL 40)

• Cual es el tratamiento de elección en los contactos de pacientes con enfermedad


meningocococica?
• R = RIFAMPICINA

• Que constituye el síndrome de austrian?


• R = MENINGITIS, ENDOCARDITIS y NEUMONÍA por estreptococo pneumoniae
en PACIENTE ALCOHÓLICOS
MENINGITIS
• Tratamiento de elección para meningitis por NEUMOCOCO?
• R = CEFTRIAXONA

• Tratamiento de elección para meningitis por S. AUREUS METICILINO SENSIBLE?


• R = Nafcilina, DICLOXACILINA

• Tratamiento de elección para meningitis por S. AUREUS METICILINO RESISTENTE?


• R = VANCOMICINA o TEICOPLANINA

• Tratamiento de elección para meningitis por LISTERIA?


• R = AMPICILINA

• Tratamiento de elección para meningitis por H. INFLUENZA?


• R = CEFTRIAXONA o cefotaxima

• Tratamiento de elección para meningitis por ANAEROBIOS?


• R = METRONIDAZOL

• Cual es el tratamiento de elección en la meningitis CRIPTOCOCOCICA?


• R = ANFOTERICINA B + 5-fluocitosina
MENINGITIS
• Característica de meningoencefalitis subaguda?
• Meningoencefelatis por tuberculosis

• Agente etiológicos en meningitis en recién nacidos?


• E.COLI (en el trabajo de parto) y estreptococos del grupo B (AGALACTIAE)

• Agente etiológico de meningitis en adultos?


• 1.Streptococo pneumoniae 2.Neisseria Meningitidis 3.Streptococo del grupo B

• CASO CLÍNICO: Recién nacido, con fontanela abombadas, petequias en la cara anterior
del tórax, esta en cunero y alado entro por deshidratación. Actualmente con fiebre,
rigidez de nuca, Kerney y Brusinski, punción lumbar con diplococos GRAM NEGATIVOS
en LCR

• Agente etiológico?:
• Meningitis por Neisseria Meningitidis (MENINGOCOCO)
• Tx:
• PGSC a dosis altas, al personal involucrado: cipro o rifampicina

• Medida implantada por la OMS para detección rápida de MENINGITIDIS?


• DETERMINACIÓN DE ANTÍGENOS CAPSULARES: Streptococo pneumone, Neisseria
Meningitidis, Hemophilus Influenza se detecta en 3 horas –aunque no los solicites-
MENINGITIS
• Meningitis POR LISTERIA (BACILO GRAM +) esta relacionado con:
• LECHE O DERIVADOS LACTEOS, CARNES MAL COCIDAS, INMUNODEPRIMIDOS y px
de la tercera edad…. Tx :AMPICILINA como primera opción

• Con respecto del Streptococo Pneumoniae caract de infección?:


• Se da principalmente en pacientes de los 2 a los 20 años, px ALCOHÓLICOS, con
OTITIS PREVIA.

• Meningitis por Bacilos GRAM NEGATIVOS caract de infección?


• Se presenta en diabéticos, cirróticos, alcohólicos px con IVU, px con cáncer y/o
inmunodeprimidos. El bacilo GRAM POSITIVO y entérico mas importante la
listeria monocitogena, otro bacilo que no es entérico es la hemophilus Influenza
(disminución en su incidencia actualmente)

• Agente etiológico de la Meningitis en un px que previamente se le realizo


procedimiento neuroquirurgico?
• STAPHILOCOCOS AUREUS

• Citocinas proinflamatorias que producen daño a nivel SNC, en la barrera


hematoencefalica, plexos coroides?
• IL 1 Y FNT que aumentan en las primeras 2 horas
MENINGITIS
• Laboratorio de LCR?
• PUNCIÓN LUMBAR: 1.-PRESION NORMAL DEL LCR 180mm de Agua, 2.-POLIMORFONUCLEARES:
menos de 5 polimorfos 3.-GLUCOSA: es el 50% dela glucosa central del px entre 45 y 55 4.-
PROTEINAS: 40mg/dl 5.-AGUA DE ROCA 6.-CLORUROS: 90 a 110 en LCR

• En quienes se realiza el cultivo en TINTA CHINA?


• CRIPTOCOCO NEOFORMANS EN PX CON VIH

• Caso clinico
• Px con rigidez de nuca, brusinski +, irritado Cefalea, fiebre, sin traumatismo craneoencefálico,
ANTECEDENTE DE NADAR EN UN MANANTIAL LANZÁNDOSE CLAVADOS, a la punción lumbar: aspecto
turbio, presión:200mmhg glucosa:20mg/dl proteínas:85 mg/dl Tinción de GRAM fue NEGATIVA, el
cultivo se realizo, que tipo de meningitis tiene este px:
• a.-M aseptica b.-M purulenta c.-M Tuberculosa (Agente etiológico: NAEGLERIA o gardenelas son
amibas de vida silvestre) muy agresivas, con pronostico mortal,
• Tx?:
• ANFOTERICINA B

• La meningitis Viral caract del LCR:


• CELULARIDAD: NORMAL PROTEÍNAS: ligeramente AUMENTADAS ASPECTO: AGUA DE ROCA
GLUCOSA: NORMAL PRESIÓN: nl o ligeramente NORMAL CLORUROS: NORMALES
MENINGITIS
• CASO CLÍNICO: Px con fiebre intensa (40), ingresa al servicio de urg, con trastorno
del edo. De conciencia, con 2 crisis convulsivas.. de ocupación CUIDADOR DE
CABALLOS (antecedente de haber enterrado a su caballo y después fue atacado
por aves de rapiña)
• LCR: Glucosa: 100 liquido transparente, presión: 185mm de agua, celularidad:
linfocitos cloruros normales
• Cual es la impresión dx:
• MENINGOENCEFALITIS VIRAL EQUINA VENEZOLANA

• CASO CLÍNICO: Niña Indígena procedente de AREA RURAL, se presenta al servicio


de urg. Con 3 semanas de evolución con fiebre persistente, vomito, trastorno del
estado de conciencia, ala exploración física presenta papiledema ligero, con
afectación del 3er, 4to, 6to par craneal, con rigidez de nuca. LCR: ASPECTO
XANTOCROMICO, Presión: 220mm de agua Glucosa: 30 Proteínas: de 75mg/dl ,
Celularidad por linfocitos CLORUROS: 70 Meningitis tuberculosa

• Tx de meningitis en Recién Nacidos?


• AMPICILINA (e.coli)- CEFOTAXIMA (streptococo)

MENINGITIS
Tx de meningitis en niños de 1 a 3 meses?
• AMPI + CEFO o CEFTRIAXONA + (valorar administración de DEXAMETASONA) para evitar
VENTRICULITIS

• Tx de meningits en px de 3 meses y menos de 50 años?


• CEFOTAXIMA + VANCOMICINA o CEFTRIAXONA + VANCOMICINA (cubrir cocos gram +, Y
bacilos gram -)

• Tx de meningitis en un px mas de 50 años con antecedente de alcoholismo o tiene DM


• AMPICILINA + VANCOMICINA + CEFALOSPORINA (tx p cocos gram + bacilos gram -)

• Px neutropenico, o px con meningitis con TRAUMATISMO CRANEOENCEFALICO, o que le


hicieron un procedimiento neuroquirurgico que tx se le dara?
• Cubrir Gram - (ceftazidil), cubrir Gram + meticilino resistente (Vancomicina)

• px que en el cultivo presenta meningococo que tx se da?


• PENICILINA G si es resistente: CEFTRIAXONA O CEFOTAXIMA y si es alérgico
VANCOMICINA (2 gr x dia)

• Px que le hicieron punción lumbar y obtuvieron en el LCR un cultivo Gram – que tx:
• Pseudomona aeroginosa: Ceftazidine(6gr x dia) Kliebsella, e coli: CEFOTAXIMA O
CEFTRIAZONA (4gr x dia)

• Tx para: Listeria: AMPICILINA(12 gr al dia) H. Influenza: CEFTRIAXONA o cefotaxima


Anaerobios: METRONIDAZOL (2 grx dia)
FOD
• Cual es la definición de FOD?
• R = Elevación de la temperatura por arriba de 38 GRADOS en varias determinaciones durante
mas de 3 SEMANAS sin llegar al diagnostico DESPUÉS DE 1 SEMANA de estudio hospitalario

• Cual es la definición de FOD en paciente hospitalizados?


• R = Paciente hospitalizado 24 HRS CON FIEBRE y 2 CULTIVOS NEGATIVOS después DE 2 DÍAS.

• Define FOD en paciente inmunodeficiente?


• R = FIEBRE >38.3 en paciente con NEUTROPENIA < 500 durante > 3 DÍAS y cultivos negativos
después DE 2 DÍAS.

• Define FOD asociado a VIH?


• R = Fiebre por encima de 38.3 GRADOS y que permanece sin diagnostico por MAS DE 3
SEMANAS

• Cuales son las principales causas de FOD?


1. Infección por TB y CMV
2. Neoplasia por linfoma o leucemia
3. Idiopática si después de 6m de estudio no hay causa
SEPSIS
• Como se diagnostica el síndrome de respuesta inflamatoria sistémica?
• R = Cuando se presentan 2 o mas de los siguientes parámetros:
1. FIEBRE corporal > 38 o hipotermia < 36
2. TAQUIPNEA con FR > 24 x minuto
3. TAQUICARDIA > 90 x minuto
4. LEUCOCITOSIS > 12,000 o leucopenia < 4,000 o 10% de bandas

• Como diagnosticas sepsis grave?


• R = Sepsis asociada a disfunción de un órgano con los siguientes datos:
1. Cardiovascular: TAS < 90 mm Hg que responde a fluidoterapia
2. Respiratoria: Relación FiO2/PaO2 < 200
3. Renal: Diuresis < 0.5 ML/KG/HR durante 1 hr a pesar de fluidoterapia
4. Trombocitopenia: < 80,000 o su reducción a la mitad con respecto a la obtenida
72 hrs antes
5. Acidosis metabólica: Con PH < 7.30
6. Choque séptico: Hipotensión con TAS < 90 durante 1 hr y que no responde a
fluidoterapia necesitando vasopresores
SEPSIS
• En caso de bacteriemia o sepsis cuales son algunas características
distintivas de los agentes patológicos?
A. Neisseria meningitidis: Se acompaña de PURPURA O PETEQUIAS
B. Rikettsia: Cuando aparecen LESIONES PETEQUIALES por mordida
de garrapata en zona endémica
C. Pseudomona aureoginosa: ECTIMA GANGRENOSO
D. S. Aureus o S. Pyogenes: ERITRODERMIA GENERALIZADA.

• Cual es la complicación mas frecuente de la bacteriemia?


• R = SIRPA caracterizado por HIPOXEMIA E INFILTRADOS
PULMONARES DIFUSOS.
NEUROCISTICERCOSIS
• Cual es el agente infeccioso en neurocisticercosis?
• R = Tenia SOLIUM

• Cual es el cc de la neurocisticercosis?
1) Forma activa: Sin evidencia del parasito con hallazgos en la TAC de
calcificaciones o hidrocefalia
2) Parenquimatosa: Larvas en corteza y ganglios basales
3) Forma subaracnoidea: Con fibrosis leptomeningea provocando
neuropatías por atrapamiento o hidrocefalia

• Cual es el método diagnostico de elección para le


neurocisticercosis?
• R = TAC de elección e IRM en casos dudosos.

• Cual es el manejo de la neurocisticercosis?


• R = ALBENDAZOL 400 mg c/12 1 SEMANA, PRAZICUANTEL X 15 DÍAS.
SE RECOMIENDA EL USO DE PREDNISONA 1 DÍA ANTES DEL FÁRMACO
Y CONTINUAR CON DISMINUCIÓN DE LA DOSIS 14 DÍAS DESPUÉS.
TETANOS
• Cual es el cuadro clínico del tétanos?
• R = Comienza con ESPASMOS LEVES EN LA MANDÍBULA (trismo), el
cuello y la cara. La RIGIDEZ se desarrolla rápidamente en el TÓRAX,
ESPALDA, MÚSCULOS ABDOMINALES y en ocasiones la laringe
interfiriendo con la respiración. Los espasmos musculares son
contracciones súbitas, fuertes y dolorosas.

• Cual es el tratamiento de elección para tétanos?


• R = PENICILINA SÓDICA CRISTALINICA 20, 000 000. Se debe
administrar globulina inmunitaria antitetánica 5000 U/IM y una vez
recuperado el paciente se da esquema completo.
CARBUNO O ANTRAX
• Cual es el agente etiológico del Carbunco o ANTRAX?
• R = BACILLUS ANTHRACIS, bacilo GRAM +

• La forma cutánea de carbunco o ántrax que es la mas común como se manifiesta?


• R = Desarrollo de PÁPULA EN ZONA DE INOCULACIÓN, RODEADA DE VESÍCULAS, LA ZONA CENTRAL SE ULCERA
Y SECA DESARROLLÁNDOSE ZONA DEPRIMIDA DE COLOR NEGRO, NO DOLOROSO. Se resuelve
espontáneamente pero 20% puede ser fatal.

• Como se manifiesta la forma intestinal de carbunco o ántrax?


• R = FIEBRE, DISNEA, CIANOSIS, desorientación y signos de septicemia. Evoluciona rápidamente a choque, coma
y muerte.

• Como se manifiesta el carbunco o ántrax respiratorio y quienes lo PADECEN FRECEUNTEMENTE?


• R = Se manifiesta en personas que MANIPULAN PIEL Y LANAS. Se produce FIEBRE CON TOS NO PRODUCTIVA y
mal estar general, con buena evolución a 2-3 días y a continuación SÚBITAMENTE PRESENTA DIFICULTAD
RESPIRATORIA GRAVE, CIANOSIS Y SEPTICEMIA FATAL CON MUERTE EN <24 HRS.

• Como realizas el diagnostico de carbunco o ántrax?


• R = Identificación del bacilo con TINCIÓN GRAM, PCR.

• Cual es el manejo de carbunco o ántrax?


• R = CIPROFLOXACINO, levofloxacino o PENICILINA G.

• Como previenes el carbunco o ántrax?


• R = PROTECCIÓN DE PIEL, MUCOSAS Y LA VACUNACIÓN
SALMONELOSIS
• Cuantos antígenos tiene la salmonella typhi?
• R = 3: H FLAGELAR, K CAPSULAR y O SOMÁTICO

• Cuales son los sitios frecuentes de infección secundaria por salmonella typhi?
• R = Hígado, bazo, la medula ósea, PLACAS DE PEYER DEL ÍLEON TERMINAL y la vesícula biliar.

• Cual es el cuadro clínico característico de salmonella typhi?


• R = ROSÉOLA TIFOIDICA que se caracteriza por MACULAS ERITEMATOSAS DE 2-4 MM QUE BLANQUEAN A
LA PRESIÓN, localizadas en la parte superior del abdomen y tórax anterior y que por lo general dura de 2-3
días.

• Cual es la complicación mas frecuente de la salmonella typhi?


• R = La mas común es la HEMORRAGIA GI que resulta de la erosión de un vaso de la pared intestinal secundaria
a la necrosis de las placas de peyer en el íleon terminal

• Cual es el medio diagnostico mas sensible en la primer semana?


• R = HEMOCULTIVO, después el mielocultivo.

• Semana en la que se presentan las complicaciones de fiebre tifoidea?


• R = Fines de SEGUNDA Y TERCERA SEMANA

• Con que inoculo de S. Tiphy se desarrolla la enfermedad?


• R = 100,000
SALMONELOSIS
• A que semana aparece la roséola tifoidica, la cual aparece a la digito presión?
• R = 2da semana

• Como dx a un portador crónico de salmonella?


• R = COPROCULTIVOS + DURANTE 1ª

• En que semana los px se perforan o tienen hemorragias con choque hipovolemico?


• R = 3ERA semana

• En que semanas en la fiebre tifoidea se realizan los cultivos específicos?


• R = 1era HEMOCULTIVO, 2da MIELOCULTIVO y 3era COPROCULTIVO/urocultivo

• Cual es el cuadro clínico de la salmonelosis?


• R = Fiebre elevada 2-3 semanas, dolor abdominal, DIARREA EN SOPA DE
CHICHARO, ROSÉOLA, DELIRIO, ESPLENOMEGALIA.
• Cual es la prueba de laboratorio confirmatoria de fiebre tifoidea?

• R = 1:640 , LEUCOPENIA y cuadro clínico característico


SALMONELOSIS
• Cual es el sitio donde suele albergarse la salmonella?
• R = Vesícula biliar

• Que se presenta en la primera semana de la salmonelosis?


• R = FIEBRE, HIPERSENSIBILIDAD ABDOMINAL, TIFLITIS (CUANDO HAY DOLOR EN FOSA ILIACA
DERECHA POR INVASIÓN DE LA PLACA DE PEYER Y CREPITA AL TACTO)

• Que se presenta en la segunda semana en la fiebre tifoidea?


• R = DIARREA, estreñimiento, delirio, EXANTEMA EN CARA ANTERIOR DE TÓRAX QUE SEDE A
LA DIGITO PRESIÓN. El sistema retículo-endotelial se hipertrofia con
HEPATO/ESPLENOMEGALIA, hiperplasia de las PLACAS DE PEYER. Bazo e hígado desarrollan
nódulos tifoideos.

• Cuales son las complicaciones habituales de la fiebre tifoidea en la 3era semana?


• R = HEMORRAGIA Y PERFORACIÓN INTESTINAL, peritonitis con placa simple de abd con aire
libre. Orquitis, meningitis, nefritis.

• Cual es el tratamiento de la salmonella multiresistente o en lugares o zonas endémicas?


• R = CIPROFLOXACINO, levofloxacino, ceftriaxona
SALMONELOSIS
• Cual es el mejor método diagnostico en un paciente que se
automedico en fiebre tifoidea?
• R = MIELOCULTIVO por que el medicamento no penetra a esa zona

• Tratamiento para fiebre tifoidea?


• R = CLORAMFENICOL 50/mg/kg por 2 SEMANAS

• Tratamiento para anemia + fiebre tifoidea?


• R = AMOXICILINA o ampicilina, NO DARLE CLORAMFENICOL por ke
causa aplasia medular.

• Tx fiebre tifoidea en embarazada?


• R = AMOXICILINA o ampicilina
SALMONELOSIS
• Tx fiebre tifoidea en niño con resistencia?
• R = QUINOLONAS NO, cefotaxima o CEFTRIAXONA

• En que casos se utiliza la dexametasona en fiebre tifoidea?


• R = En el ESTADO TIFOIDICO CON SHOQUE

• Que se le da al portador de salmonella?


• R = CIPROFLOXACINA por 3 MESES

• Que tx das a un px portador crónico que no respondió a


ciprofloxacino?
• R = COLECISTECTOMÍA, por que ahí vive la salmonella.
BRUCELOSIS
• Cuales son los principales vectores de la brucelosis y a quienes afecta
comúnmente?
• R= ORDEÑADORES, rastros, CARNICERO, veterinarios son vectores.
DERIVADOS DE LA LECHE como el queso o tejidos del animal afectado.

• Cuales son las cepas de brucella que produce enf en humano/ FIEBRE
ONDULANTE?
• R = MELITERSIS DE CABRA + frecuente a nivel mundial, SUIZ DEL CERDO +,
ABORTUS DE GANADO VOVINO +++.

• Microbiologicamente que es la brucella?


• R = COCOBACILO GRAM -, crece a 37 grados, es inmóvil a pesar de TENER
FLAGELO.

• Cual es la prueba de laboratorio para brucelosis?


• R = 2- MERCAPTOETANOL, es una inmunoglubulina IgG Vs BRUCELLA
BRUCELOSIS
• Cual es el medio de cultivo de elección para brucelosis?
• R = RUIZ CASTAÑEDA medio doble y ROSA DE BENGALA (fines
epidemiológicos)

• Que tratamiento utilizas de primera elección contra brucella?


• R = Combinado DOXICICLINA + GENTAMICINA de 3-6 semanas.

• Que tx utilizas en brucelosis en hueso y SNC?


• R = DOXICICLINA Y RIFAMPICINA

• Cual es el tratamiento de brucelosis en embarazada?


• R = TMP/SMZ+ RIFAMPICINA + ACIDO FÓLICO

• Cual es el tratamiento de niños menores de 12ª con brucelosis?


• R = TMP/SMZ + RIFAMPICINA

• Cual es el tratamiento en pacientes alérgicos a sulfas en brucelosis?


• R = RIFAMPICINA Y CEFTRIAXONA.
BOTULISMO
• Cual es el cuadro clínico del CLOSTRIDIUM BOTULINUM
transmitido por alimentos?
• R = Los síntomas comienzan en 6 HRS DESPUÉS A 2
SEMANAS, se manifiesta con DIPLOPÍA, VISIÓN BORROSA,
PTOSIS, disfagia, sensación de sequedad de mucosa oral,
DEBILIDAD MUSCULAR que afecta únicamente a los
hombros, miembros torácicos y mas tarde a los pélvicos.

• Cual es el manejo?
• R = Administración de TOXINA BOTULÍNICA con PREVENCIÓN
DE INSUFICIENCIA RESPIRATORIA que amerite ventilación
mecánica
PESTE BUBONICA
• Cual es el agente causal por la peste bubónica?
• R = YERSINIA PESTIS

• Cual es el cuadro clínico de la peste bubónica?


• R = Es la forma mas frecuente, se transmite por la PICADURA DE PULGA. Inicio con FIEBRE
ALTA, nauseas, MIALGIAS y ADENOPATÍAS DOLOROSAS REGIONALES, el PACIENTE
EVOLUCIONA A ESTUPOR, COMA Y MUERTE.
• Que formas clínicas tiene la peste?
• R = Peste bubónica, septicémica y neumónica

• Como diagnosticas peste?


• R = BIOPSIA DE GANGLIOS o EXUDADO FARÍNGEO

• Cual es el tratamiento de la peste?


• R = ESTREPTOMICINA, CLORANFENICOL O TETRACICLINAS.

• Cual es la prevención de la peste?


• R = Hay una vacuna con CEPA INACTIVADA F1
RABIA
• Cual es el agente causal de la rabia?
• R = Virus de la familia Rabdovirus

• Cual es el cuadro clínico de la rabia?


• R = Dolor en el sitio de la mordedura, fiebre, malestar general, nausea y vomito. Diez días
mas tarde hay manifestaciones del SNC pudiendo ser encefálica o paralitica y las 2 formas
evolucionan a coma - muerte.
A. ENCEFALICA: Se caracteriza por delirio, somnolencia, hidrofobia (espasmos laríngeos
dolorosos al beber agua)
B. PARALITICA: Da una parálisis ascendente.

• Cual es el manejo de la rabia?


I. Es necesario observar al animal los siguientes 10 DÍAS tomando UNA BIOPSIA DEL CUERO
CABELLUDO y analizarla con Ac fluorescente o PCR.
II. Atención de la LESIÓN LAVANDO CON AGUA Y JABÓN ABUNDANTE A CHORRO DURANTE
10 MINUTOS, para MUCOSA ORAL O NASAL CON SOLUCIÓN FISIOLÓGICA DURANTE 5
MINUTOS. Desinfectar la herida con agua oxigenada o tintura de yodo.
III. Si se requiere SUTURAR LA HERIDA se debe APLICAR PRIMERO INMUNOGLOBULINA
ANTIRRÁBICA HUMANA y se aproximan los bordes o se dan puntos temporales en caso
que la herida sea profunda,
IV. En exposición leve se administra vacunación antirrábica en la región deltoidea los días 0-3-
7-14 Y 28.
V. Se APLICARA INMUNOGLOBULINA INTRALESIONAL LA PRIMER ½ Y LA OTRA MITAD VÍA IM.
VI. Se usara SUERO HETEROLOGO EN CASO EXTREMO DE NO CONTAR CON
INMUNOGLOBULINA.
INFLUENZA
DENGUE Y FIEBRE AMARILLA
• Cual es el mosquito transmisor del dengue?
• R = AEDES AEPYPTI.

• Cual es el cc del dengue?


1) El DENGUE CLÁSICO se inicia con FIEBRE, BRADICARDIA, pulso lento, EXANTEMA
MACULAR PUNTIFORME, tos, ardor faríngeo, MIALGIAS, ARTRALGIAS.
2) El DENGUE HEMORRÁGICO se caracteriza por aumento en la permeabilidad vascular con
clasificación del I-IV.

• Que es lo que distingue a la fiebre amarilla del dengue laboratorialmente?


• R = CUERPOS DE INCLUSIÓN VIRAL llamados CUERPOS DE COUNCILMAN EN LA SANGRE.

• Cual es el cuadro clínico de la fiebre amarilla?


• R = FIEBRE, cefalea, mialgias, DOLOR LUMBOSACRO, ERITEMA OCULAR Y FACIAL,
PETEQUIAS EN ENCIAS Y MUCOSA NASAL, HEMATEMESIS, MELENA, ALBUMINURIA,
encefalitis o meningoencefalitis.

• Cual es el manejo de la fiebre amarilla y del dengue?


• R = SINTOMÁTICO
VIH
• Que células ataca el VIH?
• R = CD4

• Cual es el ciclo vital del VIH?


• R = A través de la proteína gp41, se realiza la fusión, posteriormente la RNA del VIH se
descubre e interna en la célula afectada, la enzima transcriptasa inversa del virion cataliza la
transcripción inversa del RNA en DNA, este se transfiere hacia el núcleo en el cual se integra
en los cromosomas por medio de la integrasa. Los macrófagos actúan como reservorio del
VIH y lo diseminan a otros sistemas

• Que se debe realizar como prevención en una persona que halla sido puncionada de
manera accidental con una aguja utilizada previamente en un paciente con VIH?
• R = INHIBIDORES NUCLEOSIDOS DE LA TRANSCRIPTASA REVERSA

• Como se manifiesta el síndrome retroviral agudo SRA en VIH?


• R = SÍNTOMAS SIMILARES AL RESFRIADO O MONONUCLEOSIS, fiebre, escalofríos, sudores
nocturnos y erupciones en la piel que dura de 1-3 SEMANAS.

• De acuerdo a la CDC como se realiza dx de SIDA?


• R = Recuento <200/MM DE CD4 + la presencia de ENFERMEDAD OPORTUNISTA
VIH
• Como dx VIH?
• R = ELISA y confirmatorio con WESTERN BLOOD

• Cuales son parte del grupo de fármacos que inhiben la TRANSCRIPTASA INVERSA en VIH?
• R = ZIDOVUDINA, didanosina, zalcitabina.

• Cuales son las indicaciones actuales para uso de tratamiento retro vírico en VIH?
1. Síndrome de infección aguda
2. INFECCIÓN CRÓNICA: Enfermedad sintomática o asintomática con T CD4 <350/NL o RNA
DEL VIH CON > 50, 000 COPIAS
3. PROFILAXIS POST- EXPOSICIÓN: Se recomienda utilizar combinaciones como ZIDOVUDINA
+ LAMIVUDINA + NEVERIPINA

• Cuales son las indicaciones para cambiar el tratamiento antirretrovirico?


1. Disminución <1 log del RNA del VIH con plasma a las 4 semanas de haber iniciado el
tratamiento
2. Incremento DEL TRIPLE O MAS DEL VALOR PLASMÁTICO DEL RNA NO ATRIBUIBLE A
INFECCIÓN concomitantes o vacuolas.
3. DISMINUCIÓN DEL PORCENTAJE DE CÉLULAS TCD4
4. Deterioro clínico
5. Efectos adversos

VIH
Como se manifiesta la neumonía por Pneumocystis Jiroveci?
• R = CD4 < 200/MM mas AUMENTO DE LA DHL, el diagnostico definitivo se obtiene por la TINCIÓN DE ESPUTO CON
WRIGHT-GIEMSA

• Cual es la causa mas frecuente de retinitis en pacientes con VIH?


• R = CITOMEGALOVIRUS

• Mecanismo por el que el VIH penetra a la célula?


• R = Pinocitosis

• Cual es la neoplasia maligna mas común en pacientes con VIH?


• R = Sarcoma de Kaposi

• Cual es el manejo del Sarcoma de Kaposi?


• R = Para la FORMA CUTÁNEA SE DA DOXORRUBICINA, para el INTESTINAL O VISCERAL SE DA
DOXORRUBICINA, BLEOMICINA Y VINBLASTINA.

• Como se maneja la profilaxis en pacientes con VIH de acuerdo al conteo de las células CD4?
1) CD4 < 200: VS P. JIROVECI con TMP/SMZ 1Tab/24 hrs C/3 DIA
2) CD4 75-100: VS M. AVIUM con CLARITROMICINA y se SUSPENDE CUANDO AUMENTAN LOS CD4
3) CD4 < 50: VS CITOMEGALOVIRUS con GANCICLOVIR
4) PPD con induración > 5 MM iniciar profilaxis VS M. TUBERCULOSIS con ISONIACIDA a 300 mg al día +
PIRIDOXINA durante 9-12 MESES.
VIH
• Tratamiento de primera elección para la retinitis por CITOMEGALOVIRUS?
• GANCICLOVIR

• Tratamiento para el herpes tipo 2 en pacientes con VIH?


• ACICLOVIR

• De que manera se puede inhibir el síndrome de desgaste?


• TALIDOMIDA

• Agente infeccioso de la lengua vellosa?


• Virus de EPSTEIN BARR ( relacionada con virus de mononucleosis y linfomas)
VIRUS TIPO 4
• Sitio mas común de lesión dermatológica, “SARCOMA DE KAPOSI” manchas color de piel?
• El sitio de afectación mas frecuente es la CARA EN PARPADOS Y PUENTE NASAL
RELACIONADO CON EL VIRUS TIPO 8

• Tratamiento de toxoplasmosis?
• PIREMITAMINA o sulfas + CLINDAMICINA en dosis altas
VIH
• Posibilidad de contagio por coito anal receptivo en el VIH?
• 1:100 1:30

• Zona donde se alojan el virus de la inmunodeficiencia es alta y útil para seguimiento del tx
de VIH?
• Biopsia de mucosa rectal otra opción es PCR

• Transmisión perinatal del VIH?:


• 13% al 40%

• Coito con inserción anal: 1: 1,000


• Transfusión: 1: 100,000
• Coito inserción vaginal: 1: 10,000 varia si la mujer se encuentra en periodo menstrual o si
tiene una ETS

• Aguja infectada?:
• 1-300

• Compartir agujas?:
• 1-150 (drogadictos)

• A que se refiere la regla de los 3 con relación a aguja infectada?


• Hepatitis B : 30% Hepatitis C: 3% HIV: .3%
VIH
• Diagnostico para valorar el tratamiento “respuesta terapéutica”
• RCP se realiza al mes y medio y a los 3 meses y posteriormente CADA 6 MESES (no se usa para diagnostico)

• Que pasa con una persona que tiene VIH y las siguientes enfermedades Concomitantes?
• Hepatitis B: Indiferente (no pasa nada)
• Hepatitis C: Agresividad
• HEPATITIS G: FRENA LA PROGRESIÓN DEL VIH

• Con respecto a la estatificación de etapas A-B-C y grados I al III?


• A: asintomáticos B: Síntomas Constitucionales (fiebre, perdida de peso, diarrea) C: Infecciones Oportunistas
(neumocistis jirovecci, linfoma del SNC, Criptosporidiasis, Toxoplasma del SNC, Micovacterium Avium intra
celular)
• I.- 500 copias II.-MAS DE 200 Y MENOS DE 500 III.-MENOS DE 200 copias

• INHIBIDORES DE TRANSCRIPTASA INVERSA -no análogo de los Nucleosidos- NNRTI


• De la ZIDOVUDINA AZT cual es el efecto indeseable? PROBLEMAS MEDULARES, anemia severa,
trombocitopenia, leucopenia

• DE LA EFEVIRENZ cual es el efecto indeseable?


• Insomnio y pesadillas (DEPRESIÓN)

• INHIBIDORES DE PROTEASAS
• Del Indinavir efecto indeseable mas frecuente: FORMADOR DE CÁLCULOS cólico renoureteral
• Ritonavir + lopinavir : Es el caletra efecto sinérgico
VIH
• Profilaxis (exposición en trabajadores de la salud)(exposición SEXUAL) que maniobras o
procedimientos?
• Tiene una posibilidad de infección de 1:300 SE REALIZA UN ELISA con fines legales, ( AL
MES)… si se considera EXPOSICIÓN DE BAJO RIESGO dar AZT+3TC X UN MES se REALIZA
ELISA SI ES NEGATIVO SE SUSP… si es exposición de alto riesgo AZT+3TC +UN TERCER
MEDICAMENTO…. NO DAR TX A LAS 72 POSTEXPOSICION

• Copias de CD4?
• Entre 50 y 100 copias en el momento del dx: MVAIC, CMV, Linfomas del SNC
• MENOS DE 200: Criptocococis, toxoplasmosis, NEUMOCISTIS JIROVECCI
• Entre 200 y 500: LENGUA VELLOSA, Sarcoma de Kaposi, TB Pulmonar, Infección por virus
herpes, candidiosis

• Px con HIV y tiene exposición con px con tb?


• Profilaxis: PPD MAS DE 5MM ISONIACIDA + RIFAMPICINA + PIRAZINAMIDA POR 12 MESES
+ PIRIDOXINA

• En que consiste el síndrome retroviral agudo?


1) Paciente con infección de VIH con SÍNTOMAS A CORTO PLAZO (de 2 A 3 SEMANAS y
sucede en el 40%de los casos) Y CONSISTE EN FIEBRE ADENOPATÍA, RASH CUTÁNEO
COLOR SALMON Y HEPATOMEGALIA… CUADRO MUY PARECIDO AL DE LA
MONONUCLEOSIS INFECCIOSA,
2) DX ELISA A LAS 6 SEMANAS Y 3 MESES si sale positivo se realiza el WESTERN BLOT PARA
CONFIRMARLO
VIH
• Combinaciones aprobadas:
I. Tenofovir (TDF)+ emtricitabina (FTC) y efavirenz (EFV)
II. Atazanavir (ATV) + ritonavir (rtv)+tenofovir (TDF) / emtricitabina (FTC) (si no funciona el primer
esquema)
III. Caletra +tenofovir (TDF) / emtricitabina (FTC) (El caletra se puede cambiar con AZT + 3TC)

• Efectos colaterales?
I. Zidovudina….. ANEMIA por hipoplasia medular
II. Nerfinavir…. DIARREA
III. Didanosina…. PANCREATITIS
IV. Zalcitabina….. NEUROPATÍA PERIFÉRICA
V. Estaduvidina…. NEUROPATÍA PERIFÉRICA (lamivudina +zidovudina) 3TC….. Neuropatia periférica
VI. Abacavir….. FIEBRE RASH, HIPERSENSIBILIDAD FATAL
VII. Tenofovir…. TOXICIDAD RENAL
VIII. Saquinavir…. LÍPIDOS ELEVADOS

• Px con embarazo + VIH Cual es el tx?

I. ZIDOVUDINA (2do y 3er trimestre)… se continua en el trabajo de parto (para evitar la


infección vertical)

II. POSPARTO (NO LACTANCIA 10-20 tiene posibilidad de infectar) 2da opción LAMIVUDINA
VIH
• Cuales son parte del grupo de fármacos INHIBIDORES DE
PROTEASA en VIH?
• R = SaquinAVIR, ritonAVIR, indinAVIR.

• Cuales son parte del grupo de fármacos INHIBIDORES DE LA


FUSIÓN?
• R = Enfubitida, que entre sus efectos adversos se encuentra la
hipersensibilidad y neumonía bacteriana

• Cuales son parte del grupo de fármacos que INHIBEN LA


TRANSCRIPTASA INVERSA en VIH?
• R = ZIDOVUDINA, didanosINA, zalcitabINA.
ASPERGILOSIS
• Como se adquiere la aspergilosis?
• R = Por inhalación de ESPORAS DEL HONGO, las cuales se encuentran en HOJAS SECAS,
GRANEROS DE MAIZ, ESTIÉRCOL Y VEGETALES EN DESCOMPOSICIÓN.

• En los pacientes con rinitis crónica por aspergilus, que datos de laboratorio encuentras en
el moco?
• R = Rico en EOSINOFILOS y cristales de CHARCOT-LEYDEN

• En que consiste la aspergilosis pulmonar endobronquial saprofita o aspergiloma?


• R = En el CRECIMIENTO DEL HONGO dentro de las cavidades pulmonares, SECUNDARIAS
GENERALMENTE A TB, SARCOIDOSIS, HISTOPLASMOSIS O BRONQUIECTASIAS.

• Como dx aspergilus?
• R = CULTIVO DE SABOURAD, detección de ANTICUERPOS, galactomanano en suero (chbts
en su pared celular) y biopsia

• Cual es el tratamiento de aspergilus?


• R = Esteroides, ANFOTERICINA B, LOBECTOMÍA EN CASO DE ASPERGILOMA o debridacion de
tejido infectado.

• Cual es el tratamiento de elección en caso de toxicidad secundaria al DESOXICOLATO de


ANFOTERICINA B en caso de aspergilosis invasiva?
• R = VORICONAZOL 6mg/kg/dia.
CANDIDOSIS
• Como se manifiesta la CANDIDIASIS DIGESTIVA y como se trata?
• R = Se ASOCIA a lesiones ORALES Y PERIANALES, produce DIARREA CON MOCO, el
diagnostico se realiza con el EXAMEN DIRECTO DE LAS HECES CON KOH y se trata
con NISTATINA ORAL.

• Como se manifiesta la CANDIDIOSIS URINARIA y como se trata?


• R = Se presenta como CISTITIS, pielonefritis, BOLAS FÚNGICAS RENALES, ABSCESOS
RENALES o necrosis de las papilas. Se trata con fluconazol o ANFOTERICINA B.

• Como se diagnostica CANDIDIOSIS PULMONAR y como se trata?


• R = Cuando realizas HEMOCULTIVOS POSITIVOS o ANTÍGENO MANAN DEL
SUERO. El tratamiento se realiza con ANFOTERICINA B, en los pacientes que no la
toleran se utiliza FLUCONAZOL.

• Como se manifiesta la ENDOCARDITIS por candidiosis?


• R = FIEBRE persistente, SOPLOS CARDIACOS y ESPLENOMEGALIA, en algunos casos
hay embolización a grandes arterias, como la iliaca o femoral. LA VÁLVULA MAS
AFECTADA ES LA MITRAL.
CANDIDOSIS
• Como diagnosticas ENDOCARDITIS por cándida?
• R = Ecocardiagrafia, hemocultivos seriados y detección de
ANTÍGENO MANAN en suero?

• Cual es el tratamiento de endocarditis por cándida?


• R = RESECCIÓN DE VÁLVULA afectada y ANFOTERICINA B

• Como se presenta la candidiasis diseminada aguda o sepsis?


• R = Se presenta en PACIENTES NEUTROPENICOS, CON FIEBRE,
LEUCOCITOSIS, TROMBOCITOPENIA, inestabilidad
hemodinámica y choque.
CRIPTOCOCOCIS (AVES, PALOMAS)
• Cual es el agente causal de la criptococosis y donde se
alberga?
• R = CRYPTOCOCCUS NEOFORMANS, se encuentra en HECES
DE PALOMAS (CUIDADORES DE AVES) O NIDOS

• Como diagnosticas criptococosis?


• ANTÍGENO MANAN en suero o biopsia.
• En LCR con TINCIÓN DE TINTA CHINA

• Como se trata la criptococosis?


• R = ANFOTERICINA B y 5-fluorocitosina
HISTOPLASMOSIS (CAVERNAS)
• Donde se alberga el Histoplasma capsulatum?
• R = En forma de moho en suelo RICO EN NITRÓGENO, como el de
los GALLINEROS, o donde duermen las aves y CAVERNAS DE
MURCIÉLAGOS

• Como diagnosticas histoplasmosis?


• R = El método de elección es el CULTIVO, pero tarda mucho y
haces frotis de los fluidos o tejidos infectados, las muestras para
cultivo en HISTOPLASMOSIS DISEMINADA SE OBTIENEN DE
MEDULA ÓSEA, HÍGADO, GANGLIOS LINFÁTICOS O LESIONES
MUCOCUTANEAS.

• Cual es el manejo de la hispoplasmosis?


• R = Anfotericina B
MUCORMICOSIS
• Cual es el agente etiológico de la mucormicosis?
• R = MUCOR, ABSIDIA, RIZOPUS denominado ficomicetos.

• Como se manifiesta la infección de los senos paranasales en mucormicosis?


• R = Predominantemente a PACIENTES CON DM mal tratada, con FIEBRE,
congestión nasal, SECRECIÓN NASAL SANGUINOLENTA, DOLOR SINUSAL Y
DIPLOPÍA, a la exploración física se observa DISMINUCIÓN DE LOS MOVIMIENTOS
OCULARES, quemosis y PROPTOSIS y SI INVADE EL SENO FRONTAL PRODUCE COMA.

• Como se manifiesta la mucormicosis gastrointestinal?


• R = Se disemina al peritoneo produciendo la muerte en 70 días

• Como se diagnostica mucormicosis?


• R = FROTIS EN FRESCO donde se ven HIFAS HIALINAS o en BIOPSIAS donde se
observa NECROSIS ISQUÉMICA o hemorrágica

• Cual es el manejo de la mucormicosis?


• R = El tratamiento consiste en altas dosis de ANFOTERICINA B, ASEO QUIRÚRGICO
DE LOS TEJIDOS.
COCCIDIOIDOMICOSIS
• Cuales son los signos y síntomas de la coccidioidomicosis?
• R = TOS, RINORREA, febricula, dolores musculares y en la RADIOGRAFÍA DE
TÓRAX CON CAVITACIÓN.

• - A 43-year-old man developed a cough shortly after returning from a 1-


month hiking trip in California. While there, he was hiking in the central
California valleys. During his trip, he had developed a “flu-like” illness
consisting of fever, cough, and muscle pains, which resolved spontaneously.
A CXR shows a thin-walled cavity in the right upper lobe, and the sputum
reveals fungal elements. Which of the following is the most likely causative
organism?

• (A) ringworm
• (B) Cryptococcus neoformans
• (C) Candida albicans
• (D) mycobacteria
• (E) coccidioidomycosis
BLASTOMICOSIS
• Cual es el agente causal de blastomicosis?
• R = BLASTOMYCES DERMATITIDIS

• Cuales son las manifestaciones clínicas de blastomicosis?


• R = PULMONAR Y CUTÁNEA. NEUMONÍA aguda que no responde al tratamiento y
CURA ESPONTÁNEAMENTE. Las LESIONES CUTÁNEAS predominan en las ZONAS
EXPUESTAS, característicamente son PAPULAS, NÓDULOS O PLACAS BIEN
DELIMITADAS, únicas o múltiples, NO DOLOROSAS NI PRURIGINOSAS, que
evolucionan a lesiones verrugosas, costras o ulceras.

• Como diagnosticas blastomicosis?


• R = De ELECCIÓN CULTIVO pero tarda mucho, se pueden identificar organismos con
la TINCIÓN DE PLATA, METENAMINA O ACIDO PERIODICO DE SCHIFF en la biopsia
de los tejidos infectados o en la citología de esputo. Donde se observan CÉLULAS
LEVADURIFORMES CON YEMAS DE BASE AMPLIA.

• Cual es el tratamiento de blastomicosis?


• R = En la forma LEVE - ITRACONAZOL, en la forma GRAVE - ANFOTERICINA B
PALUDISMO
• Cual es el agente causal de paludismo o malaria?
• R = PARASITOSIS intracelular eritrocitaria por PLASMODIUM, que se transmite
por el MOSQUITO ANOPHELES

• Cuantas especies de plasmodium existen?


• R = 4, P. falciparum, P. vivax, P. ovale y P. malariae

• Cual es el ciclo biológico del plasmodium?


• R = Los mosquitos se infectan al ingerir sangre infectada con micro y
macrigametocitos. Dentro del mosquito, se multiplica sexualmente en su intestino,
produciendo los ooquistes, y al madurar se rompen liberando ESPOROZOITOS, que
migran a las glándulas salivales del mosquito y son la forma infectante en los seres
humanos. Ya en el TORRENTE SANGUÍNEO viajan a hígado donde se reproducen y
forman los MEROZOITOS que tienen la capacidad de infectar los eritrocitos.

• Que produce la lisis eritrocitaria al contraer paludismo?


• R = La PLASMEPSINA, que es una proteasa del acido aspartico que DEGRADA LA
HEMOGLOBINA.
PALUDISMO
• Cual es el tipo de genero en paludismo mas frecuente?
• R = P. FALCIPARUM, es la mas frecuente y de peor pronostico con
fiebre > 40

• Como se diagnostica el paludismo?


• R = OBSERVACIÓN DE PARASITO EN FROTIS SANGUÍNEO O GOTA
GRUESA. La TINCIÓN DE GIEMSA permite observar las
granulaciones de Schuffner.

• Como se trata el paludismo?


• R = QUININA Y CLOROQUINA. El SULFATO DE QUININA se usa en
RESISTENCIA A CLOROQUINA
BORRELIOSIS O ENFERMEDAD DE
LYME
• Cual es el agente causal de la enfermedad de Lyme?
• R = Espiroqueta BORRELIA BURGDORFERI GRAM (-) de color violeta con
tinción Giemsa que se CULTIVA EN MEDIO NOGUCHI. Se adquiere por
MORDEDURA DE GARRAPATA del genero IXODES.

• Cual es el cuadro clínico de la enfermedad de Lyme?


• R = Se divide en 3 etapas
I. ERITEMA CRÓNICO O MIGRANS, pápulas eritematosas que se EXTIENDEN
DE FORMA CENTRIFUGA DEJANDO LA PARTE CENTRAL DEL CUERPO
SANA, acompañada de síntomas generales como astenia, adinamia,
fiebre, mialgias y cefalea.
II. Alteraciones neurológicas, cardiacas y reumáticas
III. POLIARTRITIS MIGRATORIA, alteraciones dermatológicas y neurológicas

• Como se diagnostica la enfermedad de Lyme?


• R = ELISA o PCR
BORRELIOSIS O ENFERMEDAD DE
LYME
• Tratamiento de elección para enf de Lyme?
• R = DOXICICLINA por mes y medio

• Cual es el tratamiento para borreliosis en su etapa


precoz?
• R = DOXICICLINA.

• Cual es el tratamiento para borreliosis en su etapa


tardía?
• R = CEFTRIAXONA
LEPTOSPIROSIS
• Cual es el vector de la leptospirosis, que enzimas produce y que nutrientes necesita?
• R = Leptospira interrogans en ORINA DE RATA. PRODUCE CATALASA Y HIALURODINASA y requieren
TIAMINA B1 Y VITAMINA B12 PARA SU DESARROLLO

• Cuales son las manifestaciones clínicas de la Leptospirosis?


• R = De inicio súbito, con FIEBRE, derrame conjuntival, epistaxis, MIALGIAS, cefalea intensa, EXANTEMA Y
HEPATOMEGALIA.

• Que es el síndrome de Weil en Leptospirosis?


• R = Al cursar la primera fase, viene una ASINTOMÁTICA para posteriormente PRESENTARSE
LEPTOSPIROSIS ANICTERICA.

• Como se manifiesta el síndrome de Weil?


• R = CON FIEBRE, CEFALEA, MIALGIAS, EXANTEMA, miocarditis, ICTERICIA, insuficiencia hepática,
INSUFICIENCIA RENAL, FENÓMENOS HEMORRÁGICOS, LEPTOSPIURIA y daño al SNC (MENINGITIS
ASEPTICA).

• Cual es la triada clásica del síndrome de Weil?


• R = Daño en HÍGADO, RIÑÓN y SNC.

• Cual es el tratamiento de la Leptospirosis?


• R = PENICILINA, estreptomicina o tetraciclinas
TULAREMIA
• Cual es el agente causal de Tularemia?
• R = FRANCISELLA TULLARENSIS POR PICADURA DE GARRAPATA
• Cual es el cuadro clínico de Tularemia?
• R = De inicio súbito con FIEBRE, ASTENIA, ADINAMIA, hiporexia, ataque al
estado general, posteriormente hay diversas formas clínicas:
ULCEROGANGLIONAR, manifestada con adenomegalias y ulceras cutáneas;
TIFOIDE GLANDULAR, donde se afecta el pulmón, ganglios, hígado y bazo.
SE PRESENTA CON NEUMONÍA, NECROSIS VISCERAL Y SEPSIS.

• Como diagnosticas Tularemia?


• R = En medio de CULTIVO DE AGAR SANGRE cisteína o ELISA.
• Como tratas la Tularemia?
• R = ESTREPTOMICINA, tetraciclina.
RICKETSIA
FIEBRE Q
LEISHMANIASIS
• Que vectores transmiten leishmaniasis?
• R = MOSQUITOS del genero LUTZOMYA Y PHLEBOTOMUS, LEISHMANIA pertenece a la familia
del tripanosoma

• Cual es el ciclo biológico de la leishmaniasis?


• R = Al picar el mosquito a un mamífero infectado, ingiere los macrófagos parasitados, que
posteriormente se destruyen, dejando libre al parasito en su forma flagelar e infectante
(promastigote), se reproducen por fision binaria longitudinal, y migran hacia la porción anterior
del aparato digestivo, hasta la probocide (boca). La infección se produce al alimentarse de un
nuevo huésped, inocula los PROMASTIGOTES, que al ser fagocitados se transforman en
AMASTIGOTES, donde se multiplican, causan la lisis de las células que los contienen e infectan a
otras células.

• En la leishmaniasis viceral o Kala-Azar a que células afecta el parasito?


• R = A los MACRÓFAGOS del sistema retículo endotelial

• Como se manifiesta la leishmaniosis cutánea?


• R = En el sitio de inoculación, PAPULA ERITEMATOSA/PRURIGINOSA – NODULO – ULCERA PIEL/TEJ.
CEL. SUB – EXUDA LIQ. SEROHEMATICO – ADENOPATÍA REGIONAL.

• Como se manifiesta la leishmaniosis mucocutanea o espundia?


• R = ULCERA CUTÁNEA – pasan years – LESIÓN OBSTRUCTIVA ÚNICA/MULTIPLE –
LARINGE/TABIQUE NASAL/ANO Y VULVA.
LEISHMANIASIS
• Como se manifiesta la leishmaniasis viceral o Kala-Azar (fiebre negra)?
• R = Hay HIPERPIGMENTACIÓN CUTÁNEA DE LAS EXTREMIDADES, mal estar general, DOLOR ABDOMINAL, HEPATO-
ESPLENOMEGALIA, LINFADENOPATIAS, DIARREA es fatal si no se trata adecuadamente.

• Como diagnosticas leishmaniasis?


• R = VER EL AMASTIGOTE EN BIOPSIAS DE PIEL, lesiones mucosas, hígado, ganglios linfáticos, medula ósea tenidas con
Giemsa. En los CULTIVOS de los tejidos se busca el PROMASTIGOTE.

• Con que prueba de laboratorio diagnosticas leishmaniasis?


• R = INTRADERMORREACCIÓN DE MONTENEGRO y ELISA.

• Cual es el tratamiento de la leishmaniasis viceral?


• R = Para la FORMA VISCERAL el tratamiento DE ELECCIÓN ES ESTIBOGLUCONATO DE SODIO O ANTIMONIATO DE
GLUTAMINA IM POR 50 DÍAS.

• Que medicamentos empleas en caso de que la leishmaniasis visceral no responda a los antiamoniales?
• R = ANFOTERICINA LIPOSOMAL POR 5 DÍAS Y EL ISETHIONATO DE PENTAMIDINA.

• Que medicamento utilizas en leishmaniasis cutánea?


• R = ISETIONATO DE PENTAMIDINA, paromomicina APLICACIÓN TÓPICA 2 VECES AL DÍA POR 15 DÍAS.
GIARDIASIS
• Como diagnosticas giardiasis?
• R = Observación de QUISTE EN HECES, ELISA.

• Que componente de los trofozoitos de giardiasis te causa


hipotrofia de las vellosidades intestinales
• R = El DISCO SUCTOR

• Cual es el tratamiento de elección para la giardiasis?


• R = El TINIDAZOL con dosis única de 2 gr. METRONIDAZOL. La
furazolidona causa hemolisis en pacientes con déficit de 6-
GDP.
AMEBOSIS
• Cual es el cuadro clínico abdominal de la amebiasis por E. Hystolitica?
1. COLITIS AMEBIANA AGUDA manifestada por dolor abd tipo cólico, tenesmo y
EVACUACIONES DIARREICAS CON MOCO Y SANGRE.
2. COLITIS FULMINANTE con mortalidad mayor al 50% se presenta con diarrea sanguinolenta
intensa, ataque al estado general, fiebre, dolor abdominal intenso y PERFORACIÓN
INTESTINAL con choque séptico secundarios a la extensión de la ulcera hasta la serosa.
3. AMEBOMA O GRANULOMA AMEBIANO que cursa con DIARREA SANGUINOLENTA Y
OBSTRUCCIÓN INTESTINAL.

• Cual es el cuadro clínico de la amebosis cutánea?


1. ULCERAS FUGEDENICAS (de COLOR PURPURA, de crecimiento rápido, MUY DOLOROSAS y
ABUNDANTE MATERIAL NECRÓTICO),
2. LESIONES VEGETANTES (friables, de evolución subaguda, LOCALIZADAS EN LOS PLIEGUES
CUTÁNEOS),
3. ENTAMOEBOSIS (se presenta por una reacción de hipersensibilidad, CON LESIONES
SEMEJANTES A URTICARIA.

• Como se manifiesta el absceso hepático amebiano?


• R = De inicio agudo, CON HEPATOMEGALIA Y DOLOR EN HIPOCONDRIO DERECHO QUE SE
IRRADIA HACIA EL HOMBRO.
AMEBOSIS
• Como diagnosticas amibiasis?
• R = COPROPARISOTOSCOPICO EN FRESCO, hematoxilina férrica o TINCIÓN
DE GOMORI, en busca de TROFOZOITOS, ELISA. En la piel se busca
trofozoito.

• Cual es el tratamiento de amibiasis?


1) METRONIDAZOL, TINIDAZOL.
2) Dehidrohemetina y emetina son amebicidas de la pared intestinal, pero no
sobre la luz intestinal siendo estos YODOQUINOL y paramomicina.

• Que medicamento es eficaz para la amebiasis hepática?


• R = CLOROQUINA
TRIPANOSOMIASIS AFRICANA O
ENFERMEDAD DEL SUENO
• Cual es el agente etiológico de la TRIPANOSOMIASIS AFRICANA O ENFERMEDAD DEL
SUEÑO y cual es su vector?
• R = Tripanosoma BRUCEI, Tripanosoma RHODESIENSE y Tripanosoma GAMBIENSE. Su
vector es la MOSCA TSE TSE.

• Cual es el agente causal de la tripanosomiasis americana o ENFERMEDAD DE CHAGAS


y cual es su vector?
• R = TRIPANOZOMA CRUZI y su vector es la chinche

• Cual es el ciclo biológico de los tripanosomas?


• R = Al alimentarse de un individuo con la infección, los vectores ingieren el amastigote,
que se multiplica dentro del tracto digestivo y las FORMAS INFECCIOSAS
(TRIPOMASTIGOTE) son eliminadas en las heces; la infección se produce al depositar las
heces en una herida de la piel antes de picar en las membranas mucosas o en la
conjuntiva. Posteriormente, se multiplican en el sitio de la entrada, y entran a la sangre
como tripomastigotes circulantes e invaden a las células, CON PREDILECCIÓN POR EL
MIOCARDIO, MUSCULO LISO Y SNC.
TRIPANOSOMIASIS O ENFERMEDAD
DE CHAGAS
• Como se manifiesta la tripanosomiasis o ENFERMEDAD DE CHAGAS POR T. CRUZY?
• R = En la piel, en el sitio de entrada se observa CHAGOMA que es PARECIDO A LA FURUNCULOSIS CON
LINFADENOPATIA REGIONAL. En conjuntiva, edema bipalpebral unilateral, CONJUNTIVITIS Y
LINFADENITIS PRE AURICULAR (SIGNO DE ROMANA) CON FIEBRE Y ESPLENOMEGALIA LEVE.
CRONICAMENTE se manifiesta por INSUFICIENCIA CARDIACA PREDOMINANTE DEL LADO DERECHO, con
arritmias ventriculares y episodios de tromboembolia sistémica o pulmonar. MEGA ESÓFAGO Y
MEGACOLON con síntomas de disfagia, constipación intensa, hipertrofia parotidea.

• Como se diagnostica la tripanosomiasis americana o ENFERMEDAD DE CHAGAS?


• R = En la FASE AGUDA se observa con la TINCIÓN DE GIEMSA de sangre anti coagulada. Diagnostico
serológico se basa en hemaglutinación indirecta o ELISA.

• Como se diagnostica la TRIPANOSOMIASIS AFRICANA?


• R = Para T. GAMBIENSE por medio de PRUEBAS DE AGLUTINACIÓN y para T. RHODESIENSE es mediante
la observación del parasito en la tinción de GIEMSA.

• Cual es el tratamiento de elección para la TRIPANOSOMIASIS AMERICANA O CHAGAS?


• R = NIFURTIMOX por 30-90 DÍAS o BENZIDIMAZOL de 30-90 DIAS.
• Cual es el tratamiento de la TRIPANOSOMIASIS AFRICANA?
• R = Para T. GAMBIENSE ES PENTAMIDINA POR 10 DÍAS. Para T. RHODESIENSE ES LA SURAMINA
POR 7 DÍAS.
TENIASIS
• Cual es el agente causal de la teniasis?
• R = T. saginata y T. solium

• Cuales son las partes estructurales que componen a la tenia?


• R = En su extremo anterior tienen una estructura llamada ESCÓLEX (CABEZA), seguida de un
cuello. En su escólex presentan cuatro ventosas orales. En sus ventosas tiene un róstelo, con
una corona doble de ganchos (escolices). El CUERPO (ESTRÓBILO) se conforma por varios
segmentos llamados PROGLOTIDOS, que en su interior tienen ÓRGANOS SEXUALES.

• Cuanto miden los adultos de T. SAGINATA?


• R = 2-7 METROS de largo

• Cuanto puede llegar a medir la T. SOLIUM?


• R = 15-18 METROS

• Cual es el ciclo biológico de la teniasis?


• R = La teniasis intestinal es causada por la INGESTA DE CISTICERCOS. Al legar a intestino
delgado, se unen a la pared intestinal. De los 2 a los 4 meses de la infección, alcanzan su
madurez, caracterizada por la presencia de proglotidos gravidos que liberan huevos y se
excretan por las heces. La cisticercosis ocurre por la ingestión de los alimentos o agua
contaminados con excremento humano que contiene huevos o proglotidos. Los huevos se
abren en el intestino, liberando a las oncoesferas, que invaden la pared intestinal pasando al
torrente sanguíneo y linfático para migrar hacia musculo estriado, cerebro e hígado.
TENIASIS
• Cual es el cuadro clínico de la teniasis?
• R = Normalmente es asintomática. Cuando cursa con síntomas son:
PLENITUD ABDOMINAL, FATIGA, DOLOR LEVE OCASIONAL E HIPOREXIA.

• Que déficit vitamínico cursa con teniasis?


• R = Déficit de VITAMINA B12

• Cual es el cuadro clínico de CISTICERCOSIS en el SNC?


• R = CRISIS CONVULSIVAS, cefaleas, vomito, HIPERTENSIÓN INTRACRANEAL.

• Como diagnosticas la teniasis?


• R = COPROPARASITOSCOPICO SE OBSERVAN HUEVOS, ELISA.

• Cual es el tratamiento para teniasis?


• R = PRAZICUANTEL a DOSIS ÚNICA DE 10 MG/KG. Ninguna droga mata a
los huevos de tenia solium. NICLOSAMIDA para TENIA SAGINATA
NEUROLOGIA

ENARM
MANIFESTACIONES DE LESION AL SNC
• Como se muestra una lesión del TRACTO ÓPTICO?
• R = Como CUADRANTOPSIA homónima contralateral.

• Que caracteriza al síndrome de Horner?


• R = Miosis, ptosis y anhidrosis debido a lesión de las vías del SISTEMA SIMPÁTICO ispolaterales.
1) Miosis: Se debe a denervación simpática del musculo tarsal
2) Anhidrosis: Consecuencia de la denervación simpática del musculo tarsal
3) Para localizar la lesión se realizan pruebas con cocaína, hidroxifentamina y fenilefrina.

• Que ocasiona las pupilas de Argyll Robertson?


1) Son PUPILAS IRREGULARES Y MÁS PEQUEÑAS de lo normal en la oscuridad.
2) Demuestran falta del reflejo fotomotor, tanto directo como consensual, sin embargo conservan la
acomodación.
3) Se asocia a NEUROSIFILIS.

• Que ocasiona las pupilas de Holmes-Adie?


1) Se trata de una PUPILA GRANDE Y TÓNICA de forma irregular
2) Tiene reacción pobre ante estimulo luminoso
3) Suele indicar una lesión de las vías parasimpáticas posganglionares, a nivel del ganglio ciliar o los nervios
ciliares cortos.
4) Se ve una REACCIÓN CONSTRICTORA EXAGERADA ante la administración de agonistas muscarinicos como la
meticolina o la PILOCARPINA.
5) Se asocian a reflejos de estiramiento muscular disminuidos o ausentes.
MANIFESTACIONES DE LESION AL SNC
• Cuales son las manifestaciones oculares en caso de obstrucción de las RADIACIONES
ÓPTICAS?
• R = CUADRANTOPSIAS HOMONIMAS

• -A 75-year-old woman presents with symptoms of visual change and facial weakness. On
examination, the pupils are equal and reactive to light, the fundi appear normal, and there is
a right homonymous visual field defect. Which of the following is the most likely cause of the
right homonymous hemianopia?

• (A) right optic nerve


• (B) chiasm
• (C) right optic radiations
• (D) right occipital lobe
• (E) left optic radiations

• - The hemianopia is due to a lesion of the left optic radiations. The posterior cerebral artery
arises from the basilar artery but is sometimes a branch of the internal carotid. With
posterior cerebral artery lesions affecting the occipital cortex, it is possible for the
hemianopia to be an isolated finding.
• CORRELACIÓN TOPOGRÁFICA DE DEFECTOS DEL CAMPO VISUAL. A: abolición CV izquierdo (lesión nervio óptico izquierdo). B: escotoma
yuncional. C: hemianopsia bitemporal heterónima (lesión quiasmática). D: hemianopsia homónima derecha (lesión quiasmática). E:
sectoronopsia cuádruple homónima derecha (lesión ganglio geniculado izquierdo). F: Sectoranopsia horizontal derecha (lesión ganglio
geniculado izquierdo). G: Cuadrantanopsia homónima inferior derecha (lesión lóbulo parietal izquierdo). H: Cuadrantanopsia homónima
superior derecha (lesión lóbulo temporal izquierdo). I: Hemianopsia homónima derecha con respeto macular (lesión occipital izquierda).
MANIFESTACIONES DE LESION AL SNC
• Que es la disautonomia?
• R = Proceso patológico central o periférico que se manifiesta por anomalías de la regulación
de la TA, diaforesis termorreguladora, función sexual.

• Que es el mioclono?
1) Es común en epilepsia, presentando ESPASMOS. Se presenta en casos de anoxia,
hiperuricemia.
2) El tratamiento es con levodopa, privación alcohólica y responde a acido valproico y
benzodiacepinas.

• Que motoneurona se ve afectada en la espasticidad?


• R = MOTONEURONA SUPERIOR

• Cual es tratamiento de espasticidad?


• R = Fisioterapia, dantroleno debilita la contractura muscular al interferir con el calcio,
diazepam, inyección de fenol para disminuir la espasticidad selectiva o toxina botulínica.
MANIFESTACIONES DE LESION AL SNC
• En que zona se encuentra la lesión en un paciente estuporoso/comatoso en ausencia
unilateral de respuesta a pesar de la aplicación de estímulos en ambos lados del cuerpo?
• R = CORTICOESPINAL

• En que zona se encuentra la lesión en paciente en ausencia bilateral de respuesta a


estímulos?
• R = Afección de TALLO ENCEFÁLICO, lesión bilateral de vía piramidal

• La postura de decorticacion a que daño se debe?


• R = Lesión de CAPSULA INTERNA y del PEDÚNCULO CEREBRAL ANTERIOR.

• La postura de decerebracion a que daño se debe?


• R = DESTRUCCIÓN MESENCÉFALO y de la porción anterior del puente de vacilio

• Que causa el síndrome de encerramiento o estado desaferentado?


1) Lesiones agudas que afectan principalmente la PARTE ANTERIOR DEL PUENTE DE VACILIO Y
RESPETAN EL TEGUMENTO.
2) Tienen respuesta ocular y están consientes pero en estado comatoso y se RECUPERAN DE
2-3ª.
Posición Decorticación Posición Descerebración
MOTONEURONAS
• Que caracteriza a las lesiones de la motoneurona?
• R = Debilidad sin perdida sensorial

• Cual es la clasificación de la enfermedad de motoneurona y que caracteriza a cada una de ellas?


1) Parálisis bulbar progresiva: degeneración que predomina en núcleos motores de pares
craneales
2) Parálisis pseudobulbar: se debe a enfermedad cortico bulbar alterando neurona motora
superior
3) Atrofia muscular espinal progresiva: Hay degeneración de las astas anteriores de la medula
espinal afectando a neurona motora inferior
4) Esclerosis lateral primaria: Déficit puro de neurona motora superior en extremidades inferiores
5) Esclerosis lateral aminotrofica: Déficit mixto de NMS e INFERIOR en extremidades pudiendo
alterar el estado cognitivo con DEMENCIA FRONTOTEMPORAL O PARKINSONISMO.

• Cual es el cuadro clínico de enfermedad degenerativa de neuronas motoras?


1) Alteración bulbar: dificultad para la deglución, masticación, tos, respiración y disartria.
2) Parálisis bulbar progresiva: descanso del paladar disminuyendo reflejo nauseoso, tos débil y
desgaste muscular. Lingual con fasciculaciones.
3) Parálisis pseudobulbar: Contractura y espasmo lingual, disminución de la motilidad de la
lengua. Hay trastorno motor en extremidades con esfínteres normales
4) ESCLEROSIS LATERAL AMINOTROFICA: ES LETAL de 3-5 años comúnmente POR INFECCIÓN
PULMONAR, presentándose con periodos de REMISIÓN Y EXACERBACIÓN DE DIFICULTAD
PARA CAMINAR, TRAGAR , TORPEZA Y FINALMENTE RESPIRAR CON CARÁCTER
AUTOSOMICO DOMINANTE EN GEN 21.
MOTONEURONAS
• Cuales son los signos positivos y negativos que indican lesión a la MOTONEURONA SUPERIOR o síndrome piramidal?
1) Positivos: ESPASTICIDAD, HIPERREFLEXIA en reflejos de estiramiento muscular, clonus, distonía, atetosis y exaltación de
reflejos cutáneos como signo de Babinski
2) Negativos: Debilidad o paresia hemicorporal y pérdida de la destreza, coordinación y control motor.
3) La combinación de ambos afectan la ejecución normal del movimiento con un deterioro en la calidad de vida del paciente.

• Cual es el cuadro clínico del síndrome de la MOTONEURONA INFERIOR?


• R = ATROFIA muscular por denervación, parálisis o PARESIA FLÁCIDA de todos los músculos de la unidad motora afectada,
HIPOTONÍA, hiporreflexia o ARREFLEXIA al interrumpirse la parte eferente del arco reflejo y fasciculaciones producidas por la
actividad espontanea de la unidad motora.

• Cuales son las características clínicas del síndrome de la columna posterior?


1) Se caracteriza por perdida o DISMINUCIÓN DE LA SENSACIÓN DE VIBRACIÓN, SENSACIÓN DE POSICIÓN, DISCRIMINACIÓN DE
DOS PUNTOS, TACTO Y RECONOCIMIENTO DE LA FORMA
2) Estimulos que no eran dolorosos SE CONVIERTEN EN DOLOROSOS y LOS ESTÍMULOS DOLOROSOS SE DESENCADENAN CON
UMBRALES DE ESTIMULACIÓN MAS BAJOS.

• Que caracteriza al síndrome de la arteria espinal anterior?


• R = También conocido como síndrome medular ventral que ocasiona PARAPLEJIA O CUADRIPLEJIA, perdida bilateral de la
sensación de dolor y temperatura por debajo del nivel de la lesión.

• Que estudios realizas para diagnosticar la enfermedad de la motoneurona?


• R = ELECTROMIOGRAFÍA: muestra DENERVACIÓN, EN ESCLEROSIS LATERAL AMINOTROFICA debe presentarse en 3
extremidades mínimo. LINFOMA DE HODKING puede cursar con NEUROPATÍA MOTORA.

• Cual es el tratamiento de lesiones de neuronas motoras?


• R = RICUZOL: disminuye la LIBERACIÓN PRESINAPTICA DE GLUTAMATO haciendo lenta la ELA progresiva.
PLASMAFERESIS logra mejoría. ANTICOLINERGICOS: atropina y amitriptilina
MOTONEURONAS
• - A 53-year-old man complains of clumsiness with both hands, like having difficulty doing up buttons or using his keys. Physical
examination reveals fasciculations of his thigh and forearm muscles; diffuse muscle weakness, loss of muscle bulk, and
increased tone in the upper and lower limbs. There is generalized hyperreflexia, and positive Babinski signs bilaterally. Which of
the following is the most likely natural progression of this condition?

• (A) a long history of remissions and exacerbations (sclerosis lateral aminotrofica)


• (B) sensory loss in the distribution of peripheral nerves
• (C) focal seizures
• (D) a progressively downhill course
• (E) cogwheel rigidity

• - This man has amyotrophic lateral sclerosis (ALS). The disease causes neuronal loss in the anterior horns of the spinal cord and
motor nuclei of the lower brain stem. The disease is one of constant progression, rather than remissions and exacerbations, and
death usually occurs within 5 years. There is no sensory loss and no seizure diathesis, because only the motor system is
involved. There can be signs ofhyperreflexia and spasticity, depending on the balance of upper and lower motor neuron
damage, but not cogwheel rigidity.

• -A 45-year-old man presents with weakness and fasciculations in his arms and legs. His cranial nerves are normal, but there is
weakness of his left handgrip and right leg quadriceps with loss of muscle bulk. There are obvious fasciculations over the left
forearm and right thigh. Tone is increased in the arms and legs and the reflexes are brisk. Which of the following is the most
likely diagnosis?

• (A) amyotrophic lateral sclerosis (ALS)


• (B) myotonic muscular dystrophy
• (C) amyotonia congenita
• (D) tabes dorsalis
• (E) migraine

• - The most common initial symptom of ALS is weakness and wasting of the extremities. The fasciculations can be a very
prominent part of the disease. This is rare in other neurologic disorders.
ESLEROSIS LATERAL AMINOTROFICA
• Que es la esclerosis lateral aminotrófica?
1) Se refiere a una condición neurología de INICIO EN LA VIDA ADULTA
caracterizada por una DEGENERACIÓN PROGRESIVA DE LAS NEURONAS
MOTORAS SUPERIORES E INFERIORES.
2) La degeneración de las neuronas de la corteza motora se manifiesta como
hiperreflexia, signos positivos de Hoffman, Babinski y clonus.
3) El termino esclerosis lateral se refiere a la DUREZA CUANDO SE PALPAN LAS
COLUMNAS LATERALES DE LA MEDULA ESPINAL en las autopsias de pacientes
con esta enfermedad.
4) El termino aminotrófica se refiere a la ATROFIA MUSCULAR, DEBILIDAD Y
FASCICULACION debidas a la degeneración de las neuronas motoras inferiores.

• Que hallazgos muestra la electromiografía al respecto de la esclerosis lateral


aminotrófica?
• R = Demuestra DENERVACIÓN de POR LO MENOS 3 EXTREMIDADES.

• Cual es el manejo de la esclerosis lateral aminotrófica?


• R = RILUZOLE QUE ES UN ANTAGONISTA DEL GLUTAMATO Y LA VITAMINA E.
Deben ser tratados como si tuvieran lesión de la columna cervical hasta que se
demuestre lo contrario.
CHARCOT-MARIE-TOOTH
• Cuales son las características de la mononeuropatia
hereditaria de Charcot-Marie-Tooth?
1) Carácter AD en CROMOSOMA 17.
2) Clínicamente con DEFORMIDADES DEL PIE,
TRASTORNOS DE LA MARCHA.
3) El electrodiagnóstico muestra DISMINUCIÓN DE LA
CONDUCCIÓN MOTORA Y SENSITIVA (neuropatía
motora y sensitiva hereditaria tipo I) NMSH tipo I.
4) NMSH tipo II disminuye solo un poco.
5) Histológicamente hay PERDIDA AXONAL.
TRAUMATISMO
• Que te indica el signo de mapache?
• R = Equimosis en parpados debido a fractura de base del craneo por TCE

• Que maniobra resulta útil en el TCE que causa salida de LCR por nariz u oídos (contiene glucosa)?
• R = CONSERVADOR, con ELEVACIÓN DE LA CABEZA, RESTRICCIÓN DE LÍQUIDOS, administración de
acetazolamida EN CASO DE PERSISTENCIA SE REALIZA DRENAJE LUMBAR SUBARACNOIDEO

• Cual es el cuadro clínico de traumatismo en medula espinal?


1) La transección medular completa resulta en PARÁLISIS FLÁCIDA INMEDIATA Y PERDIDA DE LA SENSIBILIDAD
DEBAJO DE LA LESIÓN.
2) Se pierde retención urinaria y fecal.
3) Al final hay paraplejia en las piernas en flexión o extensión.
4) En las LESIONES MENORES los pacientes que dan con DEBILIDAD LEVE DE LAS EXTREMIDADES, TRASTORNOS
SENSITIVOS DISTALES O AMBOS.

• Que caracteriza al síndrome de BROWN-SEQUARD?


• R = Es una LESIÓN UNILATERAL de la MEDULA ESPINAL que da lugar a un TRASTORNO MOTOR IPSOLATERAL con
deterioro concomitante de la PROPIOSEPCION y PERDIDA CONTRALATERAL de las apreciaciones del DOLOR Y
TEMPERATURA

• Cual es el manejo del traumatismo medular?


• R = INMOBILIDAD. GRANDES DOSIS DE ESTEROIDES metilprednisolona 300 mg en bolo seguidos por 5.4
mg/kg/hr x 24 hrs.
TRAUMATISMO
• Cual es la principal etiología de un HEMATOMA SUBDURAL y sus principales características
clínicas?
• R = A subdural hematoma is almost always of VENOUS ORIGIN and secondary to a minor or
severe INJURY TO THE HEAD, but may occur in blood dyscrasias or cachexia in the absence of
trauma. ACUTE subdural hematomas commonly present with a FLUCTUATING LEVEL OF
CONSCIOUSNESS and significant cerebral damage. CHRONIC subdurals may also present with
SEIZURES OR PAPILLEDEMA

• -A 94-year-old man presents with progressive headaches, light-headedness, drowsiness, and


unsteady gait over 6 weeks. On examination, his blood pressure is 160/90 mm Hg, pulse
70/min, lungs clear, and he has no focal weakness. His gait is unsteady but sensation in the
feet is normal. A CT scan reveals a hyperintense clot over the left cerebral cortex. Which of
the following is the most likely cause for this clot?

• (A) is venous in origin


• (B) is arterial in origin
• (C) is from injury to the middle meningeal artery
• (D) is from a subarachnoid hemorrhage
• (E) is from injury to the middle cerebral artery
ESCALA DE GLASGOW MODIFICADA (NIÑOS)

PRUEBA RESPUESTA PUNTUACIÓN


Espontanea 4
APERTURA DE A órdenes 3
OJOS Al estimulo doloroso 2
Nula 1
 Palabras apropiadas y sonrisas,
LLANTO COMO fija la mirada y sigue los objetos. 5
 Tiene llanto, pero consolable. 4
RESPUESTA  Persistente e irritable 3
VERBAL  Agitado. 2
 Sin respuesta. 1
Obedece ordenes 6
Localiza el dolor 5
RESPUESTA Retirada ante el dolor 4
MOTORA Flexión inapropiada 3
Extensión 2
Nula 1
Si EGC: 15-----------------------Consciente
Si EGC: 13-14-------------------Estupor ligero
Si EGC: 11-12-------------------Estupor moderado
Si EGC: 9-10--------------------Estupor profundo
Si EGC: 7-8---------------------Coma superficial
Si EGC: 5-6---------------------Coma moderado
Si EGC: 3-4---------------------Coma profundo
REACTIVIDAD PUPILAR.
Mioticas  Diámetro < 2mm
Según el
Medias  Diámetro 2-5mm
tamaño
Midriaticas  Diámetro > 5mm
Isocoricas  Iguales
Según relación
Anisocoricas  Desiguales
entre ellas
Discoricas  Forma irregular
Según Reactivas Contracción al foco
respuesta luminoso
a la luz Inmóviles al foco
Arreactivas
luminoso
SÍNDROME DEL TÚNEL CARPIANO
• A que se asocia el síndrome del túnel carpiano?
• R = HIPOTIROIDISMO, EMBARAZO, diabetes, AR o ACROMEGALIA.

• Cuales son las manifestaciones clínicas del síndrome del túnel carpiano?
1) HORMIGUEO, dolor, ENTUMECIMIENTO EN LA MANO, sobre todo al
comienzo de la mañana y en ocasiones con DEBILIDAD DE LA
MUSCULATURA TENAR con atrofia del abductor corto del pulgar.
2) Puede observarse el SIGNO DEL TINEL que se provoca con un golpesito en
el túnel carpiano REPRODUCIENDO DOLOR Y HORMIGUEO.

• Como dx el sx del túnel carpiano?


• R = CLÍNICA + ELECTROMIOGRAFÍA.

• Cual es el manejo del sx del túnel carpiano?


• R = Descompresión quirúrgica.
COMPRESIÓN DEL NERVIO CUBITAL

• Como se manifiesta clínicamente la compresión del nervio cubital?


1) ADORMECIMIENTO EN EL QUINTO DEDO y en el lado MEDIAL de la mano.
2) El dolor y las parestesias pueden iniciar en el codo e irradiarse a la mano.
3) LAS PARESTESIAS SE EXACERBAN CON LA FLEXIÓN DEL CODO.

• Como diagnosticas la compresión del nervio cubital?


• R = Electrofisiología.

• Cual es el manejo de la compresión del nervio cubital?


• R = Los ESTEROIDES Y LOS AINES. Se recomienda cirugía.
NERVIO CUBITAL
• Cual es la sintomatología en caso de daño al nervio cubital?
• R = Injury to the ulnar nerve results in impaired adduction and abduction of the
fingers. The nerve is commonly injured in ELBOW DISLOCATIONS and fractures. The
fibers arise from the eighth cervical and the first thoracic segments. The ulnar is a
mixed nerve with sensory supply to the medial hand.

• - A 40-year-old man is injured in a car accident and fractures his left elbow. He now
complains of numbness of his fourth and fifth fingers, and weakness in his hand
grip. Neurologic findings confirm weakness of handgrip with weakness of finger
abduction and adduction, and decreased sensation over the fifth finger and lateral
aspect of fourth finger. Which of the following is the most likely diagnosis?

• (A) ulnar nerve injury


• (B) radial nerve injury
• (C) median nerve injury
• (D) carpal tunnel syndrome
• (E) axillary nerve injury
COMPRESIÓN DEL NERVIO RADIAL

• Como se manifiesta la compresión del nervio


radial?
1) “Parálisis de la noche del sábado” por dejar
el brazo colgando después de una
borrachera.
2) MUÑECA FLÁCIDA y debilidad a la extensión
de los dedos.
CIATICO Y TARSO
• Cual es la etiología mas común de la parálisis del
ciático?
• R = INYECCIONES INTRAMUSCULARES profundas

• Que sintomatología da el síndrome del túnel del


tarso?
• R = NERVIO TIBIAL, rama del ciático, cuando se
comprime da origen a DOLOR, PARESTESIAS Y
ENTUMECIMIENTO SOBRE BASE DEL PIE
PREDOMINANDO DURANTE LA NOCHE sin afectar
talón.
MERALGIA PARESTESICA
• Que causa y como se manifiesta clínicamente la meralgia
parestesica?
1) Se debe al atrapamiento del NERVIO CUTÁNEO LATERAL DEL
MUSLO POR DEBAJO DEL LIGAMENTO INGUINAL.
2) Se produce DOLOR URENTE, PARESTESIAS Y
ENTUMECIMIENTO DE LA CARA ANTEROLATERAL DEL
MUSLO.
3) Se presenta en pacientes con EXCESO DE PESO por lo que el
TRATAMIENTO ES BAJAR DE PESO.
4) Nervio FEMOROCUTANEO EXTERNO, se origina en RAÍCES
L2 Y L3 EL CUAL SE PRESIONA EN CASO DE EMBARAZO,
DIABETES Y OBSESOS.
LESIONES DE LOS PARES CRANEALES
• Que datos clínicos se encuentran en el daño al III (OCULOMOTOR) PAR CRANEAL?
1) Frecuentemente causa ANISOCORIA, en la que la pupila mas dilatada denota al LADO AFECTADO.
2) El fenómeno de MARCUS GUNN consiste en que con el ESTIMULO LUMINOSO NO OBTENEMOS
RESPUESTA EN EL LADO AFECTADO después de presentar contracción por efecto de reflejo consensual
al estimular contralateralmente
3) Suele deberse a CAUSAS HIPOXICAS
4) En pacientes con compresión cerebral se mostrara midriasis ipsolateral al sitio afectado como
resultado de compresión del III par craneal en cualquier parte de su recorrido debido a que las fibras
que inervan el esfínter de la pupila discurren en porción periférica del nervio.
5) PTOSIS PALPEBRAL CONTRALATERAL a la lesión.
6) La lesión en este par craneal afecta los MÚSCULOS ELEVADOR DEL PARPADO, recto medial, recto
superior, recto inferior y oblicuo inferior (TODOS MENOS EL RECTO EXTERNO Y EL OBLICUO
SUPERIOR), lo que provoca que los movimientos de ADUCCIÓN, ELEVACIÓN Y DEPRESIÓN DEL OJO
IPSOLATERAL ESTÉN AFECTADOS.
7) Lo anterior provoca que cuando el paciente este de frente el ojo tiende a DESVIARSE HACIA ABAJO Y
AFUERA.

• Que datos clínicos se encuentran en el daño al IV (PATÉTICO O TROCLEAR) PAR CRANEAL?


1) Presentan debilidad del MUSCULO OBLICUO SUPERIOR
2) DIPLOPIA CON ADUCCION Y HACIA ABAJO

• Que datos clínicos se encuentran en el daño al VI (OCULOMOTOR EXTERNO) PAR CRANEAL?


1) Afecta al MUSCULO RECTO LATERAL
2) El ojo afectado se dirige A LA LINEA MEDIA CON INCAPACIDAD DE ABDUCIR EL OJO.
LESIONES DE LOS PARES CRANEALES
• Que datos clínicos se encuentran en el daño al V (TRIGÉMINO) PAR CRANEAL?
1) La parálisis de la primera división (OFTÁLMICA) involucra PERDIDA DE LA SENSIBILIDAD EN LA FRENTE
junto con parálisis del III par y IV.
2) La parálisis de la segunda división (MAXILAR) resulta en la PERDIDA DE SENSIBILIDAD DE LA MEJILLA y se
debe a lesiones del seno cavernoso.
3) La parálisis de la tercera división (MANDIBULAR) resulta en DÉFICIT EN LA MASTICACIÓN

• Que cuadro clínico resulta del síndrome ocasionado por la afección del ganglio de Gasser?
• R = Síndrome caracterizado por DOLOR FACIAL, CEFALEA, PERDIDA DE LA SENSIBILIDAD SENSORIAL, SORDERA Y
PARÁLISIS DEL VI Y VII PARES. EL DOLOR AUMENTA AL MOVER LA MANDÍBULA.

• Cual es el cuadro clínico resultante de la neuralgia del glosofaríngeo?


• R = Es un trastorno unilateral que afecta principalmente a V3 presentándose en forma de ESPASMOS
DOLOROSOS que pueden durar de segundos a minutos y pueden estar asociados a estímulos cutáneos y
auditivos.

• Cual es la etiología de la parálisis de Bell?


• R = Se asocia a HERPES VIRUS

• Que características clínicas tiene la parálisis de Bell?


1) PARÁLISIS DEL VII PAR CON INICIO SÚBITO DE DEBILIDAD FACIAL y pérdida del reflejo acústico.
2) Hay PRÓDROMO VIRAL y adormecimiento o dolor en el oído, lengua o cara.
3) La CUERDA DEL TÍMPANO SE VE ERITEMATOSA y HAY REDUCCIÓN EN LA LAGRIMACIÓN IPSOLATERAL o
salivación.
4) El paciente no puede cerrar los ojos por lo que se debe evitar el daño a la cornea
5) Se diagnostica con ELECTROMIOGRAFÍA denotándose una DISMINUCIÓN EN LA VELOCIDAD DE CONDUCCIÓN
LESIONES DE LOS PARES CRANEALES
• Cual es el manejo de la parálisis de Bell?
• R = VALACICLOVIR o famciclovir y PREDNISONA

• Cual es la característica clínica de la afectación del VIII (VESTIBULOCOCLEAR) par craneal?


1) Porción coclear: El principal síntoma asociado a la lesión o atrofia de esta porción es la HIPOACUSIA
SENSORINEURAL. Se realiza el diagnostico por AUDIOMETRÍA.
2) Porción vestibular: VÉRTIGO POSTURAL PAROXÍSTICO BENIGNO.

• Que características clínicas se observan por daño al XI (ESPINAL ACCESORIO) par craneal?
• R = Dolor significativo acompañado de una DISMINUCIÓN DE LA MOVILIDAD DEL HOMBRO, siendo la
lesión iatrogena la mas común (p.e la BIOPSIA DE LOS GANGLIOS LINFÁTICOS).

• Cuales son los datos clínicos que involucra una lesión en los pares craneales IX Y X (GLOSOFARÍNGEO Y
VAGO)?
1) DISFAGIA, parálisis de la cuerda vocal ipsolateral, así como DEBILIDAD DEL PALADAR Y LA FARINGE.
2) La ELEVACIÓN DEL PALADAR ES ASIMÉTRICA y la ÚVULA SE DESVÍA AL LADO CONTRARIO DE LA
LESIÓN.
3) Se pierde la capacidad sensorial de UN TERCIO POSTERIOR DE LA LENGUA, con disminución en la
sensibilidad en la faringe posterior y laringe.
4) La LESIÓN más común del VAGO es la que involucra al NERVIO LARÍNGEO RECURRENTE, lo que
provoca PARÁLISIS DE LA CUERDA VOCAL IPSOLATERAL Y SUBSECUENTE DISFONÍA.
5) La neuralgia del glosofaríngeo tiene CARACTERÍSTICAS SIMILARES A LA NEURALGIA DEL TRIGÉMINO,
pero es MAS EVIDENTE EN LA LENGUA, LAS AMÍGDALAS, OÍDO Y ANGULO DE LA MANDIBULA
LESIONES DE LOS PARES CRANEALES
• Que características clínicas tiene el daño al XII (HIPOGLOSO) par craneal?
• R = DEBILIDAD en la lengua, FASCICULACIONES y DESVIACIÓN DE LA LENGUA hacia el lado de la lesión.

• Que medicamento es de elección en neuralgia del trigémino y del glosofaríngeo?


• R = CARBAMACEPINA, pero también puede incluir baclofen y fenitoina.

• - A 52-year-old man complains of episodes of severe unilateral, stabbing facial pain that is intermittent for
several hours, and then disappears for several days. The pain is describes as “electric shock-like” and only lasts
a few seconds. Physical examination of the face and mouth is entirely normal. Which of the following
treatments is most effective for this condition?

• (A) morphine
• (B) indomethacin
• (C) cimetidine
• (D) carbamazepine
• (E) lidocaine (Xylocaine) gel

• - This patient has trigeminal neuralgia. Carbamazepine (an anticonvulsant drug) is given in doses varying from
600 to 1200 mg/day. Phenytoin has also been used. The two drugs can also be used in combination. Operative
procedures include alcohol injection of the nerve or ganglion, partial section of the nerve in the middle or
posterior fossa, decompression of the root, and medullary tractotomy.Radiofrequency surgery can destroy pain
fibers but spare motor fibers.
VERTIGO
• Cual es la diferencia entre vértigo de origen central y de origen periférico?
• R = In CENTRAL VERTIGO, the vertigo can be MILD AND CHRONIC. In PERIPHERAL disease, THE SYMPTOMS ARE
GENERALLY MORE SEVERE, BUT FINITE (although often recurrent).

• - A 63-year-old woman develops symptoms of nausea, vomiting, and dizziness, which she describes as a to-and-
fro movement of the room like as if she is on a boat. Which of the following findings suggests the vertigo is
central in origin?

• (A) deafness
• (B) symptoms are more protracted but less severe
• (C) unidirectional nystagmus
• (D) visual fixation inhibits vertigo and nystagmus
• (E) spinning sensation is toward the fast phase of nystagmus

• Cuales son las características del vértigo de ORIGEN PERIFÉRICO?


• R = TINNITUS AND DEAFNESS may be found in PERIPHERAL VERTIGO, but NOT CENTRAL. The nystagmus is
usually unidirectional and is never vertical. VISUAL FIXATION INHIBITS VERTIGO AND
NYSTAGMUS during testing in peripheral vertigo
• - A 47-year-old man presents to the emergency room with symptoms of dizziness and difficulty walking. He
describes his dizziness as a spinning sensation of the room with associated nausea and vomiting. Which of the
following findings suggests the vertigo is peripheral in origin?

• (A) optic neuritis


• (B) tinnitus
• (C) bidirectional nystagmus
• (D) vertical nystagmus
• (E) visual fixation does not affect vertigo or nystagmus
VERTIGO
• Cual es la sintomatología del vértigo benigno posicional paroxístico y que maniobra utilizas
para demostrarlo?
• R = Benign paroxysmal positional vertigo (BPPV), which is characterized by sudden-onset
brief episodes of vertigo LASTING LESS THAN A MINUTE. The symptoms are USUALLY
BROUGHT ON BY HEAD MOVEMENT. The cause is commonly attributed to CALCIUM DEBRIS
IN THE SEMICIRCULAR CANALS, KNOWN AS CANALITHIASIS. The debris is loose otoconia
(calcium carbonate) within the utricular sac. Although BPPV can occur after head trauma,
there is usually no obvious precipitating factor. IT GENERALLY ABATES SPONTANEOUSLY AND
CAN BE TREATED WITH VESTIBULAR REHABILITATION. A DIX-HALLPIKE MANEUVER
REPRODUCES HIS SYMPTOMS

• - A 48-year-old man complains of recurrent episodes of sudden-onset dizziness. He notices


an abrupt onset of a spinning sensation when sitting up or lying down in bed. The symptoms
last for 30 seconds and then resolve completely. He has no hearing change or other
neurologic symptoms, and his physical examination is completely normal. A Dix-Hallpike
maneuver reproduces his symptoms. Which of the following is the most likely mechanism
for his vertigo symptoms?

• (A) basilar migraine


• (B) brain stem ischemic events
• (C) benign cerebellar tumors
• (D) calcium debris (calcium carbonate crystals) in the semicircular canals
• (E) Meniere’s disease
EVC
• Cual es la duración de un ataque isquémico transitorio?
• R = Dura < 24 HRS y el DÉFICIT CLÍNICO SE RESUELVE EN MENOS DE 1-2 HRS. Se
debe utilizar ASA COMO PROFILAXIS.

• Cuanto debe medir un infarto lacunar y como se debe ver en la TAC?


• R = Mide <5 MM de diámetro, en la TAC se observan como LESIONES PEQUEÑAS EN
SACABOCADO HIPODENSAS, con RESOLUCIÓN parcial o completa de 4-6 SEMANAS

• Que sintomatología da el EVC en arteria cerebral anterior?


• R = DEBILIDAD y perdida sensitiva en la pierna contralateral y debilidad leve en
brazo.

• Que cambios produce el infarto cerebral anterior?


• R = CAMBIOS CONDUCTUALES y alteración de la MEMORIA.

• Que sintomatología da la obstrucción de la arteria cerebral media?


• R = AFASIA, HEMIPLEGIA CONTRALATERAL, perdida sensitiva, HEMIANOPSIA
HOMÓNIMA, desviación ocular al lado de la lesión.
EVC
• Que sintomatología causa la obstrucción de la ARTERIA CEREBRAL ½ en su PORCIÓN POSTERIOR?
• R = Causa AFASIA DE WERNIKE y HEMIANOPSIA HOMONIMA

• - A 74-year-old woman develops acute neurologic symptoms and presents to the emergency room. An urgent
magnetic resonance imaging (MRI) scan demonstrates acute occlusion in the right posterior cerebral artery. Which of
the following clinical symptoms is she most likely to have?

• (A) homonymous hemianopia


• (B) total blindness
• (C) expressive aphasia
• (D) ataxia and dysarthria
• (E) a right-sided hemiplegia

• -Occlusion of the right posterior cerebral artery is most likely to cause homonymous hemianopia. This artery conveys
blood to the inferior and medial portion of the posterior temporal and occipital lobes and to the optic thalamus.

• - A 63-year-old man developed a transient episode of vertigo, slurred speech, diplopia, and paresthesias. He is
symptom-free now, and clinical examination is entirely normal. His pastmedical history is significant for hypertension
and dyslipidemia. Which of the following is the most likely cause for symptoms?

• (A) posterior circulation transient ischemic attack (TIA)


• (B) anterior communicating artery aneurysm
• (C) hypertensive encephalopathy
• (D) pseudobulbar palsy
• (E) occlusion of the middle cerebral artery

• -Posterior circulation TIA is suggested by the transient episodes. The basilar artery is formed by the two vertebral
arteries and supplies the pons, the midbrain, and the cerebellum. With vertebrobasilar TIAs, tinnitus, vertigo, diplopia,
ataxia, hemiparesis, and bilateral visual impairment are common findings.
EVC
• Cual es el cuadro clínico de la obstrucción de la arteria basilar?
• R = Alteración del estado de alerta, parálisis de los nervios craneales y caídas.

• Como manejas la hipertensión maligna?


• R = Esta entidad se da con una TAD >130, esta se debe REDUCIR A 1/3 pero NUNCA a un VALOR MENOR DE 95
MM HG. NO se debe usar NEFEDIPINO sublingual. En el caso de HIPERTENSIÓN INTRACRANEAL NO SE DEBE
USAR ESTEROIDES.

• Como se procede ante un EVC < 3 hrs?


• R = TROMBOLISIS IV con activador del plasminogeno no tisular. En los que esta CONTRAINDICADA ESTA
TERAPIA son aquellos con:
1) TAS > 185 mm Hg o TAD > 110
2) Sintomatología leve o que mejore de forma rápida
3) CONVULSIONES al inicio de TCE en los 3 MESES PREVIOS
4) CIRUGÍA MAYOR 2 semanas previas
5) Glucemia < 50 o > 400
6) PLAQUETAS < 100,000
7) MUJERES EMBARAZADAS o en lactancia
8) Ingesta actual de anticoagulantes orales con TP > 15

• Cuales son los datos clínicos de una hemorragia intracraneal?


• R = Además de depender de la localización del hematoma suele observarse CEFALEA, CRISIS CONVULSIVAS,
NAUSEA Y VOMITO.
EVC
• Cual es el cuadro clínico de la HEMORRAGIA SUBARACNOIDEA?
1) Se debe principalmente a la ROTURA DE UN ANEURISMA SECUNDARIA A UN TRAUMATISMO en pacientes con
ANTECEDENTE DE HAS
2) CEFALEA SUBITA, intensa y pulsátil espontanea o ASOCIADA A ALGÚN
ESFUERZO FISICO.
3) El px lo refiere como EL DOLOR MAS INTENSO QUE HA TENIDO EN SU VIDA
4) SE PUEDE ACOMPAÑAR DE NAUSEA, VOMITO, FONOFOBIA Y FOTOFOBIA.
5) Si la evolución es rápida el px puede evolucionar RAPIDAMENTE A ESTADO DE COMA.
6) Este tipo de hemorragia puede presentarse como MUERTE SUBITA.
7) Para su diagnostico se utiliza la TAC, si hay DUDA DX SE REALIZA PUNCIÓN LUMBAR PARA BILIRRUBINA Y CRENOCITOS QUE
CONFIRMAN EL DIAGNOSTICO
8) LA HIPERGLUCEMIA Y EL AUMENTO DE LA TEMPERATURA SE ASOCIAN A MAL PRONÓSTICO.

• Cual es el manejo de la HEMORRAGIA SUBARACNOIDEA?


1) REPOSO ABSOLUTO
2) Monitorización
3) Vendaje de miembros inferiores
4) No estimulación
5) ANALGÉSICOS IV
6) NIMODIPINA ORAL para vasoespasmo cerebral sintomático
7) HIPERVOLEMIA PROFILÁCTICA para prevenir vasoespasmo cerebral
• Cuales son las características de la HEMORRAGIA SUBDURAL?
1) Son hematomas localizados entre la DURAMADRE Y LA ARACNOIDES
2) TCE CON GLASGOW < 8
3) AGUDO < 24 HR, SUBAGUDO >24 HRS Y < 2 SEMANAS Y CRÓNICO > 2 SEMANAS
4) Los síntomas son FOCALIZACIÓN E HIPERTENSIÓN INTRACRANEANA; en las FORMAS SUBAGUDA Y CRÓNICA PREDOMINAN LA
CEFALEA Y DEBILIDAD.
5) Aquellos hematomas que DESPLACEN EL SEPTUM PELUCIDUM > 5 MM de la línea media DEBEN SER EVACUADOS.
EVC
• Cuales son las características de la HEMORRAGIA EPIDURAL?
1) DISMINUCIÓN DEL ESTADO DE ALERTA seguido de
2) INTERVALO DE LUCIDEZ y posteriormente
3) DETERIORO NEUROLÓGICO PROGRESIVO

• Cuales son las características de la trombosis venosa cerebral?


1) 90% presenta como síntoma principal CEFALEA de inicio gradual
2) Debe sospecharse ante un paciente con TAC Y PUNCIÓN LUMBAR NORMAL
3) 40% convulsiones
4) En la TROMBOSIS DEL SENO SAGITAL puede haber DELIRIUM Y AMNESIA.
5) A la exploración física con papiledema y en trombosis venosa focalización
6) El ESTÁNDAR DE ORO PARA EL DIAGNOSTICO ES UNA ANGIOGRAFÍA EN FASE VENOSA ASÍ
COMO RM CON VENOGRAFÍA
7) El tratamiento de elección es la ANTI COAGULACIÓN formal por lo menos SEIS MESES
debido al riesgo de recurrencia así como de tromboembolia pulmonar.
TUMORES SNC
• Cuales son las manifestaciones clínicas de una TUMORACIÓN EN EL LÓBULO FRONTAL?
• R = Declinación progresiva del intelecto y CAMBIOS EN LA PERSONALIDAD.
• Cuales son las manifestaciones clínicas que te da una TUMORACIÓN EN EL LÓBULO
TEMPORAL?
1) Si afectan el TERCIO ANTERIOR se presentara EPILEPSIA Y TRASTORNOS DE LA CONDUCTA
como agresividad e impulsividad
2) SI afecta los 2/3 POSTERIORES se pueden manifestar con HEMIANOPSIA LATERAL
HOMÓNIMA
3) Convulsiones con alucinaciones olfativas o gustativas. Alteraciones en el campo visual.

• Cuales son las manifestaciones clínicas que da una TUMORACIÓN EN EL LÓBULO PARIETAL?
• R = TRASTORNOS CONTRA LATERALES DE LA SENSIBILIDAD. ESTEREOGNOSIA (no
reconocen objetos colocados en la mano), SINDROME DE GERTSMAN
“combinación de alexia, agrafia, acalculia y agnosia digital”.

• Que manifestaciones clínicas da una TUMORACIÓN EN EL LÓBULO OCCIPITAL?


• R = HEMIANOPSIA HOMÓNIMA CRUZADA o defecto del campo visual,
PROSOPAGNOSIA ( no reconoce familiares)
• Que manifestaciones clínicas da la lesión en el TALLO CEREBRAL Y CEREBELOSAS?
• R = PARÁLISIS DE LOS PARES CRANEALES, ATAXIA, incoordinación, nistagmo.
TUMORES DEL SNC
• Cual es el tipo de tumor mas frecuente en la medula espinal?
• R = Ependimoma

• Que es la enfermedad de Von Recklinghausen?


• R = Es una alteración en el CROMOSOMA 17, llamado NEUROFIBROMATOSIS TIPO I que consiste en maculas
MÚLTIPLES HIPERPIGMENTADAS Y NEUROFIBROMAS

• - A 10-year-old boy has multiple tan-colored patches on his skin, and freckle-like skin changes in his armpit
area. The rest of clinical examination is normal. Which of the following conditions is also found in patients
withthis disorder as they get older?

• (A) bilateral eighth nerve tumors


• (B) irregular small pupils
• (C) multiple cutaneous and subcutaneous tumors
• (D) cataracts
• (E) hip involvement

• -The two common forms of neurofibromatosis (NF-1 and NF-2) are genetically distinct. NF-1 is the type with
multiple café au lait spots and is associated with axillary or inguinal freckling, iris hamartomas (Lisch nodules),
peripheral neurofibromas, and bony abnormalities (including kyphoscoliosis). NF-2 is associated with CNS
tumors, particularly bilateral eighth nerve tumors. Skin lesions are spare or absent, and early lens opacities can
occur.

TUMORES DEL SNC
Que caracteriza a la neurofibromatosis tipo II?
• R = Es una alteración en el CROMOSOMA 22. Provoca TUMORES DEL 8vo PAR CRANEAL

• Que es el síndrome de STURGE-WEBER?


• R = ANGIOMA CAPILAR CUTÁNEO congénito que puede ocasionar ANGIOMA DE COROIDES

• Cuales son las características de los astrocitomas (gliomas)?


1) Tumores intracraneales MAS FRECUENTE EN NIÑOS y jóvenes entre 20 y 40ª.
2) EL ASTROCITOMA PILOCITICO es la FORMA MAS BENIGNA de los astrocitomas, PREDOMINA EN NIÑOS y
adultos jóvenes, afectando PRINCIPALMENTE AL CEREBELO.

• Cuales son las características de los OLIGODENDROGLIOMAS?


1) Estos tumores suelen tener zonas de CLACIFICACIONES Y HEMORRAGIAS
2) Tumor de crecimiento lento

• Cuales son las características de los ependimomas?


1) Junto al sistema ventricular REVESTIDO DEL EPÉNDIMO
2) En los niños aparecen principalmente en la primera década de la vida DENTRO DE LOS VENTRÍCULOS.

• Cuales son las características de los germinomas?


• R = Estos tumores tienen muchas características comunes a las neoplasias de células germinales que se
localizan en las gónadas, incluidos los marcadores tumorales

• Cual es el cuadro clínico del meduloblastoma?


1) CRISIS CONVULSIVAS PARCIALES O GENERALIZADAS
2) Síndrome de hipertensión intracraneana caracterizado por la triada de: CEFALEA, VOMITO Y EDEMA DE
PAPILA BILATERAL.
3) Alteraciones cognitivas que se pueden presentar en tumores del lóbulo frontal.
PARKINSON
• Que tratamiento resulta efectivo en el temblor esencial benigno?
• R = PROPANOLOL y si falla primidona o CLONACEPAM

• Que medicamentos causan PARKINSON-LIKE?


• R = Neurolépticos, METOCLOPRAMIDA o reseprina.

• Cual es el neurotransmisor involucrado en Parkinson y cual es la fisiopatologia?


1) DOPAMINA, habiendo un desequilibro con acetilcolina
2) HISTOLOGICAMENTE la enfermedad de Parkinson se caracteriza por DEGENERACIÓN NEURONAL DE LA
SUSTANCIA NIGRA, PARS COMPACTA (PRODUCTORA DE DOPAMINA) y las fibras que la comunica con el
estriado.
3) En el microscopio se encuentran los CUERPOS DE LEWY en la sustancia nigra pars compacta (CUERPOS DE
INCLUSION INTRANEURONAL, EOSINOFILOS CONSTITUIDOS POR ALTAS CONCENTRACIONES DE LA PROTEINA
ALFA-SINUCLEINA).

• Cual es el cuadro clínico del Parkinson?


1) Motoras: TEMBLOR EN REPOSO, RIGIDEZ y bradiscinecia. CON LOS MOVIMIENTOS VOLUNTARIOS SE
ATENUA EL TEMBLOR Y SE EXACERBA CON LAS EMOCIONES. La bradicinecia se manifiesta como lentitud de
movimientos voluntarios. El paciente presenta PERDIDA DE LA EXPRESIÓN FACIAL, menores movimientos de
los labios y lengua al hablar (hipofonia), PERDIDA DE LOS MOVIMIENTOS FINOS DE ESCRITURA (micrografía) y
manipulación de objetos pequeños, DIFICULTAD PARA LEVANTARSE DE UNA SILLA E INICIAR LA MARCHA la
cual se describe como FESTINANTE CON PASOS CORTOS Y RÁPIDOS (APRESURADA).
2) Dermatológica: SEBORREA DE CUERO CABELLUDO y cara con aumento de la sudoración.
3) Psiquiátrica: DEPRESIÓN.
PARKINSON
• Que es la enfermedad de Wilson en el dx diferencial de Parkinson?
• R = Inicio a temprana edad, ANILLOS DE KAYSER-FLEISCHER, hepatitis crónica y aumento de la CONCENTRACIÓN
TISULAR DE COBRE.

• - A 24-year-old man presents with mild jaundice, tremor, and personality changes. Examination reveals
slowness of finger movement, rigidity, and coarse tremor of the outstretched hands. As well there is abnormal
slow movement of the tongue and pharynx resulting in a change in speech and occasional difficulty swallowing.
He is icteric, the liver span is 10 cm, and no spleen is palpable. Which of the following findings is most likely
seen in this condition?

• (A) a reduction of copper excretion in the urine


• (B) an increase of the serum ceruloplasmin content
• (C) no renal involvement
• (D) retention of normal neurologic movements
• (E) a peculiar greenish-brown pigmentation of the cornea

• - In Wilson’s disease, there is usually a reduction of the serum ceruloplasmin content. Signs and symptoms of
injury to the basal ganglia are accompanied by cirrhosis of the liver. Renal involvement is characterized by
persistent aminoaciduria. The most common neurologic finding is tremor. The corneal pigmentation (Kayser-
Fleischer ring) is the most important diagnostic finding on physical examination. If it is absent, any neurologic
findings cannot be ascribed to Wilson’s disease.

• Cual es el estándar de oro en el diagnostico de Parkinson?


1) La NECROPSIA
2) RM donde se observa REDUCCIÓN DE LA SUSTANCIA NEGRA PARS COMPACTA en la porción mesencefalica
superior.
3) PET donde hay MENOR CAPTACIÓN del REACTIVO FLUORO-1-DOPA en el cuerpo estriado.
PARKINSON
• Cual es el tratamiento del Parkinson?
1) LEVODOPA en pacientes MAYORES A 70ª que es AGONISTA DE LA DOPAMINA
utilizada en combinación con CARBIDOPA prevenir la conversión periférica de la
levodopa a dopamina por acción de la ENZIMA DOPADESCARBOXILASA.
2) ANTICOLINERGICOS: en pacientes MENORES A 70ª, CONTRAINDICADOS
EN HPB Y GLAUCOMA DE ÁNGULO CERRADO.
• Que mecanismo de acción tiene la carbidopa?
• R = INHIBE LA ENZIMA QUE CONVIERTE LA LEVODOPA A DOPAMINA

• Cuales son los agonistas de la dopamina utilizados en Parkinson?


• R = BROMOCRIPTINA y pergolida.

• Como actúan los inhibidores de la COMT?


• R = Inhibidores de la Catecolamina-0-Metiltransferasa REDUCEN EL METABOLISMO
DE LA LEVODOPA A 3- METILDOPA y por lo tanto alteran la farmacocinética de la
levodopa en plasma llevando a NIVELES MAS SOSTENIDOS DE DOPAMINA
PLASMÁTICA: TOLCAPONA/ ENTAGAPONA

• Cuales son los fármacos ANTIPSICOTICOS en Parkinson?


• R = RISPERIDONA, clozapina: pudiendo esta causar depleción de MO
PARKINSON
• Que medida quirúrgica se puede utilizar en Parkinson refractario a tx medico?
• R = TALAMOTOMIA o PALIDOTOMIA de 1 solo lado.

• - Which of the following is the most likely finding in a 79-year-old woman with Parkinson’s disease?

• (A) constant fine tremor


• (B) muscle atrophy
• (C) akinesia
• (D) pupillary constriction
• (E) spontaneous remission

• - The characteristic triad in Parkinson’s disease (Tremor, Rigidity, Akinesia) has been expanded to include Postural instability. This
forms the mnemonic TRAP. Autonomic instability is also common. Findings on examination also include masklike facies,
dysarthria, stooped posture, and abnormal gait

• Que fármacos producen Parkinsonismo?


1) FENOTIACINAS, tioxantenos, butiprofenonas, neurolépticos, valproato y fluoxetina.
2) Produciendo SÍNTOMAS PARECIDOS A LOS DEL PARKINSON causando distonias, rigidez, fascies en mascara, bradicinesia y
temblor con menor frecuencia.
3) Los síntomas desaparecen una vez suspendido el fármaco.

• Que entidades conforman al Parkinsonismo plus?


• R = Son síndromes que incluyen SIGNOS Y SÍNTOMAS DE LA ENFERMEDAD DE PARKINSON RELACIONADOS CON
ANORMALIDADES EN PROTEÍNA TAU tales como:
1) Paralisis supranuclear progresiva: Es un trastorno neurodegenerativo que se presenta en la 6ta a 7ma década de la vida y se
caracterizan por rigidez, acinesia o bradicinesia, inestabilidad, caídas, disartria, disfagia y demencia. En la RM se observa
atrofia del coliculo superior en el mesencéfalo. La respuesta al tratamiento con agonistas de dopamina y levodopa es pobre.
2) Degeneracion corticobasal: Los síntomas se desarrollan en la sexta década de la vida incluyendo rigidez focal o asimétrica,
bradicinesia, temblor de acción y reposo y distonía marcada. En la RM se observa perdida cortical localizada en la zona
superior de los lóbulos frontales y parietal contralaterales.
3) Enfermedad difusa de los cuerpos de Lewy: Es un TRASTORNO NEURODEGENERATIVO caracterizado por la presencia de
síntomas parkinsonianos y alteraciones neuropsiquiatricas comúnmente acompañados por DEMENCIA.
ALZHEIMER
• Cual es la manifestación principal de la enfermedad de Alzheimer?
• Déficit de ACETILCOLINESTERASA
• La PERDIDA DE MEMORIA a corto plazo

• Cual es la manifestación encontrada en TAC y Biopsia en Alzheimer?


1) Alzheimer’s disease can be quite diffuse, but there is particular INVOLVEMENT OF THE MEDIAL
TEMPORAL LOBES AND CORTICAL ASSOCIATION AREAS.
2) THE ATROPHY OF THE HIPPOCAMPUS IS PARTICULARLY MARKED.
3) Microscopic examination reveals NEUROFIBRILLARY TANGLES AND AMYLOID PLAQUES.

• - A 69-year-old woman presents to the clinic with memory difficulty. The patient’s daughter is
concerned because she is having difficulty doing her finances, such as paying bills. Memory
impairment testing reveals the poor ability to generate lists of words or copy diagrams
(intersecting pentagons). Her remaining physical examination is normal. Which of the following
anatomic findings is most likely with her condition?

• (A) atrophy of the medial temporal lobes


• (B) atrophy of the entire frontal and temporal lobes
• (C) cranial nerve involvement
• (D) transient episodes of hemiplegia
• (E) atrophy of the caudate
HUNTINGTON
• En que gen se ha identificado la enfermedad de Huntington?

• R = Herencia AD CROMOSOMA 4
• Cual es el cuadro clínico de la enfermedad de Huntington?
• R = Aparece entre los 30-50 años. MOVIMIENTOS ANORMALES y CAMBIOS INTELECTUALES CON IRRITABILIDAD,
ESTADO DEL ANIMO, TERMINANDO EN DEMENCIA.

• - A 38-year-old man presents with involuntary facial grimacing, shrugging of the shoulders, and jerking
movements of the limb. His father was similarly affected. There is also a history of mood changes for the past 3
months. On examination, he appears restless with intermittent slow movements of his hands and face. He has
difficulty performing rapid finger movements, and tone is decreased in the upper and lower limbs. Which of
the following is most likely to represent the progression of his illness?

• (A) a normal life span


• (B) a 50% chance of only male children being similarly affected
• (C) mental deterioration
• (D) eventual development of rigidity
• (E) development of hemiparesis

• - This is a case of Huntington’s chorea. It is an autosomal dominant gene (found on the short arm of
chromosome 4), and male and female children are equally affected. Movement disorder, mental deterioration,
and personality change are the hallmarks of the disease, but can be very subtle initially. The disease starts
typically between ages 35 and 40 (although the variation is wide) and runs its course in about 15 years. The
akinetic rigid variety (Westphal variant) of Huntington’s typically has a childhood onset.
HUNTINGTON
• Que datos de TAC se obtienen en corea de Huntington?
• R = Atrofia cerebral y del núcleo caudado

• Cuando hay corea sin antecedentes se duda del dx, pero que causa
la corea de Syndenham?
• R = Es autolimitada por infección por estreptococo del Gpo A

• Que padecimiento AD con datos clínicos similares a CH se debe a


mutación del gen 12 y con ascendencia japonesa?
• R = Atrofia dentorubro palidolisiana con tx igual a CH

• Cual es el neurotransmisor involucrado en corea de hungtinton y


que tratamiento bloqueador de receptores de ese neurotransmisor
controlan la discinecia y cambio conductual?
• R= GABA y se utiliza HALOPERIDOL
DEMENCIA POR CUERPOS DE LEWY
GILLES DE LA TOURETTE
• Que caracteriza al síndrome de Gilles de la Tourette?
• R = Los TICS MOTORES constituyen el 80% siendo común >21ª
afectando PRINCIPALMENTE LA CARA.

• Cual es el cuadro clínico del síndrome de Gilles de la


Tourette?
1) Tics motores: Olfateo, PARPADEO, fruncir el ceno, ENCOGER
LOS HOMBROS O ASENTIR CON LA CABEZA.
2) Tics fónicos: Murmullos, quejidos, ladridos y aullidos,
coprolalia.

• Cual es el tratamiento de elección del sx de Gilles de la


Tourette?
• R = HALOPERIDOL
ESCLEROSIS MULTIPLE
• Que caracteriza a la esclerosis múltiple?
1) Quizá tenga base autoinmune con desmielinizacion con gliosis reactiva en sustancia blanca encefálica y de la medula espinal
2) Se le ha asociado con Chlamydia pneumoniae, virus herpes 6 y adenovirus
3) El gen HLA-DRB1 en el cromosoma 6p21 es, entre los muchos genes HLA asociados con esclerosis múltiple.
4) El OLIGODENDROCITO encargado de la mielinizacion ES EL BLANCO PRINCIPAL del proceso inflamatorio
principalmente mediado por LT CD 8 CITO TÓXICOS y macrófagos.
5) Patológicamente se distingue por placas de desmielinizacion afectando mas frecuentemente la sustancia blanca
periventricular

• Cual es la clasificación de la esclerosis múltiple?


1) Existen 4 tipos básicos de esclerosis múltiples según su progresión en brote - remisión, primaria progresiva, secundaria
progresiva y progresiva-recurrente.
2) 90% de los pacientes comienza con el TIPO BROTE- REMISIÓN caracterizado por la presentación de uno a dos ataques
por year con remisiones y recuperaciones en los meses posteriores.

• Cual es el cuadro clínico de la esclerosis múltiple?


1) La PRESENTACIÓN COMÚN ES DEBILIDAD, ENTUMECIMIENTO, HORMIGUEO, INESTABILIDAD DE UNA
EXTREMIDAD, paraparesia espástica, NEURITIS RETROBULBAR, trastornos esfinterianos. Los síntomas desaparecen
de días a semanas. En el examen aparecen ATROFIA ÓPTICA, nistagmo, disartria y déficit sensitivo en extremidades.
2) Patrón ataque - recaída - exacerbación, definiéndose ataque como el cuadro agudo de disfunción del SNC que dure mas de
24 hrs en ausencia de fiebre, infecciones o alteraciones metabólicas.
3) FENÓMENO DE UTHOFF, el cual consiste en la aparición o exacerbación de los síntomas tras el ejercicio o un baño con
agua caliente
4) TANTO LAS TAREAS COGNITIVAS COMO LA ACTIVIDAD FÍSICA PROVOCAN CANSANCIO PROFUNDO Y
LOS PACIENTES TARDAN MAS TIEMPO EN RECUPERARSE
5) El CANSANCIO es uno de los síntomas mas comunes y discapacitantes de la enfermedad.
6) La afectación de la vía visual es casi una regla con la presencia de NEURITIS ÓPTICA que se manifiesta como CEGUERA
MONO O BINOCULAR.
7) EL SIGNO DE LHERMITTE (sensación de electricidad que va de la medula hacia las extremidades al flexionar el cuello).
ESCLEROSIS MULTIPLE
• Cual es el estudio de elección para dx de esclerosis múltiple?
1) IRM de ENCÉFALO o MEDULA ESPINAL que demuestra MIELOPATIA FOCAL O MÚLTIPLE
EN LA SUSTANCIA BLANCA. Se aplican los criterios de Barkhof y Tintore.
2) El estudio del LCR permite valorar la inflamación intratecal. SE BUSCAN BANDAS
OLIGOCLONALES DE IgG que no están presentes en suero. Un total de PROTEÍNAS
MENOR A 100 Y LEUCOCITOS MENORES A 50 ES LO COMÚN
3) Los POTENCIALES EVOCADOS VISUALES son los mas SENSIBLES Y ESPECÍFICOS, los cuales
DENOTAN DESMIELINIZACION.

• En que consiste la malformación de Arnold Chari, la cual debe descartarse en esclerosis


múltiple?
• R = Observándose la región del agujero magno UNA PARTE DEL CEREBELO Y PARTE INFERIOR
DEL TALLO ENCEFÁLICO SE DESPLAZAN HACIA EL CONDUCTO CERVICAL produciendo déficit
piramidal y cerebeloso en extremidades.

• Cual es el tratamiento de esclerosis múltiple?


• R = INTERFERON B disminuye exacerbaciones al igual que inmunoglobulina IV. NO HAY
FORMA DE DETENER EL AVANCE, se utiliza PREDNISONA EN RECIDIVAS.

• Cual es el pronostico de la esclerosis múltiple?


• R = Mientras mayor sea el numero y volumen de las lesiones peor será el pronostico.
ESCLEROSIS MULTIPLE
• -A 22-year-old woman presents with acute vision loss and pain in the left eye, but no other symptoms. On examination she
appears well, visual acuity is not possible, and she can only perceive movement and bright light. The direct papillary reflex is
absent but the indirect (consensual) response is normal. The optic disc is edematous. Which of the following symptoms is also
most likely present in patients with this condition?

• (A) limb weakness


• (B) hemiplegia
• (C) cervical myelopathy
• (D) sphincter impairment
• (E) seizures

• - This patient has multiple sclerosis (MS). Weakness or numbness in one or more limbs is the initial manifestation of disease in
about half the patients. Other common initial presentations include optic neuritis (25%) and acute myelitis. Hemiplegia,
seizures, and cervical myelopathy (in older patients) occur occasionally as the initial manifestation. Sphincter impairment
usually occurs later in the disease.

• -A 27-year-old woman presents with acute vision loss and pain in the left eye, but no other symptoms. On examination, she
appears well, visual acuity is not possible, and she can only perceive movement and bright light. The direct pupillary reflex is
absent but the indirect (consensual) response is normal. The optic disc is edematous. Which of the following is the most likely
diagnosis?

• (A) diabetic microvascular disease


• (B) arteriosclerosis
• (C) trauma
• (D) multiple sclerosis
• (E) Creutzfeldt-Jakob disease

• -Visual loss in multiple sclerosis varies from slight blurring to no light perception. Other eye symptoms include diplopia and
pain. The classic syndrome of optic or retrobulbar neuritis occurs commonly at some point in the disease, and it is the
presenting symptom in 25% of cases.
EPILEPSIA
EPILEPSIA
ATAXIA DE FRIEDREICH
• Que es la ataxia de Friedreich?
1) AR, en CROMOSOMA 9 q13-9 q21.
2) Es una MARCHA ATÁXICA, manos torpes, respuesta extensora plantar y disminución de ROTS.
3) The pathologic changes are found in the SPINAL CORD TRACTS. Degeneration is seen in the
POSTERIOR COLUMNS, the lateral corticospinal tract, and the spinocerebellar tracts.
4) PIE CAVO BILATERAL

• -A 19-year-old man has had progressive ataxia of gait and great difficulty in running. In the past year, he has
developed hand clumsiness. Physical examination reveals pes cavus, kyphoscoliosis, and both cerebellar and
sensory changes in the legs. There is a positive family history of Friedreich’s ataxia. Where are the pathologic
changes seen in this condition most likely to be found?

• (A) spinal cord tracts


• (B) basal ganglia
• (C) cerebral cortex
• (D) peripheral autonomic nerves
• (E) peripheral motor nerves

• - This young man has Friedreich’s ataxia, associated with a gene defect on chromosome 9. The pathologic
changes are found in the spinal cord tracts. Degeneration is seen in the posterior columns, the lateral
corticospinal tract, and the spinocerebellar tracts. Ataxia, sensory loss, nystagmus, reflex changes, clubfeet, and
kyphoscoliosis are the characteristic findings. The heart is frequently involved, and cardiac disease is a common
cause of death.
DEFICIT VITAMINA E y B12
• Que produce el déficit de vitamina E a nivel SNC?
• R = Se presenta degeneración medulocerebelosa que afecta
principalmente al cordón posterior de la medula espinal.

• Cuales son los datos clínicos de déficit de vitamina E?


• R = Ataxia de extremidades, disminución sensitiva, disminución de ROTS,
confusión y degeneración pigmentaria de la retina.

• Cual es el tratamiento para el déficit de vitamina E?


• R = Alfa tocoferilo

• Que trastorno en el SNC te ocasiona el déficit de vitamina B12?


• Degeneración combinada subaguda de medula espinal.
• Se acompaña de polineuropatía, cambios mentales o neuropatía óptica.
ENCEFALOPATÍA DE WERNICKE
• Que caracteriza la encefalopatía de Wernicke?
• R = Déficit de “TIAMINA B1” y es COMÚN EN BORRACHOS.

• Cual es el cc de la encefalopatía de Wernike?


• R = CONFUSIÓN, ATAXIA Y NISTAGMO que da lugar a oftalmoplejia (parálisis de la mirada
conjugada, debilidad del musculo recto externo)

• -A 43-year-old man is referred from the emergency department with memory loss and difficulty walking. He
was brought in by his wife who has noticed personality changes, truancy from work, and lack of personal care
over the past 2 years. On examination he appears unkempt, smells of urine, and is uncooperative. He cannot
recall the date or season, and gets angry when asked questions. His answers are often fabricated when checked
with his wife. His gait is wide-based, and there is loss of sensation in his feet up to the shins. His motor strength
and reflexes are normal. His ocular movements are normal and there is no nystagmus. In the past he has had
multiple admissions for alcohol withdrawal. Which of the following is the most likely diagnosis?

• (A) Wernicke’s encephalopathy


• (B) Wernicke-Korsakoff syndrome
• (C) Alzheimer’s dementia
• (D) Charcot-Marie-Tooth disease
• (E) vascular dementia

• -The combination of symptoms is typical of chronic alcohol abuse. The mental symptoms are suggestive of
Wernicke-Korsakoff syndrome. Adistal limb sensory-motor neuropathy is also typical of alcoholism. Confusion,
tremulousness, and disorientation are typical for acute alcohol intoxication. Wernicke’s encephalopathy is a
symptom complex of ophthalmoplegia, ataxia, nystagmus, and acute confusional state
DEGENERACIÓN COMBINADA
SUBAGUDA
• Cual es la etiología de la degeneración combinada
subaguda y cual es el cuadro clínico característico?

1) Es causado por la deficiencia de COBALAMINA (VB12)


2) En un inicio encontramos signos y síntomas de lesión de la
COLUMNA POSTERIOR (PARESTESIAS DE MANOS Y PIES,
INESTABILIDAD DE LA BIPEDESTACIÓN Y MARCHA,
ALTERACIONES DE VIBRACIÓN Y POSICIÓN)
3) Después de un tiempo a las 2 semanas posteriores se
identifica una paraparesia atáxica simétrica con
hiperreflexia o hiporreflexia tendinosa y signo de Babinski.
VITAMINAS HIDROSOLUBLES
VITAMINAS LIPOSOLUBLES
LYME
• Cuales son las manifestaciones neurológicas
de la enfermedad de Lyme?
1) Meningitis,
2) Poliradiculopatia,
3) Mononeuropatia multiple y
4) Neuropatía cutánea.
SARCOIDOSIS
• Cuales son las manifestaciones neurológicas
de la sarcoidosis?
• R = Parálisis de los pares craneales, en especial
el 7 que responde a esteroides
SÍNDROME DE GUILLAIN-BARRE
• Cual es la etiología del síndrome de Guillain-Barre?
1) Suele asociarse a una infección de vías respiratorias o gastrointestinal.
2) Principalmente el CAMPYLOBACTER JEJUNI Y CMV.

• Cuales son los signos y síntomas de la poliradiculoneuropatia progresiva


aguda/sub aguda conocido como síndrome de Guillain- Barre?
• R = Debilidad ascendente de comienzo en piernas.

• Cuales son los estudios para el diagnostico de síndrome de Guillain- Barre?


1) LCR con AUMENTO DE PROTEÍNAS
2) Células normales
3) Electrofisiología con alteraciones
4) Histología que muestra DESMIELINIZACION PRIMARIA

• Cual es el manejo del síndrome de Guillain-Barre?


• R = Administración de INMUNOGLOBULINA IV O PLASMAFERESIS.
MIASTENIA GRAVE
• Cual es el cuadro clínico de la miastenia grave?
1) DIPLOPÍA, debilidad de músculos oculares, debilidad progresiva pudiendo
causar parálisis respiratoria, PTOSIS.
2) El cuadro clínico confirma DEBILIDAD Y FATIGA DE MÚSCULOS EL CUAL
MEJORA CON EL REPOSO.

• Como se realiza el diagnostico de miastenia grave?


1) Respuesta a ANTICOLINESTERASA de acción corta con EDROFONIO con
vida media de 1 minuto en el cual los PACIENTES CON RESPUESTA TIENEN
MEJORÍA NOTORIA DURANTE 5 MINUTOS.
2) NEOSTIGMINA, la respuesta dura 2 hrs, con sulfato de atropina en caso de
efectos muscariniocos adversos.
3) ELECTROFISIOLOGIA que muestra DISMINUCIÓN DE LA
TRANSMISIÓN.
4) LAB con Ac VS RECEPTOR DE ACETILCOLINA

• Cual es el tratamiento de miastenia grave?


• R = NEOSTIGMINA. Timectomia en caso de timoma. PROSCRITOS
AMINOGLUCOSIDOS por que causan bloqueo neuromuscular
MIASTENIA GRAVE
• - A 30-year-old woman complains of double vision, and easy fatigue with
exercise. The fatigue improves with resting, but it is interfering with her work.
Examination reveals ptosis and impaired eye movements with normal pupillary
response. The double vision is brought out by asking her to look at the ceiling, and
after a sustained interval, the eyes slowly drift down. Which of the following is the
most likely diagnosis?

• (A) optic atrophy


• (B) ophthalmic zoster
• (C) paralysis agitans
• (D) Horner syndrome
• (E) myasthenia gravis

• - In myasthenia gravis, weakness of the facial and levator palpebrae muscles


produces a characteristic expressionless face, with drooping of the eyelids.
Weakness of the ocular muscles may cause paralysis or weakness of individual
muscles, paralysis of conjugate gaze, ophthalmoplegia, or a pattern similar to
internuclear ophthalmoplegia. The presence of normal pupillary responses to light
and accommodation with weakness of extraocular muscles, levators, and
orbicularis oculi is almost completely diagnostic of myasthenia
SÍNDROME DE LAMBERT-EATON O
MIASTENIFORME
• Que puede ocasionar el síndrome de lambert-eaton o miasteniforme?
1) En este hay liberación DEFECTUOSA DE ACETILCOLINA.
2) Se presenta con DEBILIDAD DE MÚSCULOS PROXIMALES y se
diferencia de miastenia gravis por que LA FUERZA SE AUMENTA
CON CONTRACCIÓN SOSTENIDO.
• Como diagnosticas el síndrome de lambert-eaton?
• R = Electrofisiología por AUMENTO DE LA RESPUESTA
MUSCULAR A LA ESTIMULACIÓN
• Cual es el manejo del síndrome de lambert-eaton?
• R = PLASMAFERESIS, inmunodepresores (azatropina, prednisona).


Ayuda la toma de LCR en el caso de absceso cerebral?
R = NO, de echo ESTA PROSCRITO por que puede causar herniación. NEUROINFECCION
• Cual es la complicación mas frecuente del SIDA en el SNC?
• R = This is one of the most common neurologic complications of AIDS. Its pathologic substrate is
degeneration of the spinal tracts in the posterior and lateral columns, which have a vacuolated
microscopic appearance. Although the morphologic changes and clinical manifestations are similar
to those associated with vitamin B12 deficiency, the pathogenetic mechanism is probably not
related to dietary deficiencies. Since there is no specific clinical or laboratory test available for the
diagnosis of this syndrome, VACUOLAR MYELOPATHY IN AIDS PATIENTS REMAINS A DIAGNOSIS OF
EXCLUSION. This implies that other HIV-related neurologic complications must be ruled out

• -A 56-year-old man is brought to the emergency department by his wife because of memory loss
and difficulty walking. She has noticed personality changes, truancy from work, and lack of
personal care over the past 1 year. On examination he appears unkempt, smells of urine, and is
uncooperative. He cannot recall the date or season,and gets angry when asked questions. His
answers are often fabricatedwhen checked with his wife. The blood pressure is 150/90 mm Hg,
pulse 100/min, and he is diaphoretic and tremulous. His gait is wide based, and motor strength and
reflexes are normal. His ocular movements are normal but there is nystagmus on lateral gaze. In the
past he has had multiple admissions for alcohol withdrawal. Which of the following is the most
appropriate next step in management?

• (A) prophylactic phenytoin administration


• (B) prophylactic diazepam administration
• (C) prophylactic carbamazepine administration
• (D) calcium administration
• (E) steroid administration

• - Prophylactic administration of diazepam in a withdrawing alcoholic can prevent or reduce severe


syndromes such as delirium tremens (DTs). Prophylactic phenytoin, however, is not helpful. Acalm,
quiet environment with close observation and frequent reassurance is very important. Vitamin
administration (especially thiamine) is important, but frequently, severe magnesium depletion
slows improvement.
LCR
BOTULISMO
• Cual es la fisiopatología del botulismo?
• R = La toxina EVITA LA LIBERACIÓN DE ACETILCOLINA en las uniones
neuromusculares y sinapsis autónomas, transmitida por COMIDAS ENVASADAS

• Cual es el cuadro clínico del botulismo?


1) Desarrollo SÚBITO DE DEBILIDAD INTENSA DESPUÉS DE 72 HRS DE LA COMIDA.
2) El típico cuadro es de DIPLOPÍA, VISIÓN BORROSA, midriasis no reactiva,
estreñimiento por íleo paralitico, PTOSIS, debilidad facial, DIFICULTAD
RESPIRATORIA Y DEBILIDAD EN EXTREMIDADES INFERIORES CON SENSIBILIDAD
CONSERVADA.

• Como se dx el botulismo?
• R = SUERO CON TOXINA. Electrofisiológico con estimulación repetida con aumento
de la respuesta muscular de manera progresiva

• Cual es el manejo del botulismo?


1) ANTITOXINA en pacientes NO ALÉRGICOS AL SUERO DE CABALLO.
2) GUANIDINA facilita la liberación de acetilcolina
MIGRANA
• Cual es el cuadro clínico de la migraña?
1) Cefalea intensa, pulsátil, unilateral en la región frontal o temporal.
2) La migraña con aura consiste en un inicio con alteraciones visuales con
luces-centilleos o luces de colores, palidez, vértigo, alteraciones
sensitivas o debilidad unilateral,
3) Afasia transitoria o alteraciones en el lenguaje que aparecen
previamente al establecimiento de la migraña.

• A que se refiere el termino estatus migranoso?


• R = Se refiere a pacientes ya conocidos con migraña con manifestaciones
clínicas por mas de 72 Hrs

• Cual es el manejo de la migraña?


• R = Triptanos y derivados de la ergotamina
MIGRANA
• Cuales son las 2 variantes de migraña a considerar?
1) Los síntomas prodrómicos son el resultado de una disfunción en el territorio de la circulación
cerebral posterior con sintomatología visual bilateral, ataxia, disartria, vértigo, parestesias de las
extremidades e incluso debilidad. Puede haber perdida de la conciencia antes de comenzar la
cefalea.
2) Migraña hemipléjica: Es poco frecuente e incluye hemiplejia, que puede persistir días después de
calmarse la cefalea.

• - An 18-year-old woman has periodic episodes that begin with severely decreased vision, followed
by ataxia, dysarthria, and tinnitus. The symptoms last for 30 minutes and are then followed by a
throbbing occipital headache. Which of the following is the most likely diagnosis?

• (A) vertebral-basilar insufficiency


• (B) chronic basilar artery dissection
• (C) classic migraine
• (D) ophthalmoplegic migraine
• (E) basilar migraine

• Basilar migraine can be very dramatic, and can resemble ischemia in the territory of the basilar
posterior cerebral arteries. The visual symptoms of basilar migraine typically affect the whole of
both visual fields, and can even cause temporary cortical blindness. There can also be an alarming
period of coma or quadriplegia.
CEFALEA TENSIONAL
• Cual es el cuadro clínico de la cefalea tensional?
1) Dolor difuso, en banda, de carácter sordo, que
puede acompañarse de hiperestesia en el cuero
cabelludo y agravarse con el ruido y la luz.
2) Típicamente la distribución del dolor es en
forma de banda, bilateral y se extiende desde la
frente hacia los temporales y nuca.
3) Como profilaxis pueden utilizarse antidepresivos
triciclicos y la amitriptilina es la más eficaz.
CEFALEA EN RACIMOS O ENFERMEDAD DE
HORTON
• Que caracteriza a la cefalea en racimos o
enfermedad de Horton?
1) Intenso dolor unilateral periorbitario
acompañado de inyección conjuntival, lagrimeo,
rinorrea y síndrome de Horner
2) El comienzo del dolor suele ser en la madrugada
o al despertar muy temprano.
3) Para su tratamiento son útiles los fármacos
utilizados en migraña.
4) Nifedipino y berapamil como parte del manejo
profiláctico.
• Cual es la sintomatología en caso de narcolepsia?
NARCOPLEPSIA
• R = The early age of onset and otherwise good health suggest a diagnosis of narcolepsy, which is usually accompanied by other
symptomatology. Hypnagogic hallucinations are almost always visual. They occur most frequently at the onset of sleep, either
during the day or at night. They are generally very vivid. Cataplexy is a brief loss of muscle power without loss of consciousness.
The patient is fully aware of what is going on. The paralysis may be complete or partial.

• -A 31-year-old woman complains of excessive sleepiness during the daytime for years despite adequate nighttime sleep. She
has episodes of intense drowsiness three to four times a day, even when at work or while eating meals. She has sought medical
attention in the past, after falling asleep while driving. She is slender and otherwise healthy and on no medications. Which of
the following treatments is most likely indicated for her condition?

• (A) a device providing continuous positive airway pressure (CPAP) at night


• (B) oral surgery
• (C) tracheostomy
• (D) amphetamines
• (E) benzodiazepines at bedtime

• -This woman does not have risk factors for sleep apnea (older age, snoring, obesity) and likely has narcolepsy. Adrenergic
stimulant drugs such as methylphenidate or amphetamines help the sleepiness, and tricyclic compounds can help the cataplexy.
Strategically planned naps can also be helpful.

• -A 25-year-old man complains of excessive sleepiness during the daytime for years despite adequate nighttime sleep. He has
sought medical attention after falling asleep while driving. He is slender and otherwise healthy and on no medications. Which of
the following symptoms might he also complain about?

• (A) excessive snoring (wife’s report)


• (B) automatic behavior (wife’s report)
• (C) restless sleep (wife’s report)
• (D) paresthesias
• (E) morning headache

• -Automatic behavior with amnesia is a common manifestation of the narcolepsycataplexy syndromes, occurring in 50% of cases.
Automatic behavior can be confused with complex partial seizures. Paresthesias are not part of narcolepsy syndrome. Snoring,
restless sleep, and morning headache suggest sleep apnea.
ENDOCRINOLOGIA

ENARM
SINDROME METABOLICO
• PARA EL DIAGNOSTICO DE ESTE SÍNDROME ES
NECESARIO LA PRESENCIA DE 3 O MAS DE LOS
SIGUIENTES CRITERIOS:

– OBESIDAD ABDOMINAL MUJERES > 88 CMS Y HOMBRE >


102 CMS.
– TRIACILGLICERIDOS >O IGUAL A 150 MG/DL O CON
TRATAMIENTO.
– HDL <40 MG/DL (HOMBRES) Y < 50 MG/DL (MUJERES).
– TENSIÓN ARTERIAL SISTÓLICA >O IGUAL A 130 MMHG Y
DIASTÓLICA >O IGUAL A 85 MMHG.
– GLICEMIA > 110 MG/DL O CON TRATAMIENTO
HIPOGLICEMIANTE.
HIPERPROLACTINEMIA
• Cual es una hormona inhibidora de la prolactina?
• R = DOPAMINA, el aumento de estrógenos la inhibe.

• Cuales son los signos y síntomas de hiperprolactinemia?


1) Hombres: DISFUNCIÓN ERÉCTIL, HIPOGONADISMO HIPOGONADOTROFICO, INFERTILIDAD y
disminución de la libido. En ocasiones GINECOMASTIA SIN GALACTORREA.
2) Mujeres: OLIGOMENORREA, amenorrea, GALACTORREA, si el hipogonadismo no es tratado da osteoporosis.

• Que lab se solicitan en hiperprolactinemia y que padecimientos se encuentran relacionados?


• R = HCG en caso de embarazo, HIPOTIROIDISMO, IR y cirrosis.

• Que estudio realizas en hiperprolactinemia no asociada a hipotiroidismo, embarazo o inducida por fármacos?
• R = IRM buscando micro <2 cm o macroadenoma hipofisiario >3 cm.

• Que medicamentos no deben recibir las mujeres con prolactinomas?


• R = ESTRÓGENOS (<2cm no afectan), TETOSTERONA o EMBARAZO.

• Cual es el tratamiento para hiperprolactinemia?


• R = BROMOCRIPTINA. Antagonistas de la dopamina como CENTENO O CABERGOLINA al acostarse,
Quinagolida a los intolerantes derivados del cornezuelo de centeno
SSIADH
• En que región del hipotálamo se secreta la ADH y que funciones tiene?
1) En el NÚCLEO SUPRA ÓPTICO
2) La hormona antidiurética (ADH), o arginina vasopresina (AVP), es una hormona LIBERADA
PRINCIPALMENTE EN RESPUESTA A CAMBIOS EN LA OSMOLARIDAD SÉRICA O EN EL
VOLUMEN SANGUÍNEO. También conocida como argipresina. Hace que los riñones conserven agua
mediante la concentración de orina y la reducción de su volumen, estimulando la reabsorción de agua.
Recibe su nombre de esta importante función como regulador homeostásico de fluidos.

• Cual es la forma mas común de HIPONATREMIA NORMOVOLEMICA?


• R = SSIADH

• Cual es el cc del SSIADH?


1) Neuromusculares: IRRITABILIDAD, debilidad muscular, letargo, CONFUSIÓN y coma.
2) Gastrointestinales: NAUSEA, VOMITO y anorexia
3) La intensidad de los síntomas se relaciona con el grado de hiponatremia

• Como diagnosticas SSIADH?


1) Por exclusión
2) Prueba de sobrecarga hídrica y hallazgos de niveles plasmáticos inapropiadamente elevados de vasopresina
en relación a la osmolaridad plasmática

• Cual es el manejo del SSIADH?


1) RESTRICCIÓN HÍDRICA 500-1000 ml/24 hrs
2) DEMECLOCICLINA (inhibe la respuesta renal a la vasopresina)
PATOLOGIA DE TIROIDES
• Que características clínicas acompañan a la tirotoxicosis?
• R = Nerviosismo, PALPITACIONES, TAQUICARDIA, fatiga, debilidad, pérdida de peso con buen
apetito, DIARREA, INTOLERANCIA AL CALOR, PIEL CALIENTE, transpiración excesiva, labilidad
emocional, alteraciones menstruales, TEMBLOR FINO EN LAS MANOS, alteraciones oculares
y aumento variable de la glándula. FA.

• Como se manifiesta la CRISIS O TORMENTA TIROIDEA?


• SE MANIFIESTA POR DELIRIO, TAQUICARDIA, VOMITO, DIARREA y FIEBRE.

• Cual es el manejo de la tormenta tiroidea?


• R = Dosis altas de PROPILTIUORACILO, yodo o contrastes yodados, PROPANOLOL
Y DEXAMETASONA

• Cuales son los análisis mas específicos para enfermedad de Graves?


• R = TSH-R AB 2da generación y TSH-R recombinante son > 75%
• Que diferencia hay entre la captación de yodo de las siguientes patologías?
1) Enfermedad de Graves y bocio multinodular toxico: ALTO
2) Tiroiditis sub aguda: BAJO
PATOLOGIA DE TIROIDES
• La enfermedad de GRAVES es la causa mas común de TIROTOXICOSIS, que la caracteriza?
1) Enfermedad de Basedow en Europa.
2) ETIOLOGÍA AUTOINMUNE, aumento de hormonas tiroideas, AUMENTO DE GLÁNDULA, AC VS TSH.
3) Mujeres 8:1. SE RELACIONA con enfermedades autoinmunes como ANEMIA PERNICIOSA, MIASTENIA GRAVE.
4) Asociación con HLA-B8 y HLA DR3, AC VS PEROXIDASA, AC VS TIROGLOBULINA y ANA están presentes

• Que es la enfermedad de Plummer en tirotoxicosis?


• R = Son adenomas tóxicos tiroideos simples

• Que es la enfermedad de Jodbasedow en tirotoxicosis?


• R = Hipertiroidismo inducido por yodo

• Que patologías extraglandulares originan tirotoxicosis?


• R = Aumento en la HCG, EMBARAZO MOLAR, CORIOCARCINOMA y neoplasias testiculares.
Administración de AMIODARONA en arritmias.

• Cuales son los signos y síntomas de tirotoxicosis?


1) Mirada fija, CAÍDA DE PARPADO, taquicardia o FA, TEMBLORES FINOS, PIEL HUMEDA/CALIENTE.
2) GRAVES: Oftalmopatia con quemosis, conjuntivitis y PROPTOSIS. DERMOPATIA “MIXEDEMA PRETIBIAL”
con ACUMULACIÓN DE GLUCOSAMINOGLUCANOS que ocasiona edema con textura rugosa.
PATOLOGIA DE TIROIDES
• Cual es el manejo de la enfermedad de Graves?
• 1. PROPANOLOL: taquicardia, temblores y ansiedad
• 2. Derivados de la tiourea (METIMAZOL O PROPILTIURACILO): Puede causar
AGRANULOSITOSIS
• 3. Metimazol: < riesgo de necrosis hepática, fulminante
• 4. Propiltiuracilo: Es de ELECCIÓN en EMBARAZO O LACTANCIA
• 5. Agente de contraste yodados: ayudan en tirotoxicosis de cualquier etiología los cuales
inhibe la trimonoyodinacion de T4 por lo tanto aumenta T3 y disminuye tirotoxicosis
• 6. YODO RADIACTIVO: destruye tejido tiroideo sobre activo

• Que medidas se usan para complicaciones de enfermedad de Graves:


• 1. Oftalmopatia de Graves: EXOFTALMUS; ESTEROIDES, progresivo-radioterapia
• 2. Cardiacas: taquicardia sinusal; PROPANOLOL
• 3. Insuficiencia cardiaca: DIURÉTICOS Y DIGOXINA

• Que causa dermopatia de Graves?


• R = Engrosamiento de piel por glucosaminoglucanos, se le conoce como MIXEDEMA
PRETIBIAL Y SU TRATAMIENTO SON ESTEROIDES TÓPICOS.
PATOLOGIA TIROIDES
• Que caracteriza en la tirotoxicosis a la PARÁLISIS PERIÓDICA HIPOKALEMICA TIROTOXICA?
• R = PARÁLISIS FLÁCIDA SIMÉTRICA REPENTINA, HIPOKALEMIA E HIPOFOSFATEMIA. El
tratamiento es con PROPANOLOL que revierte la parálisis en 3 hrs. Dextrosa IV o CHBTS lo
agravan.

• Cual es el manejo de los nódulos tiroideos solitarios tóxicos?


• R = PROPANOLOL, YODO RADIACTIVO

• Cual es el manejo del bocio multinodular toxico?


• R = PROPANOLOL, YODO RADIACTIVO

• Cual es el cc de la TIROIDITIS DE HASHIMOTO O TIROIDITIS LINFOCITICA CRÓNICA y a que


enfermedad progresa?
1) Tiroides con AUMENTO DE TAMAÑO SIN DOLOR, de consistencia dura,
ASIMÉTRICO y fijo
2) PROGRESIÓN A HIPOTIROIDISMO.
3) Depresión, fatiga crónica, XEROSTOMÍA, XEROFTALMIA.

• Que anticuerpos son los principales implicados en la tiroiditis de Hashimoto?


• R = AC ANTIMIELOPEROXIDASA y LINFOCITOS TCD 4
PATOLOGIA DE TIROIDES
• Cual es el manejo de tiroiditis de Hashimoto al causar hipotiroidismo, bocio y Hashitoxicosis?
1) Hipotiroidismo: LEVOTIROXINA
2) Bocio: T4 para disminuir tamaño de glándula
3) Hashitoxicosis: Aumenta liberación de T4 por acumulo, PROPANOLOL o yodato sódico

• Cuales son los medicamentos principalmente vinculados con la tiroiditis inducida por fármacos?
• R = AMIODARONA, Interferon alfa, LITIO e interleucina 2.

• Cual es el cuadro clínico de la tiroiditis por fármacos?


• R = Tiroiditis INDOLORA

• Que caracteriza a la TIROIDITIS DE RIEDEL /tiroiditis fibrosa invasiva, tiroiditis leñosa, tiroiditis
lingeosa o tiroiditis invasiva?
1) Provoca HIPOTIROIDISMO e HIPOPARATIROIDISMO.
2) Se presenta en EDAD AVANZADA.
3) GLÁNDULA DURA CON ADHERENCIAS al cuello ocasionando DISNEA, DISFAGIA, DOLOR Y
RONQUERA.
4) Los trastornos incluyen FIBROSIS RETROPERITONEAL.

• Cuales son las características laboratoriales en la producción de T3 o T4 de distinción entre los


padecimientos causantes de hipertiroidismo?
1) TIROIDITIS SUB AGUDA (QUERVAIN)/ HASHIMOTO: >T4 + que T3
2) GRAVES/ BOCIO NODULAR TOXICO: >T3
PATOLOGIA DE TIROIDES
• Cual es la causa de la tiroiditis aguda o supurativa?
• R = Infecciosa

• Cual es el cuadro clínico de la tiroiditis aguda o supurativa?


• R = El paciente cursa con FIEBRE, disfagia, disfonía, DOLOR EN LA
REGIÓN ANTERIOR DEL CUELLO, eritema y GLANDULA TIROIDEA SENSIBLE

• Que muestra la biopsia en la tiroiditis aguda o supurativa?


• R = INFILTRADO LEUCOCITARIO

• Cual es el manejo de tiroiditis supurativa?


• R = Antibióticos + drenaje quirúrgico

• Cual es la etiología de la tiroiditis subaguda posparto?


• R = Es causada por la INFLAMACIÓN LINFOCITICA de la tiroides en los
primeros 3 A 6M POSTERIORES AL PARTO.
PATOLOGIA DE TIROIDES
• Cual es el cc de la tiroiditis subaguda posparto?
• R = EL BOCIO es INDOLORO

• Como se encuentra la VSG en la tiroiditis subaguda posparto?


• R = VSG NORMAL que la DIFERENCIA DE LA DE QUERVAIN.

• Cual es el manejo de la tiroiditis subaguda posparto?


1) Rara vez requiere tratamiento
2) En la FASE DE TIROTOXICOSIS es útil un BETABLOQUEADOR
3) En la fase de HIPOTIROIDISMO puede usarse LEVOTIROXINA
4) Los fármacos anti tiroideos y los esteroides NO ESTÁN INDICADOS

• Cual es la etiología de la tiroiditis subaguda o de Quervain?


1) Suele ser precedida por un PROCESO INFECCIOSO DE LAS VAS
2) Se ha relacionado con ECHOVIRUS, ADENOVIRUS, COXACKIE VIRUS Y VIRUS DE LA
PAROTIDITIS
3) La edad de presentación es entre 20-40ª
PATOLOGIA DE TIROIDES
• Cual es el cuadro clínico de la TIROIDITIS DE QUERVAIN?
EL cuadro clínico comienza con pródromos de :
1) MIALGIAS GENERALIZADAS
2) Faringitis
3) Fatigas
4) FIEBRE
5) DOLOR Y EDEMA DEL CUELLO
• Como se encuentra la VSG en la tiroiditis de Quervain?
1) ELEVACIÓN DE LA VSG
2) Elevación de PCR

• Cual es el manejo de tiroiditis subaguda o de Quervain?


• R = ASA es de elección, PROPANOLOL para SÍNTOMAS TIROTOXICOS

• Cual es el manejo del ESTROMA DE YEDEL?


• R = TAMOXIFENO de elección por años
PATOLOGIA DE TIROIDES
• Un teratoma ovárico que patología puede ocasionar, relacionado a la producción de
hormonas?
• R = TIROTOXICOSIS por producción de TSH

• Como afecta el aumento de hormonas tiroideas a los huesos?


• R = Aumenta la estimulación de la RESORCIÓN ÓSEA.

• Cuales son las manifestaciones clínicas del CRETINISMO?


• R = Alteración del desarrollo esquelético y del SNC, CARA TOSCA, PROTRUSIÓN DE LA
LENGUA Y HERNIA UMBILICAL.

• A que se le llama síndrome de plumer?


• R = Cuando uno de los nódulos del BOCIO MULTINODULAR se hace HIPERFUNCIONANTE
(bocio multinodular toxico), puesto que la mayoría de los pacientes son eutiroideos

• Porque en un paciente que utiliza corticosteroides de manera crónica se adelgaza y se ve


fina la piel?
• R = Debido al CATABOLISMO PROTEICO DE LAS PROTEÍNAS consistentes en perdidas de
colágeno y resorción ósea.

• Que alimentos se consideran bociogenos?


• R = Mandioca, col de brucelas, repollo y coliflor
PATOLOGIA DE TIROIDES
• Cual es la estirpe histológica mas común del carcinoma de tiroides y que lo caracteriza?
1) Carcinoma PAPILAR en 70-90%,
2) Asociado al GEN RET
3) El antecedente de RADIACIÓN EN CUELLO en la infancia se relaciona
4) Manifestándose como NÓDULO ÚNICO,
5) Donde la citología muestra CÉLULAS NUCLEARES SOBREPUESTAS CON APARIENCIA
DE VIDRIO ESMERILADO y surcos longitudinales.
6) Mejor pronostico,

• Con que enfermedades se relaciona el carcinoma papilar de tiroides?


1) Síndrome de GARDNER (poliposis adenomatosa familiar de colon)
2) Enfermedad de COWDEN (bocio familiar y hamartromas cutáneos)
3) Síndrome de TURCOT (tumores cerebrales mas poliposis en intestino grueso)

• A que tipo de cáncer tiroideo son mas propensos los pacientes con enfermedad de
Cowden?
1) FOLICULAR, AD,
2) Lo causa perdida de gen supresor de tumores.
3) Se caracteriza comúnmente por macrocefalia, HAMARTROMAS MÚLTIPLES, cáncer
mamario o PÓLIPOS INTESTINALES.
PATOLOGIA DE TIROIDES
• Que distingue al carcinoma FOLICULAR tiroideo del papilar?
1) Que ESTA ENCAPSULADO,
2) INVADE VASOS SANGUÍNEOS.
3) Su VARIEDAD extensamente INVASIVA son las CÉLULAS DE HURTLE
4) Tiene MAL PRONÓSTICO.
5) Al extirparlo se hace SEGUMIENTO CON TIROGLOBULINA para descartar metástasis

• Que origen tiene el CARCINOMA MEDULAR DE TIROIDES y que sustancia produce?


1) Se origina de las CÉLULAS C PARAFOLICULARES,
2) Secretor de CALCITONINA.
3) Secretor de ACTH
4) Asociado a mutación en oncogen RET, debido a eso SE DEBEN ESTUDIAR TAMBIÉN A
FAMILIARES

• Cual es el carcinoma de tiroides con peor pronostico?


1) ANAPLASICO con mortalidad del 95% en los primeros 6 meses.
2) Crecimiento rápido
3) Se han descrito supresión del gen supresor de tumores p53
PATOLOGIA TIROIDES
• Cual es el tratamiento de elección en carcinoma
tiroideo?
1) Extirpación quirúrgica
2) Tratamiento medico con la INHIBICIÓN DE LA
TSH DANDO LEVOTIROXINA A DOSIS ALTAS
3) Yodo radiactivo indicado en pacientes
POSTOPERADOS DE CA TIROIDEO
4) Radioterapia externa: Solo en pacientes con
EXTIRPACIÓN QUIRÚRGICA INCOMPLETA
5) Vigilancia clínica y por us
SX DE CUSHING
• Cual es la principal causa del Sx Cushing y su cuadro clínico?
1) Exógena por esteroides y 2da causa es por micro adenoma hipofisario
2) OBESIDAD CENTRAL
3) HAS por aumento del ANGIOTENSINOGENO HEPATICO
4) Intolerancia a la glucosa por incremento de la GLUCONEOGENESIS
inducida por los glucocorticoides
5) FACIES DE LUNA LLENA
6) Trastornos menstruales
7) HIRSUTISMO

• Como diagnosticas el sx de Cushing?


1) Inicialmente se debe demostrar HIPERCOLESTEROLISMO
2) Determinación de CORTISOL LIBRE URINARIO con sensibilidad de 90%
3) SUPRESIÓN CON DOSIS BAJAS DE DEXAMETASONA (prueba de
NUGENT)
4) Cuando un paciente tiene un cortisol urinario y serico a las 8:00 am > 1.8
después de 1 mg/dl de dexametasona a las 23 hrs de la noche previa, el
paciente tiene síndrome de Cushing.
SX DE CUSHING
• En cuanto al origen del síndrome de Cushing cuales son los estudios a realizar?
1) Se determina la hormona ADRENOCORTICOTROPICA plasmática, LOS VALORES POR
DEBAJO DEL LIMITE DE DETECCIÓN CATALOGAN AL SÍNDROME COMO DE ORIGEN
SUPRARRENAL.
2) La RM es la técnica de imagen de elección en el síndrome de Cushing de ORIGEN
HIPOFISARIO.
3) La TAC es de elección para visualizar las GLÁNDULAS SUPRARRENALES.

• En que consiste la PRUEBA DE NUGENT en el diagnostico de sx de Cushing?


• R = SUPRESIÓN DE ACTH con administrar 1mg de dexametasona a las 23 Hrs y con nueva
medición a las 8 AM.

• En que consiste el tratamiento medico del sx de Cushing?


• R = KETOCONAZOL, pues INHIBE LA SÍNTESIS DE CORTISOL pero se emplea por periodos
cortos pues es hepatotoxico.

• En que consiste el tratamiento quirúrgico del sx de Cushing?


1) CIRUGÍA HIPOFISARIA
2) RADIOTERAPIA HIPOFISARIA que ha mostrado ser efectiva en los casos que no se pueda
realizar la CIRUGÍA TRANSESFENOIDAL
ENFERMEDAD DE ADDISON
• Cual es la principal causa de la enfermedad de Addison/insuficiencia suprarrenal?
1) 90% adrenalitis AUTOINMUNE,
2) En países subdesarrollados TB.

• Cual es el cuadro clínico y DHE de la enfermedad de Addison?


1) HIPERPIGMENTACION de la piel (debido a que la deficiencia de cortisol,
provoca un amento compensatorio de la PROPIOMELANOCORTINA que es
precursora de la ACTH y de la hormona estimulante de melanocitos)
2) Vomito
3) Fatiga
4) Hipoglucemia
5) HIPOTENSIÓN
6) HIPONATREMIA
7) HIPERKALEMIA.
• Cual es el desequilibrio hidroelectrolitico principal en la enfermedad de Addison?
• R = ACIDOSIS METABOLICA con HIPERCALCEMIA.
ENFERMEDAD DE ADDISON
• Cuales son las manifestaciones clínicas de una CRISIS ADISONIANA o suprarrenal?
1) FIEBRE,
2) Deshidratación,
3) DOLOR ABDOMINAL
4) HIPOTENSIÓN O CHOQUE HIPOVOLEMICO,
5) Urea elevada
6) HIPONATREMIA,
7) HIPERKALEMIA,
8) HIPERCALCEMIA y
9) ACIDOSIS METABÓLICA.
• Cual es el tratamiento de la crisis Adisoniana?
• R = Administración inmediata de HIDROCORTISONA IV C-8

• Como realizas el diagnostico de enfermedad de Addison?


1) ESTANDAR DE ORO es la prueba de estrés con INSULINA IV la cual hace el diagnostico de adrenalitis
secundaria, DONDE EL CORTISOL DEBE AUMENTAR, SINO ENTONCES SE HACE EL DIAGNOSTICO.
2) Medición de CORTISOL DURANTE LA MAÑANA y ADRENOCORTICOTROPINA con NIVELES BAJOS.
3) ACTH IM y MEDIR CORTISOL A LOS 60 MIN, EL CUAL DEBE AUMENTAR.
4) En los pacientes con adrenalitis autoinmune se encuentran AUTOANTICUERPOS ANTI-21HIDROXILASA.

• Cual es el manejo de la enfermedad de Addison?


• R = Hidrocortisona o PREDNISONA.
HIPERALDOSTERONISMO
• Cual es la etiología del hiperaldosteronismo?
1) Primario: Hipersecreción de aldosterona la cual NO SE SUPRIME EN RESPUESTA A LA EXPANSIÓN DE
VOLUMEN.
2) Secundario: Altos niveles de renina la cual se reprime en respuesta a la expansión de volumen.

• Cuales son los datos clínicos del hiperaldosteronismo?


1) Puede cursar asintomático o
2) Cefalea
3) HAS
4) DEBILIDAD MUSCULAR
5) PARESTESIAS
6) Tetania
7) Parálisis

• Cuales son los datos de laboratorio encontrados en Hiperaldosteronismo primario?


• R = HIPERNATREMIA, HIPOKALEMIA y ALCALOSIS METABÓLICA

• -A 30-year-old man is having symtoms of muscle weakness, fatigue, and headaches. On examination, his blood
pressure is 180/100 mm Hg, pulse 80/min, JVP 4 cm, heart sounds normal, and lungs clear. His serum
potassium level is 2.5 mEq/L and bicarbonate 30 mEq/L. The urine potassium is 40 mEq/L. Plasma renin is low
and aldosterone is high.

• -Primary hyperaldosteronism is characterized by hypertension with high plasma aldosterone and low plasma
rennin
HIPERALDOSTERONISMO
• Como diagnosticas hiperaldosteronismo?
• R = ELEVACIÓN DE ALDOSTERONA PLASMÁTICA POSTERIOR A LA INGESTIÓN DE
CLORURO DE SODIO por cada comida durante dos a tres días o un NIVEL DE
ALDOSTERONA EN PLASMA ALTO DESPUÉS DE LA INFUSIÓN INTRAVENOSA DE
SOLUCIÓN SALINA (1 lt x hr durante 4 hrs).

• Cual es el manejo del hiperaldosteronismo?


1) CIRUGÍA A PACIENTES CON ALDOSTERONOMA con adrenalectomía laparoscópica
2) En el caso de HIPERPLASIA SUPRARRENAL BILATERAL IDIOPÁTICA se debe recibir
TRATAMIENTO MEDICO CON ESPIRONOLACTONA
3) En el caso de hiperaldosteronismo tratable con esteroides se utiliza la
DEXAMETASONA para mantener una supresión del eje hipotálamo hipófisis.

• Cuales son los datos que caracterizan al síndrome de Conn en


hiperaldosteronismo?
• R = Hipotensión, hipokalemia, aumento de aldosterona y supresión secundaria de
renina.
HIPOALDOSTERONISMO
• Cual es la etiología del hipoaldosteronismo?
1) Puede producirse por un estimulo insuficiente por parte de la renina
(HIPORRENINEMICO) “NEFROPATÍA DIABÉTICA”
2) Por un fallo primario en la secreción de aldosterona (HIPERRENINEMICO)
“ENFERMEDAD DE ADDISON”
3) Por resistencia a la acción de la aldosterona
(SEUDOHIPOALDOSTERONISMO).
4) Algunos medicamentos pueden ocasionarlo como los AINES, heparina (suprime la
secreción de renina), IECAS.

• Cual es el cuadro clínico del hipoaldosteronismo?


1) CALAMBRES
2) Debilidad muscular

• Cuales son los datos de laboratorio del hipoaldosteronismo?


1) HIPERKALEMIA
2) HIPONATREMIA
3) ACIDOSIS METABOLICA HIPERCLOREMICA
4) Deshidratación
5) Se debe hacer diagnostico diferencial de enfermedad de Addison
HIPOALDOSTERONISMO
• Como se diagnostica el hipoaldosteronismo?
1) Analíticamente se deben obtener los niveles plasmáticos de aldosterona,
cortisol y actividad de renina
2) Si se sospecha de enfermedad de ADDISON SE DEBE REALIZAR LA PRUEBA
DE ESTIMULACIÓN CON HORMONA ACTH.
3) Si existe aldosterona baja con actividad de renina (HIPORRENINEMICO),
debemos pensar en NEFROPATIA DIABETICA,
4) Si obtenemos aldosterona baja y renina alta (HIPERRENINEMICO),
debemos sospechar ADISON o una forma grave de hiperplasia suprarrenal
congénita.
5) Si lo que nos reportan es ALDOSTERONA ELEVADA Y ACTIVIDAD DE
RENINA ALTA, debemos pensar en SEUDOHIPOALDOSTERONISMO.

• Cual es el manejo del hipoaldosteronismo


1) INGESTIÓN LIBRE DE SAL como TRATAMIENTO MEDICO no farmacológico
2) El tratamiento medico CON FÁRMACOS es con FLUDROCORTISONA.
ALTERACIONES POR DHE, Na y K
• ENFERMEDAD DE ADDISON:
• Acidosis Metabolica con hipercalcemia
• DHE: Hiponatremia – Hiperkalemia
• Cual es el manejo de la enfermedad de Addison?
• R = Hidrocortisona o PREDNISONA.

• HIPOALDOSTERONISMO:
• Acidosis Metabolica Hipercloremica
• DHE: Hiponatremia – Hiperkalemia
• Cual es el manejo del hipoaldosteronismo
1) INGESTIÓN LIBRE DE SAL como TRATAMIENTO MEDICO no farmacológico
2) El tratamiento medico CON FÁRMACOS es con FLUDROCORTISONA.

• HIPERALDOSTERONISMO:
• Alcalosis Metabolica
• DHE: Hipernatremia - Hipokalemia
• Cual es el manejo del hiperaldosteronismo?
1) CIRUGÍA A PACIENTES CON ALDOSTERONOMA con adrenalectomía laparoscópica
2) En el caso de HIPERPLASIA SUPRARRENAL BILATERAL IDIOPÁTICA se debe recibir
TRATAMIENTO MEDICO CON ESPIRONOLACTONA
3) En el caso de hiperaldosteronismo tratable con esteroides se utiliza la DEXAMETASONA
para mantener una supresión del eje hipotálamo hipófisis.
HIPERCALCEMIA
• Cuales son los eventos metabólicos implicados en la regulación del calcio?
1) En presencia de hipercalcemia se estimula la secreción de CALCITONINA, la cual INHIBE LA OSTEOLISIS y
ESTIMULA LA CALCIURIA.
2) Cuando la hipercalcemia permanece, la CALCITONINA INHIBE LA REABSORCIÓN OSTEOCLASTICA, LA
SECRECIÓN DE PARATHORMONA Y LA SÍNTESIS DE VITAMINA D.

• Cual es la etiología de la hipercalcemia?


1) HIPERPARATIROIDISMO PRIMARIO: Puede deberse a ADENOMA ÚNICO o HIPERTROFIA GLANDULAR. Se
caracteriza por ELEVACIÓN DE LA PTH, HIPERCALCEMIA E HIPOFOSFATEMIA.
2) Hiperparatiroidismo secundario: Puede deberse a IR, sx de mal absorción y raquitismo.
3) El adenoma y las neoplasias malignas causan 90% de las hipercalcemias.
4) Es frecuente en el CÁNCER DE MAMA.

• Cual es el cuadro clínico de la hipercalcemia?


1) Ca entre 10.5-12 g/dL es asintomático
2) Niveles mayores forman TRIADA CLÁSICA DE ESTREÑIMIENTO, NAUSEA Y VOMITO.
3) Fatiga, letargo, cefalea, sed, nausea, vomito, NEFROLITIASIS,
4) ACORTAMIENTO DEL INTERVALO QT EN EL EKG.

• Cual es el manejo de la hipercalcemia?


1) HIDRATACION
2) FUROSEMIDE es eficaz para evitar la reabsorción tubular de calcio

• Cual es el fármaco que se indica en hipercalcemia crónica?


• R = TIAZIDAS pues previenen cálculos renales.
HIPOCALCEMIA
• Cual es la etiología de la hipocalcemia?
1) Ca sérico menor 8.5. Previamente se debe confirmar que la albumina se encuentre en niveles séricos
normales, ya que el descenso de 1 g/dL de albumina se acompaña de un descenso de 0.8 mg/dL de
calcio.
2) La causa mas común de HIPOPARATIROIDISMO ES LA CIRUGÍA TIROIDEA Y LA SEGUNDA ES LA
RADIACIÓN.
3) La hipomagnesemia < 1 mg/dL puede producir hipocalcemia
4) La hipermagnesmia reduce la secreción de PTH.
5) La hiperfosfatemia causa hipercalcemia por formación de complejos con el calcio.
6) En pacientes transfundidos se produce hipocalcemia ya que el citrato del paquete globular actúa como
paquete globular.
7) EL SINDROME DE DIGEORGE se caracteriza por AUSENCIA DE GLÁNDULAS PARATORIDEAS, con DÉFICIT
DE PTH, ASOCIADO A DISPLASIA TIMICA, y malformaciones cardiacas por alteración en el desarrollo
de LAS TERCERAS Y CUARTAS BOLSAS FARÍNGEAS.
8) Las formas adultas de hipoparatiroidismo se asocian a enfermedades autoinmunes como insuficiencia
suprarrenal, anemia perniciosa e hipotiroidismo.
9) En la PANCREATITIS, el calcio se deposita en áreas con necrosis grasa.
10) EL SINDROME DE HUESO HAMBRIENTO se presenta en paratiroidectomia.
11) En enfermedades del APARATO DIGESTIVO se favorece el déficit de vitamina D por disminución en la
absorción.
12) La DIFENILHIDANTOINA inhibe la absorción de calcio.

• Cual es el cuadro clínico de la hipocalcemia?


1) HIPEREXITABILIDAD neuromuscular.
2) Parestesias, HIPERREFLEXIA, ESPASMO CARPOPEDAL, irritavilidad, SIGNO DE CHEVOSTEK Y TROSSEAU.
3) El EKG MUESTRA PROLONGACIÓN DEL ST.
OSTEOPOROSIS
• Cual es el manejo de la osteoporosis?
1) ALENDRONATO, RISENDRONATO.
2) PTH a dosis bajas, al igual que los bifosfonatos han demostrado aumento en la
masa ósea,
3) Terapia de reemplazo hormonal con estrógenos previene fracturas vertebrales.
4) RALOXIFENO Y TAMOXIFENO ya que AUMENTAN LA DENSIDAD ÓSEA,
DISMINUYEN RIESGO DE FRACTURAS VERTEBRALES Y EL CÁNCER DE MAMA
5) La TIBOLONA es una sustancia sintética con acción tejido específica que tiene
efecto ESTROGÉNICO, PROGESTACIONAL Y ANDROGÉNICO según el lugar de
acción. Esta molécula mejora los síntomas vasomotores, el estado anímico, la
libido, la atrofia urogenital y previene la osteoporosis postmenopáusica; no
produce mastalgia ni modifica la densidad mamográfica. Se observan además
efectos beneficiosos sobre algunos parámetros cardiovasculares y hemostáticos.
EN EL ENDOMETRIO, LA TIBOLONA SE CONVIERTE EN SU METABOLITO
PROGESTAGÉNICO/ANDROGÉNICO Y EN CONSECUENCIA NO PRODUCE
ESTIMULACIÓN ENDOMETRIAL NI SE NECESITA AGREGAR UN PROGESTÁGENO.
Esta ventaja mejora el cumplimiento de la terapia hormonal de reemplazo (THR)
debido que no provoca sangrados por disrupción
DENSITOMETRIA OSEA
OSTEOMALACIA
• Cual es la etiología de la osteomalacia?
• R = Déficit de VITAMINA D EN ADULTOS

• Cual es el cuadro clínico de la osteomalacia?


1. DEBILIDAD MUSCULAR PROXIMAL, asociada con desgaste muscular, hipotonía y dificultad para la
movilización.
2. EL OSTEOCOPO (dolor óseo), es mas notorio en LA REGION LUMBAR, PELVIS Y EXTREMIDADES
PELVICAS DONDE SE PRESENTAN FRACTURAS.
3. Las fracturas suceden con un mínimo de traumatismo.
4. Los síntomas deben incluir: DOLOR OSEO, DEFORMIDADES DEL ESQUELETO Y/O CRANEO, piernas
arqueadas o rodillas malformadas, PECHO EN PALOMA, Deformaciones dentales.

• Como se diagnostica la osteomalacia?


1. MARCLAJE CON TETRACICLINAS. Este medicamento se deposita en forma de bandas en el sitio de
mineralización y al ser fluorescentes pueden ser visualizados al microscopio. La tasa de crecimiento
del esqueleto puede estimarse en las biopsias de las crestas iliacas al medir la distancia entre las
bandas de tetraciclina. Si disminuye la distancia entre las bandas se hace el diagnostico.
2. EL HALLAZGO RADIOGRÁFICO CARACTERÍSTICO ES LA REDUCCIÓN EN LA
DENSIDAD ÓSEA CON ADELGAZAMIENTO DE LA CORTEZA.
3. Laboratorio: Los pacientes con déficit de vitamina D se presentan con HIPOFOSFATEMIA, CALCIO
SÉRICO BAJO E HIPOPARATIROIDISMO SECUNDARIO.

• Cual es el manejo de la osteomalacia?


• R = Calcitriol o CALCIO + VITAMINA D en combinación.
DIABETES INSIPIDA CENTRAL
• A que se debe la diabetes insípida central primaria y secundaria?
1) Primaria : Ac vs ARGININA-VASOPRESINA.
2) LESIÓN DEL HIPOTÁLAMO, del tallo hipofisario o por cualquier CAUSA
INFECCIOSA, TRAUMÁTICA etc..

• Cuales son las causas de DI central y cuales son los hallazgos en la TAC?
• R = DI IS USUALLY CAUSED BY DESTRUCTION, OR AGENESIS, OF THE POSTERIOR
PITUITARY, its normal signaling is lost. PITUITARY DI CAN ALSO RESULT FROM
TRAUMA, TUMORS (BOTH PRIMARY AND SECONDARY), granulomas, infections,
inflammatory diseases, chemical toxins, congenital malformations, and genetic
disorders. Depending on the cause, the MRI may demonstrate other associated
findings.

• Que causa diabetes insípida en el ultimo trimestre de embarazo-puerperio?


• R = Es inducido por VASOPRESINA en el ultimo trimestre de embarazo, relacionada
frecuentemente con oligohidramnios, preeclampsia o disfunción hepática.
RESPONDE A DESMOPRESINA
DIABETES INSIPIDA NEFROGENICA
• Como diagnosticas DM insípida?
1) JUICIO CLÍNICO, no hay prueba de laboratorio contundente.
2) RECOLECCIÓN DE ORINA DE 24 HRS MIDIENDO VOLUMEN <2LTS/24 HRS SIN
HIPERNATREMIA.
3) Desmopresina: MIDIENDO VOLUMEN URINARIO 12 HRS ANTES Y 12 HRS DESPUÉS, EN EL
CUAL LOS PACIENTES CON DIABETES INSÍPIDA CENTRAL HABRÁ DISMINUCIÓN DE SED Y
POLIURIA,
4) Realizándose IRM analizando hipófisis e hipotálamo, se observa saco hipofisario
engrosado: Células de langerhans, sarcoidosis, hipofisitis linfocitica, Histiocitosis
(proliferación de macrófagos en diferentes aparatos y sistemas).
5) Desmopresina en DM nefrogenica valorando su concentración con restricción de líquidos
aumentando sus valores.

• Cual es el manejo de la DM insípida nefrogenica o central?


1) CENTRAL: DESMOPRESINA tmb se utiliza en DM INSÍPIDA RELACIONADA CON
EMBARAZO o puerperio, evitar factores de agravamiento como administración de
glucocorticoides.
2) NEFROGENICA: HIDROCLOROTIAZIDA con suplemento de potasio, INDOMETACINA en
fase aguda.

ACROMEGALIA
Cual es la causa principal de acromegalia?
• R = ADENOMA HIPOFISIARIO, también por secreción ectópica de GnRH o GH por tumores carcinoides hipotalámicos,
bronquiales o pancreáticos.

• Que características forman parte del SÍNDROME DE CARNEY?


• R = MIXOMA AURICULAR, NEUROMA DEL ACÚSTICO Y PIGMENTACIÓN PUNTEADA DE LA PIEL

• Cuales son los signos y síntomas de la acromegalia o gigantismo?


1) ESTATURA ELEVADA, gigantismo antes del cierre hiposifiario.
2) Acromegalia significa crecimiento de extremidades.
3) SÍNDROME DEL TÚNEL CARPIANO.
4) RASGOS FACIALES TOSCOS, PROGNATISMO CON MALA OCLUSIÓN.
5) HAS 50%. DM 30%
• Cuales son los lab para dx acromegalia o gigantismo?
1) AYUNO. > IGF-1 5 VECES DE LO NORMAL.
2) Prolactina aumentada algunas veces.
3) Glucosa: la DM ES COMÚN.
4) Calcio: Para descartar hiperparatiroidismo.
5) Fosforo: Se encuentra aumentada.
6) T4 y TSH: HIPOTIROIDISMO COMÚN.
7) Se recomienda ADMINISTRAR UNA SOLUCIÓN DE DEXTROSA MIDIENDO LA GH 1 HR DESPUÉS, SI LOS VALORES ESTÁN < O
NORMALES AL IGUAL QUE LA IGF-1, YA QUE ESTA HORMONA RESPONDE CON DISMINUCIÓN A LA
INGESTA DE GLUCOSA.
• Que medicamentos se utilizan en caso de no remitir > GH o IGF-1 con adenectomía transesfenoidal hipofisiaria?
1) ANÁLOGOS DE LA SOMATOSTATINA COMO OCTREOTIDE para conservar disminución del tamaño del adenoma. GH sérica 1
mg y 2.5 mg, pero no responde bien si previos al tratamiento la GH > 20 ng/ml.
2) PEGVISOMANT es un antagonista del receptor de la GH bloqueando sus efectos, vía SC, pero no reduce el tamaño de los
adenomas.
3) RADIOCIRUGÍA: en los pacientes no responden al tratamiento quirúrgico o medico, no se realiza si hay extensión suprecelar
por que puede dañar el quiasma óptico.
NEM I
• Cuales son las características de la NEM I O SÍNDROME DE WERMER?
1) Se caracteriza por neoplasias en PARATIROIDES, PÁNCREAS y ADENOHIPOFISIS.
2) Se hereda de forma AD en el CROMOSOMA 11

• Cual es la clínica del NEM I o Sindrome de Wermer?


1) HIPERPARATIROIDISMO PRIMARIO: Que puede manifestarse con MIALGIAS, DEBILIDAD, HIPERCALCEMIA,
ALTERACIONES MENTALES, NEFROLITIASIS Y ANORMALIDADES ÓSEAS. La hipercalcemia puede provocar
aumento de la secreción de gastrina y un Sx Zollinger-Ellison secundario. EL TRATAMIENTO CONSISTE EN LA
EXTIRPACIÓN QUIRÚRGICA DE LAS PARATIROIDES.
2) TUMORES DE LAS CÉLULAS DE LOS ISLOTES PANCREÁTICOS: Los GASTRINOMAS SON LA CAUSA MAS
IMPORTANTE de morbimortalidad y la mayoría son multiples. Los insulinomas son generalmente benignos y
únicos.
3) NEOPLASIA DE LA ADENOHIPOFISIS: La neoplasia MAS FRECUENTE ES EL PROLACTINOMA seguida del tumor
secretor de hormona del crecimiento que causa acromegalia, pueden presentarse como enfermedad de
Cushing secundaria a adenoma hipofisario.

• Como realizas el diagnostico de NEM I?


1) Con 2 de las 3 neoplasias características del síndrome
2) El MEN I familiar se define como la presencia de un paciente afectado con esta enfermedad y un familiar de
primer grado que presente al menos una de las tres neoplasias características

• Cual es el manejo del NEM I?


1) CIRUGÍA
2) PARA CADA MANIFESTACIÓN de sobre producción hormonal se da el medicamento individual
MEN II
• Cual es la etiología cromosómica del MEN II?
• R = Se hereda de forma AD, en el CROMOSOMA 10.
• Cual es la clasificación del MEN II?
 MEN II A o Síndrome de Sipple: Con la presencia de cáncer medular de
tiroides, FEOCROMOCITOMA e hiperparatiroidismo primario.
 MEN II B: Con la presencia de cáncer medular de tiroides, feocromocitoma,
NEUROMAS MUCOSOS, ganglioneuromatosis intestinal y HABITO
MARFANOIDE.

• Como diagnosticas el MEN II?


1) 2 a 3 tumores de los antes mencionados
2) La mutación del protoncogen RET esta indicada en todos los familiares de
primer grado.
3) EL CARCINOMA MEDULAR DE TIROIDES SUELE SER LA PRINCIPAL CAUSA
DE MUERTE.
GASTRINOMA
• Cuales son los tumores de las células de los islotes pancreáticos?
• R = Gastrinoma, insulinoma, glucagonoma, vipoma y somatostinoma. GIGVS
• Cual es el cuadro clínico del gastrinoma?
1) Enfermedad ulcerosa con dolor abdominal
2) Diarrea
3) ERGE.
4) Se debe sospechar en ULCERAS PÉPTICAS MÚLTIPLES.

• Como diagnosticas el gastrinoma?


1) Los niveles séricos de GASTRINA EN AYUNO MAYORES DE 1000 Y PH < 2.5 EN LA SECRECIÓN GÁSTRICA
HACEN EL DIAGNOSTICO DE GASTRINOMA.
2) Se debe SUSPENDER LA ADMINISTRACIÓN DE ANTAGONISTAS DE H2 UNA SEMANA ANTES Y DE
OMEPRAZOL.
3) En paciente con niveles séricos menores a 1000 y PH < 2.5 se debe pensar en H. Pilory
4) La aclorhidria por gastritis atrófica o anemia perniciosa es una de las causas mas frecuentes de
hipergastrinemia.

• Cual es el manejo del gastrinoma?


1) Para la hipersecreción acida OMEPRAZOL
2) Gastrinoma RESECCIÓN QUIRÚRGICA PARA EVITAR METÁSTASIS

INSULINOMA
Cual es el cuadro clínico del insulinoma?
1) SECUNDARIOS AL EFECTO HIPOGLUCEMICO como cefalea, mareo, debilidad, confusión, desorientación y coma.
2) Síntomas secundarios al aumento de catecolaminas circulantes provocado por la hipoglucemia como sudación, temblor y
palpitación.
3) La mayoría de las crisis se relaciona con el ayuno.

• Como realizas el diagnostico de insulinoma?


1) Se basa en la TRIADA DE WIPPLE la que consiste en síntomas de neuroglucopenia, niveles de glucosa en sangre por debajo
de 45 y mejoría inmediata de los síntomas después de la administración de glucosa
2) Si se sospecha de insulinoma se debe realizar una PRUEBA DE AYUNO DURANTE 72 HRS, considerado como el método
diagnostico de elección, donde se miden los niveles de glucosa, péptido C e insulina cada 6 hrs y siempre que exista clínica de
hipoglucemia la prueba se suspende si aparece una glucemia inferior a 45 o si se presentan síntomas neuroglucopenicos. En
sujetos normales la insulina debe disminuir en presencia de hipoglucemia. Los criterios diagnósticos de insulinoma incluyen
insulina plasmática mayor de 6 cuando la glucemia es menor de 40, proinsulina > .2, péptido C > 200 y cociente de
insulina/glucosa > .3.
3) Existen AINTICUERPOS ANTIINSULINA Y NIVELES ELEVEADOS DE PROINSULINA.

• Cual es el manejo del insulinoma?


1) Para las crisis de hipoglucemia se realizan comidas frecuentes y de escasa cantidad
2) En el caso de hipoglucemia se administra 50 ml de dextrosa al 50%, si no hay respuesta dentro de los primeros 15 minutos, la
dosis puede repetirse, en caso de que no se pueda canalizar al paciente se administra 1 mg de guagón por vía IV, IM o
subcutánea.
3) El DIAZOXIDO inhibe la secreción de insulina, pero provoca retención de líquidos e hirsutismo con frecuencia.
4) La resección completa del tumor cura a la mayoría de los pacientes. Cuando no se logra localizar el tumor se realiza una
pancreatectomia distal gradual hasta localizarlo dejando por lo menos 20-30% del páncreas hasta localizarlo.
5) EN EL CASO DE TUMOR MALIGNO CON METÁSTASIS O TUMOR IRRESECABLE SE RECOMIENDA EL
TRATAMIENTO A LARGO PLAZO CON DIAZOXIDO, SI NO RESPONDE SE INICIA QUIMIOEMBOLIZACION.
• Que son los vipomas?
VIPOMAS
• R = Son tumores derivados de las CÉLULAS DE LOS ISLOTES DEL PÁNCREAS que producen grandes
cantidades de PEPTIDO INTESTINAL VASOACTIVO, el cual AUMENTA LA CONTRACCIÓN
DEL MUSCULO LISO, INHIBE LA SECRESION GÁSTRICA Y TIENE EFECTOS VASODILATADORES.

• Cual es el cuadro clínico asociado a vipomas?


1) El síntoma predominante es una DIARREA ACUOSA, que produce deshidratación,
hipokalemia y debilidad.
2) La DIARREA ES SECRETORA, persiste durante el ayuno y NO RESPONDE A
ANTIDIARREICOS.
3) Acidosis metabólica

• Como realizas el diagnostico del vipoma?


1) Presencia de DIARREA SECRETORA
2) Niveles elevado de PÉPTIDO INTESTINAL VASOACTIVO
3) TUMORACION pancreática

• Como se maneja el vipoma?


1) Corregir la deshidratación y desequilibrio hidroelectrolitico, en especial de la HIPOKALEMIA.
2) Resección del tumor y en presencia de metástasis se reduce primero el tumor.
3) Si no se encuentra el tumor se busca en las glándulas suprarrenales y las cadenas simpáticas, en caso
de no identificarse en ninguno de estos sitios se realiza resección del 80% del tumor.
4) En el caso de tumor maligno inoperable o presencia de metástasis se recomienda el uso del análogo
de la somatostatina de acción prolongada, también puede utilizarse la embolizacion de la arteria
hepática.
GLUCAGONOMAS
• Que son los glucagonomas?
1) Son tumores derivados de las CELULAS ALFA DE LOS ISLOTES DEL PANCREAS los cuales producen una gran
cantidad de la HORMONA GLUCAGON y puede secretar otros péptidos (polipeptido pancreático, gastrina y somatostatina).
2) Es mas frecuente en mujeres PERI O POSTMENOPAUSICAS

• Cual es el cuadro clínico de los glucagonomas?


1) DERMATITIS NECROLITICA MIGRATORIA es característica de los glucagonomas. Inicia como una zona eritematosa y
escamosa en la ingle y glúteo, se extiende al perineo y extremidades inferiores, en ocasiones hay lesiones peri orales,
posteriormente se hace ampolloso y deja erosiones cuando las ampollas se rompen.
2) Intolerancia a la glucosa o DIABETES
3) Anemia
4) QUEILITIS Y ESTOMATITIS
5) Diarrea
6) Hipercolesterolemia

• Como diagnosticas los glucagonomas?


1) Con el cuadro clínico y se confirma al encontrar niveles plasmáticos de GLUCAGON EN AYUNO > 1000 (NORMAL 150) Y
UNA TUMORACIÓN PANCREÁTICA.
2) NO SE SUPRIME LA LIBERACIÓN DEL GLUCAGON CON LA ADMINISTRACIÓN DE GLUCOSA Y TAMPOCO SE
ESTIMULA CON LA ADMINISTRACIÓN DE ARGININA.

• Cual es el manejo del glucagonoma?


1) Debido a la presencia de METASTASIS EN LA MAYORIA DE LOS PACIENTES, no es posible una resección quirúrgica
curativa, pero en estos casos se extirpa el tumor primario y se reduce el volumen de las metástasis.
2) Los SUPLEMENTOS DE ZINC mejoran la dermatitis
3) Los ANALOGOS DE LA SOMATOSTATINA normalizan los valores de glucagon y aminoácidos séricos, mejoran la dermatitis
y la diarrea, promueven el aumento de peso, pero NO corrigen la intolerancia a la glucosa.
4) La quimioterapia NO es útil pero cuando se decide utilizarla, la combinación que ha demostrado mas eficacia es la de
ESTREPTOZOCINA +DACARBACINA.
5) EL 50% DE LAS MUERTES ES CAUSADO POR EVENTOS TROMBOEMBOLICOS.
SOMATOSTINOMAS
• Que son los somatostatinomas?
1) Son tumores derivados de las CELULAS DELTA DEL PANCREAS productores de
somatostatina, al igual que los VIPomas y glucagonomas
2) 70% SON MALIGNOS
3) La somatostatina es un inhibidor potente de la secreción gástrica y pancreática

• Cual es el cuadro clínico de los somatostatinomas?


1) Es causado por la inhibición de la secreción tanto exocrina como endocrina del páncreas
2) LA TRIADA CLASICA CONSISTE EN DIABETES, DIARREA-ESTEATORREA Y
COLELITIASIS.

• Como diagnosticas los somatostatinomas?


• R = La mayoría se localiza incidentalmente al realizar cirugías abdominales o endoscopias. EL
diagnostico se realiza al demostrar la somatostatina elevada en plasma.

• Cual es el manejo de los somatostatinamas?


1) ANÁLOGOS DE LA SOMATOSTATINA con el fin de inhibir la activación de los receptores por
parte de la somatostatina secretada por el tumor.
2) EXTIRPACIÓN QUIRÚRGICA si se localiza el tumor
3) Para enfermedad metastasica se usa QUIMIOTERAPIA
FEOCROMOCITOMA
• Cual es la etiología del feocromocitoma?
1) Tumores de origen NEUROECTODERMICO desarrollado a partir de las CÉLULAS CROMAFINES DEL SNS (ganglios
simpáticos y medula suprarrenal) productores de CATECOLAMINAS y la mayoría de ellos libera epinefrina y
norepinefrina.
2) Los feocromocitomas ASOCIADOS A NEM SECRETAN SOLO EPINEFRINA.

• Cual es la clasificación y localización mas común de los feocromocitomas?


1) Feocromocitomas si provienen de las células cromafines y paragangliomas si son de origen extra renal.
2) La localización mas frecuente es la adrenal 85%, siempre es unilateral, en caso de ser bilateral debe
sospecharse de NEM

• Cual es el cuadro clínico del feocromocitoma?


1) HAS 90%
2) Crisis adrenal caracterizada por triada clásica de: CEFALEA, PALPITACIONES Y SUDORACIÓN.

• Cuales son los métodos diagnósticos de feocromocitoma?


1) Demostración de niveles elevados de catecolaminas o sus metabolitos en sangre o en orina, lo que establece el
diagnostico.
2) Se determina el nivel de catecolamina libre o no metabolizadas (epinefrina y norepinefrina) o sus metabolitos (
metanefrina, normetanefrina y ACIDO VANILMANDELICO) en orina de 24 hrs.

• Cuales son los métodos de imagen para dx de feocromocitoma?


1) TAC
2) Gamagrafia para localizar paragangliomas

• Cual es el manejo del feocromocitoma?


1) Cirugía, pero antes de realizarla es necesario realizar un bloqueo alfadrenergico, generalmente se utiliza
FENOXIBENZAMINA durante 10-14d previos a la cirugía.
2) Durante la anestesia o manipulación del tumor SUELE APARECER HIPERTENSIÓN Y ARRITMIAS, LA PRIMERA SE
TRATA CON FENTOLAMINA Y LA SEGUNDA CON PROPANOLOL.
TUMOR CARCINOIDE
• Cual es el cuadro clínico producido por el tumor carcinoide?
• R = Producen síntomas inespecíficos, vagos, relacionados con el órgano afectado tales como:
1) Carcinoide apendicular: El la LOCALIZACIÓN MAS FRECUENTE. Si el tumor mide mas de 2 cm se debe realizar una
hemicolectomia derecha, si mide entre 1-2 cm e invade la serosa se debe realizar una hemicolectomia derecha y
linfadenectomia regional. LA APENDICETOMÍA SE CONSIDERA CURATIVA CUANDO MIDEN MENOS DE 1 CM.
2) Carcinoide rectal: Estos secretan mas glucagon mas que serotonina. Puede manifestarse como SANGRADO RECTAL,
DOLOR O CONSTIPACION. Los tumores mayores de > 2 cm se tratan con una resección anterior baja o una
abdominoperineal.
3) Carcinoide del intestino delgado: La mayoría se encuentra el ILEON. Pueden provocar desde molestias abdominales
leves hasta dolor abdominal intermitente causado por la obstrucción intestinal, la cual puede ser secundaria a la
fibrosis que producen estos tumores o con menor frecuencia se manifiestan con diarrea, hemorragia por ulceración
de la mucosa o intususcepción. SON LA CAUSA MAS FRECEUNTE DE SX CARCINOIDE.
4) Carcinoides pulmonares y bronquiales: Neumonías recurrentes, tos hemoptisis, dolor pleurítico. La resección en
cuna o segmentaria se recomienda para los tumores localizados en caso de los carcinoides atípicos se utilizan
resecciones mas amplias.
5) Carcinoides gástricos: Se originan de las células enterocromafines de la mucosa gástrica, se manifiestan como
anemia o dolor abdominal. EL tipo 1 y mas frecuente en mujeres se asocia a hipergastrinemia y gastritis atrófica.
6) Carcinoides del colon: La mayoría en COLON DERECHO, siendo el CIEGO el sitio mas afectado. Pueden producir dolor,
anorexia y perdida de peso.

• Como diagnosticas los tumores carcinoides?


1) Cuadro clínico
2) Concentración de serotonina o de sus metabolitos en orina
3) Imagen
4) Se confirma por biopsia
5) Medición del ACIDO 5-HIDROXINDOLACETICO en una muestra de ORINA de 24 hrs, el cual si es
mayor de 10 mg indica EXCESO DE SEROTONINA.
6) En SANGRE se miden los niveles de CROMOGRANINA A, glucoproteina producida por las células derivadas de la
cresta neural y secretada en los tumores neuroendocrinos, relacionada directamente con el volumen tumoral
HIPERPLASIA SUPRARRENAL CONGÉNITA
• Cual es la etiología del hiperandrogenismo suprarrenal?
• R = Déficit de 21-hidroxilasa, el gen responsable se denomina CYP21B localizado en el brazo
corto del cromosoma 6

• Cual es el cuadro clínico del hiperandrogenismo renal por déficit de 21-hidroxilasa?


A. FORMA CLÁSICA (PERDEDORA DE SAL O VIRILIZANTE SIMPLE): Implica la existencia de
hiperandrogenismo in útero que condiciona la aparición de macrogenitosomia en el varón
y un grado variable de virilización en la mujer. En la forma perdedora de sal existe un
DÉFICIT TANTO DE CORTISOL COMO DE ALDOSTERONA y se manifiesta en ambos sexos por
perdida salina en la etapa neonatal. En la virilizante simple no esta afectada la síntesis de
aldosterona por lo que se mantiene la homeostasis del sodio.
B. FORMA NO CLÁSICA: Existe HIPERANDROGENISMO DE APARICIÓN POSNATAL. Los
síntomas mas frecuentes durante la infancia son pubarquia prematura, piel grasa con acné,
aceleración del crecimiento y maduración ósea y en las niñas aparece hipertrofia de
clítoris, en la adolescencia y la edad adulta las mujeres pueden presentar trastornos
menstruales, hirsutismo, acné, calvicie, ovarios poliquísticos e infertilidad, los varones
afectados pueden cursar con oligoespermia, acné e infertilidad.

• Como diagnosticas el déficit de la enzima 21-hidroxilasa?


• R = Demostración de VALORES ELEVADOS DE 17-HIDROXIPROGESTERONA

• Cual es el manejo del déficit de enzima 21-hidroxilasa?


• R = GLUCOCORTICOIDES y en niños recién nacidos es FLUOROHIDROCORTISONA y en
adultos es a base de PREDNISONA.
ENFERMEDAD DE PAGET
• A 63-year-old asymptomatic woman is investigated for a HIGH ALKALINE
PHOSPHATASE (ALP) level. X-rays of the pelvis show multiple porotic and
sclerotic lesions with characteristic whorls of trabeculation. HER EXCRETION
OF URINARY HYDROXYPROLINE IS ALSO ELEVATED. Which of the following is
the most likely diagnosis?

• R = Enfermedad de Paget del Hueso

• The elevated ALP AND HYDROXYPROLINE are diagnostic for Paget’s


disease. The bony lesions are blastic and the sacrum and pelvis are most
frequently involved, followed closely by the tibia and femur. Hypercalcemia
can complícate immobilization. The etiology is unknown, but a viral agent
has been postulated. Symptoms may be absent or severe (pain, deformity).
In metastatic cancers of most types the lesion are lytic, and the other
metabolic abnormalities do not have an elevation in hydroxyproline.
Insulina e Hipoglucemiantes orales.
Hipoglucemiantes orales.

• Sulfonilureas:
– 1a generación: tolbutamida (TBT), clopropramida, tolazamida,
acetohexamida.
– 2a generación: gliburida, glipizida, glibenclamida, glicazida. Son más
potentes.
Insulina e Hipoglucemiantes orales.
Hipoglucemiantes orales.

• Sulfonilureas.
– Mecanismos:
• Estimulan liberación de insulina, disminuyen eliminación hepática,
aumentan somatostatina (disminuye glucagon).
• Regulación a la baja de receptores: al susp pueden tener nuevam
respuesta.
• Bloq canal K sensible a ATP: despolarización: aumenta Ca.
Insulina e Hipoglucemiantes orales.
Hipoglucemiantes orales.

• Sulfonilureas.
– Farmacodinamia.
• VO mejor 30´preprandial.
• Unión PP 90-99% (alb).
• T 1/2 tolazamida 4-7hrs, clorpropramida 24-48hrs. 2da generación 1.5-
5hrs (efecto disminuye glucosa x 12-24hrs).
• Metab hígado. Excreción renal. Cuidado en insuf hepática y renal.
Insulina e Hipoglucemiantes orales.
Hipoglucemiantes orales.
• Sulfonilureas.
– Efectos colaterales.
• 1a generación:40%. Menos frec 2da generación.
• Hipoglucemia.
• Medicamentos: desplazan de PP: sulfonamidas, clofibrato, dicumarol,
salicilatos, fenilbutazona. El alcohol:aumenta su acción..
• Náusea, vómito, ictericia colestásica, agranulocitosis, A aplásica, A
hemolítica, alergia.
• Clorpropramida: dosulfiram 10-15%, hiponatremia 5% x aumentar ADH en
túbulo colector.
• TBT: aumenta 2v riesgo de muerte cardiovascular.
Insulina e Hipoglucemiantes orales.
Hipoglucemiantes orales.

• Sulfonilureas.
– Usos.
• DMIND.
• Contraindicada en DMID, embarazo, lactancia, insuf renal y hepática
significativa.
Insulina e Hipoglucemiantes orales.
Hipoglucemiantes orales.
• Biguanidas.
– Metformin y fenformin.
– Fenformin asociado a ác láctica.
– Metformin T1/2 1.3-4.5 hrs. Máx 3g/d. No produce hipoglucemia, no
libera insulina.
– Aumenta acción perif insulina, inhibe gluconeogénesis, disminuye
absorción gluc intestinal.
– VO.
– No en insuf renal o hepática: ác láctica, hipoxemia cr, insuf cardiaca.
– Efectos colat: 20%. Diarrea, náusea, malestar abd, sabor metálico,
anorexia, disminuye absorción vit B12 y folatos.
Insulina e Hipoglucemiantes orales.
Hipoglucemiantes orales.

• Otros.
– Ciglitazona, pioglitazona, troglitazona: thiazolidinedionas. No dan
hipoglucemia. Disminuyen resistencia a insulina x aumentar
transportadores glucosa.
– Inhib alfa glucosidasa: acarbosa. La inhiben en el borde en cepillo:
inhiben absoción CHOS.
– Efectos colaterales: malaabsorción, flatulencia, distensión abd.
NEUMOLOGIA

ENARM
REACCIONES DE HIPERSENSIBILIDAD
ALTERACIONES NOMBRADAS
TIPO NOMBRE ALTERNATIVO MEDIADORES
FRECUENTEMENTE
ATOPIA
1 ALERGIA (IMMEDIATA) ANAFILAXIA IGE
ASMA

ANEMIA HEMOLÍTICA
AUTOINMUNE
IGM O IGG
2 ANTICUERPO DEPENDIENTE TROMBOCITOPENIA
(COMPLEMENTO)
ERITROBLASTOSIS FETALICOS
SÍNDROME DE GOODPASTURE

ENFERMEDAD DEL SUERO


ENFERMEDAD DE COMPLEJO REACCIÓN DE ARTHUS IGG
3
INMUNE LUPUS ERITEMATOSO (COMPLEMENTO)
SISTÉMICO

DERMATITIS DE CONTACTO
TEST DE MANTOUX
CITOTÓXICA HIPERSENSIBILIDAD
4 RECHAZO CRÓNICO DE ÓRGANO CÉLULA T
RETARDADA
TRASPLANTADO
ESCLEROSIS MÚLTIPLE4
SARCOIDOSIS PULMONAR
• En que consiste la sarcoidiosis?
• R = Enfermedad sistémica de ETIOLOGÍA DESCONOCIDA, caracterizada en 90% por INFLAMACIÓN
GRANULOMATOSA PULMONAR

• Que datos clínicos o de laboratorio son mas frecuentes a encontrar en sarcoidiosis?


1) Clínicos: Tos crónica, ERITEMA NODOSO, UVEITIS, parotiditis, hepato-esplenomegalia +
LINFADENOPATIA.
2) Laboratorio: AUMENTO DE ECA 40-80%, hipercalciuria.
3) ANERGIA a pruebas cutáneas 70%.
4) LAVADO BRONCOALVEOLAR con alto contenido de CD4 Y CD8.

• Cuales son los datos radiológicos que te da sarcoidosis?


1) AUMENTO DE TAMAÑO DE GANGLIOS LINFÁTICOS HILIARES BILATERALES en pacientes asintomáticos
con exploración física normal y exámenes de rutina normales se presenta en el 95% de pacientes con
sarcoidosis pulmonar
2) Adenopatia hiliar bilateral/ afección parenquimatosa bajo la forma de infiltrado reticular difuso.

• Cuales son los signos espirometricos mas frecuentes en sarcoidosis?


• R = CAMBIOS RESTRICTIVOS con disminución de los volúmenes pulmonares y capacidad de difusión.

• Que te indica la biopsia de un paciente con sarcoidosis?


• R = GRANULOMAS NO CASEIFICANTES PATOGNOMONICOS

• Cual es el tratamiento de sarcoidosis?


• R = ESTEROIDES E INMUNOSUPRESORES
NEUMONIA NOSOCOMIAL
• Como se define la neumonía nosocomial (NNC)?
• R = Se define la neumonía adquirida en hospital como aquella que se desarrolla después de una estancia
intrahospitalaria de 48 hrs.

• Cual es la etiología de la NNC?


• R = Los patógenos mas comunes incluyen BACILOS GRAMNEGATIVOS, como PSEUDOMONAS
AUROGINOSA, E. coli, Klebsiella y Acinetobacter.

• Cual es la sospecha diagnostica de la NNC?


1) Px presenta infiltrados nuevos o progresivos
2) Hallazgos clínicos como fiebre, leucocitosis o leucopenia, esputo purulento.
3) Los hallazgos radiológicos pueden variar DESDE INFILTRADOS AÉREOS EN FORMA DE PARCHE HASTA
CONSOLIDACIONES LOBARES CON BRONCOGRAMA AÉREO O INFILTRADOS DIFUSOS ALVEOLARES O
INTERSTICIALES.

• Cual es el manejo de la NNC?


1) DE INICIO EMPÍRICO
2) Esquema antibiótico de espectro limitado: px con < 5d de hospitalización con Ceftriaxona, levofloxacina,
Ampicilina y Ertrapenem
3) Esquema antibiótico de espectro amplio: px con >5d de hospitalización Cefalosporinas antipseudomonica
como CEFEPIME, Fluoroquinolona antipseudomonica como CIPROFLOXACINO, Aminoglucosido como
AMIKACINA.
4) La terapia es normalmente de 14 a 21d.
DERRAME PLEURAL
• Cual es la diferencia entre derrame pleural de tipo exudativo y de tipo
trasudativo?

• TRASUDADO: se define como aquel derrame pleural ORIGINADO POR


FACTORES SISTÉMICOS que producen alteraciones en la formación o
reabsorción del líquido pleural. La principal causa es ICC y cirrosis.

• EXUDADO: Es originado por varios factores locales que perturban la


formación y absorción de líquido pleural. SON DEBIDOS PRINCIPALMENTE
A NEUMONÍA BACTERIANA, NEOPLASIAS MALIGNAS, INFECCIONES
VIRALES Y EMBOLIA PULMONAR.
DERRAME PLEURAL
• Cuales son las características de laboratorio del EXUDADO pleural?
1) Relación entre proteínas del liquido pleural y proteína serica >.5,
2) Relación entre DHL de liquido pleural y DHL sérica >.6,
3) DHL >2/3 partes de la DHL sérica.

• Cuales son los datos de laboratorio de un TRASUDADO pleural?


• R = Glucosa igual a glucosa sérica, PH 7.40-7.55, < 1000 leucocitos

• Que características tiene el derrame pleural reumatoide?


• R = Liquido turbio VERDE - AMARILLENTO, FR aumentado, CRISTALES DE
COLESTEROL.

• Que fármacos se utilizan en la pleurodiesis?


• R = DOXICICLINA y talco
PATOLOGIA MEDIASTINAL
• En lo que respecta a la patología mediastinal, cual es la división anatómica?
1) Mediastino ANTERIOR: Compuesto por tráquea, ganglios linfáticos, arco aórtico y sus grandes vasos,
venas inominadas y TIMO
2) Mediastino MEDIO: CORAZON, pericardio, ganglios linfáticos, hilios pulmonares, vena cava superior,
nervio frénico y vago.
3) Mediastino POSTERIOR: ESOFAGO, aorta descendente, vena ácigos, conducto torácico, nervios vagos
y cadena simpática

• Cuales son las lesiones que ocupan el MEDIASTINO ANTERIOR y sus principales características?
1) TIMOMA: Mas común en ADULTOS que en niños. Los síntomas mas comunes son dolor torácico, tos,
disnea. La IMAGEN RADIOGRAFICA TÍPICA ES UNA MASA LOBULADA, bien definida y la TAC muestra
una masa de tejido blando, encapsulada, bien definida y en ocasiones con necrosis o hemorragia. Se
desarrollan síndromes PARATIMICOS como la miastenia grave, hipogammaglobulinemia y aplasia pura
de glóbulos rojos. El síndrome de Good es una inmunodeficiencia por déficit de anticuerpos asociada a
timoma, generalmente benigno.
2) TUMORES DE CELULAS GERMINALES: Como teratomas, seminomas y tumores no seminomatosos.
3) TERATOMAS: Mezcla de tejidos provenientes de las 3 capas germinativas con potencial de
transformación maligna.
4) SEMINOMAS: Los síntomas mas frecuentes son disnea, dolor retroesternal, tos, fiebre,
GINECOMASTIA, perdida de peso y síndrome de vena cava superior por compresión tumoral.
5) NO SEMINOMATOSO: Se asocian a KLINEFELTER. Tiene elevación de hormonas como la
ALFAFETOPROTEINA y gonadotropina corionica humana.
6) BOCIO MEDIASTINAL: Suelen ser EUTIROIDEOS. Con disnea, estridor y disfagia.
7) LINFOMA PRIMARIO: Linfoma Hodking siendo el no Hodking el menos común. Con sintomatología de
FIEBRE, SUDORACION NOCTURNA, perdida de peso, asociada a síntomas por compresión de
estructuras vecinas.
8) LINFOMA NO HODKING: Sus subtipos mas comunes son el linfoma linfoblastico y linfoma células B de
células grandes.
PATOLOGIA MEDIASTINAL
• Cuales son las masas del MEDIASTINO MEDIO mas frecuentes y
sus principales características:
1) QUISTES MEDIASTINALES: Los principales síntomas son tos,
disnea, fiebre, expectoración purulenta y dolor torácico.
2) LINFANGIOMAS: Anormalidades de los vasos linfáticos.

• Cuales son las masas del MEDIASTINO POSTERIOR mas frecuentes


y sus principales características:
1) TUMORES NEUROGENICOS: Se encuentra los tumores benignos y
malignos de la vaina nerviosa
2) TUMORES DE LA VAINA NERVIOSA: Son tumores de crecimiento
lento y en su mayoría sintomáticos siendo los Schwannomas
3) TUMORES MALIGNOS DE LA VAINA NERVIOSA: Se asocia a
neurofibromatosis
MEDIASTINITIS
• Cual es la etiología de la mediastinitis?
• R = Infecciosa

• Cual es el cuadro clínico de la mediastinitis?


1) Escalofríos, FIEBRE, mal estado general y dolor
torácico.
2) Al examen físico puede haber dolor en el esternón,
crepitación y el característico pero inconstante SIGNO
DE HAMMAN (sonido crujiente que se escucha en la
pared anterior del tórax durante la sístole cardiaca).
ASMA
• En que casos se eleva la CAPACIDAD TOTAL DE DIFUSIÓN
PULMONAR?
• R = ICC, ASMA y hemorragia pulmonar debido a un
incremento del volumen sanguíneo en capilares pulmonares.

• En que casos se emplea la broncoscopia rígida?


• R = Hemorragia masiva, extracción de grandes objetos y
biopsia de tumores traqueales.

• Que parámetros evalúan la obstrucción respiratoria de aire?


• R = La reducción del VEF1/CVF < 75%.

• Con que medicamentos se realizan las pruebas de


estimulación bronquial?
• R = Histamina, METACOLINA.
EPOC
• Como se define la bronquitis crónica?
• R = Secreción excesiva de moco bronquial manifestada por tos productiva durante 2-3 meses
en 2 años consecutivos.

• Que enzima liberada es la que produce el enfisema pulmonar?


• R = PROTEASA de PMN

• Cual es la clasificación HISTOLOGICA del enfisema pulmonar y con que se relacionan cada
una mas frecuentemente?
1) CENTROLOBULILLAR: Se caracteriza porque el área afectada está en el lobulillo proximal,
en especial por destrucción de los bronquiolos respiratorios y dilatación de los lóbulos
superiores sin afectar a los alvéolos distales. Representa el 95% de los casos de enfisema y
la principal manifestación en los FUMADORES asociándose comúnmente con una
BRONQUITIS CRÓNICA.
2) PANACINAR: Es la forma más comúnmente asociada a una DEFICIENCIA DE ALFA-1
ANTITRIPSINA y se caracteriza por involucrar al extremo ciego de los alvéolos de manera
homogénea, mas que a los bronquiolos respiratorios y acompañado de los característicos
cambios destructivos. Es más frecuente en la base de los pulmones.
3) PARASEPTAL: La enfisema paraseptal interesa prevalentemente a la parte periférica del
lobulillo, vecina a la pleura creando grandes espacios aéreos en la región interlobulillar. Es
más frecuente en el ápice pulmonar que en las bases y ocasionalmente se ASOCIA CON
NEUMOTÓRAX ESPONTÁNEO.
4) IRREGULAR: Es una enfisema cicatrizante, comprometido de manera irregular al acino
asociado a géneros asintomáticos. Se le llama también enfisema paracicatrizal o
paraseptal.
EPOC
• Como se encuentra la capacidad vital forzada pulmonar en
EPOC grave?
• R = Disminuida

• Como se encuentra la CTP en EPOC sin complicaciones?


• R = Aumentada

• Como se encuentra el volumen residual en EPOC?


• R = Aumentado (VR)

• Como se encuentra la relación VR/CPT en EPOC?


• R = Aumentada
EPOC
• Cual es la principal característica del soplador ROSADO en EPOC (enfisema
predominante)?
• R = DISNEA, edad de presentación a los 50ª, pacientes con tos y esputo mucoide,
pacientes delgados con bajo peso y no hay edema periférico.

• Cual es el principal síntoma del AZUL abotagado EPOC (bronquitis


predominante)?
• R = TOS CRÓNICA con ESPUTO PURULENTO, edad de presentación 30-40ª, disnea
leve y edema periférico.

• Que muestra la Rx de tórax en el soplador rosado (enfisema)?


• R = Hiperaereacion con hemidiafragmas aplanados.

• Que muestra la Rx de tórax en el azul abotagado (bronquitis predominante?


• R = Sombras intersticiales.

• Que iniciales controlan el tratamiento de EPOC?


• R = GOLD
FIBROSIS QUISTICA
• Que es la fibrosis quística?
• R = Es una enfermedad multisistemica que se transmite de forma AR y es
letal.

• Cual es el tipo de mutación que se encuentra en 60% de los casos en FQ?


• R = Brazo corto de cromosoma 7 posición AF508

• Que dato te sugiere fibrosis quística en el RN?


• R =Impactación fecal

• Cual es la fisiopatología de la fibrosis quística?


• R = Anomalías en la proteína de canal de cloruro de la membrana celular

• Cual es el resultado de la prueba del sudor en fibrosis quística?


1) Concentración de CLORURO mediante iontoforesis > 60 mEq en 2
ocasiones en la prueba de Gibson.
2) TRIPSINA INMUNORREACTIVA en neonatos, la cual se basa en el hecho
de que en etapas tempranas, esta proteína se encuentra anormalmente
elevada por la obstrucción de conductos pancreáticos.
FIBROSIS QUISTICA
• Que tienen en común los pacientes que tienen FQ?
• R = Casi todos los varones tienen AUSENCIA BILATERAL CONGÉNITA DEL
CONDUCTO DEFERENTE CON AZOOSPERMIA EN 95%.

• Que signos y síntomas es común encontrar en FQ?


1) Antecedentes de enfermedad pulmonar crónica, pancreatitis recurrentes e
infertilidad.
2) Esteatorrea, dolor abdominal y diarrea son comunes.
3) La malabsorción se manifiesta por deposiciones abundantes, fétidas y de
características grasas.

• Cuales son los agentes etiológicos de las bronquiectasias infectadas en FQ?


• R = Haemophilus influenzae, Staphylococcus Aureus o Pseudomonas aeruginosa

• Cual es el manejo terapéutico de la FQ?


• R = Comprende a la nutrición, prevención de infecciones, inhaloterapia y manejo
de exacerbaciones inflamatorias.

• Que medicamentos en FQ disminuyen las exacerbaciones respiratorios, consumo


de antibiótico y aumento de la VEF1?
• R = Desoxirribonucleasa humana recombinante inhalada.
BRONQUIECTASIAS
• Que enfermedad es la responsable de mas del 50% de
bronquiectasias?
• R = FIBROSIS QUÍSTICA y el resto se debe secundario a infecciones

• Cual es la sintomatología de las bronquioectasias?


• R = Tos crónica, aumento del esputo purulento diario de cantidad
significativa, hemoptisis y neumonías recurrentes.

• Que datos se observan en una radiografía de tórax y una TAC con


bronquiectasias?
1) Radiografía de tórax: Incremento inespecífico de la trama vascular,
atelectasias u opacidades irregulares
2) TAC: imagen en “RIELES DE TRANVIA” que refleja el engrosamiento
de las paredes y la fibrosis peribronquial.

• Cual es el manejo de las bronquiectasias?


1) Agentes mucoliticos como acetilcisteina, salina hipertónica
2) Antimicrobianos en prevención y para tratamiento
TUBERCULOSIS
• Efecto adverso más común de la ISONIACIDA?
• R = NEUROPATÍA PERIFÉRICA

• Efecto adverso más común de la RIFAMPICINA?


• R = Hemorrágicos, HEPATITIS.

• Efecto adverso del ETAMBUTOL?


• R = NEURITIS ÓPTICA

• Efecto adverso de ESTREPTOMICINA?


• R = DAÑO AL VIII PAR CRANEAL, nefrotoxicidad.

• Medicamentos para Tb en EMBARAZADA?


• R = Isoniacida, Rifampicina y Etambutol por 4-8 semanas o 9 meses.

• En una paciente con VIH + en el que usas isoniacida se debe usar que
medicamento para evitar sus reacciones adversas mas comunes?
• R = VITAMINA B6
TUBERCULOSIS
• Por que esta contraindicada la estreptomicina en el embarazo?
• R = Puede causar sordera congénita.

• Están contraindicados los antituberculosos durante la lactancia?


• R = No

• Antes de usar el ETAMBUTOL que pruebas debes realizar?


• R = AGUDEZA VISUAL, valorando colores rojo y verde.

• Que es necesario medir en suero antes del uso de PIRAZINAMIDA?


• R = ACIDO ÚRICO

• Que prueba es necesesaria realizar antes del uso de ESTREPTOMICINA?


• R = AUDIOMETRICA.

• Que agente etiológico es el mas común en enfermedad pulmonar por MNT?


• R = M. Avium y M. Kanassi

• Que fármacos utilizas en MNT mas antituberculosos?


• R = Claritromicina o azitromicina.
CANCER PULMONAR
• Cual es la neoplasia pulmonar benigna mas frecuente?
• R = Hamartroma

• Cuales son las neoplasias pulmonares malignas en cáncer pulmonar?


1) Cáncer broncogénico de células pequeñas (CPCP): representa el 20% de los canceres
pulmonares
2) Cáncer broncogénico de células no pequeñas (CPCnP): representa el 80% de los
canceres pulmonares y abarca los subtipos de ADENOCARCINOMA 60%, escamoso o
epidermoide 30% y el carcinoma indiferenciado de células grandes < 10%

• Que es el tumor de Pancoast en el cáncer pulmonar?


• R = Los carcinomas que se localizan en el surco pulmonar superior. Tiene una evolución
lenta, tendencia a infiltrar los linfáticos subpleurales y por contigüidad las raíces
nerviosas del 8vo nervio cervical y del 2do y 3er dorsales.

• Cuales son las manifestaciones clínicas del tumor de Pancoast?


1) DOLOR LOCALIZADO EN EL HOMBRO DE CARACTERÍSTICAS RADICULARES,
IRRADIADO A LO LARGO DEL BRAZO Y PARESTESIAS
2) SÍNDROME DE HORNER con debilidad y atrofia de los músculos extrínsecos

• Cual es la incidencia de ca de células escamosas en cáncer pulmonar primario?


• R = 25-35%, se presenta con hemoptisis y se puede diagnosticar mediante citología de
esputo.
CANCER PULMONAR
• Cual es la estirpe etiológica mas común de ca pulmonar?
• R = Adenocarcinoma 60 %

• Cual es la presentación típica del adenocarcinoma pulmonar?


• R = Masas o nódulos periféricos

• Cual es el pronóstico del CPCP?


• R = Se propaga por VÍA HEMATOGENA muy rápido con SOBREVIDA MUY CORTA, de
aproximadamente 2m después del diagnostico sin tratamiento.

• Cual es el pronostico de CPCnP?


• R = Se propaga mas lentamente con BUENA RESPUESTA AL TRATAMIENTO

• Que porcentaje de pacientes con CPCP desarrolla secreción inadecuada de ADH?


• R = 15% ADH

• Que porcentaje de pacientes con carcinoma de CPCnP de CÉLULAS ESCAMOSAS desarrolla


hipercalcemia?
• R = 10% HIPERCALCEMIA

• Cuales son los fármacos antineoplásicos mas novedosos?


• R = Pacitaxel, gemcitabina, vinorelbina.
CARCINOIDE PULMONAR
• Cuales son los síntomas mas comunes de los
tumores carcinoides bronquiales?
• R = Hemoptisis, tos, sibilancias y neumonía
RECURRENTE

• Que caracteriza al síndrome carcinoide?


• R = Rubores, diarrea, sibilancia e hipotensión.

FIBROSIS PULMONAR
Que enfermedad pulmonar te da el patrón radiográfico llamado, en panal de abejas?
• R = Fibrosis pulmonar y Enfermedad intersticial pulmonar o enfermedad parenquimatosa pulmonar
difusa

• Cual es el nombre actual que recibe la fibrosis pulmonar idiopática?


• R = Neumonía intersticial fibrosante idiopática

• Cual es el cuadro clínico y datos de espirometria de la fibrosis pulmonar idiopática?


1) Tos seca
2) Disnea que se incrementa con ejercicio
3) Capacidad vital forzada (CVF) disminuida
4) Relación CVF/ VEF 1 dentro de los LIMITES NORMALES O AUMENTADA

• Cuales son los criterios para fibrosis pulmonar idiopática?


1) Mayores: Exclusión de otras causas conocidas de enfermedad pulmonar intersticial, como toxicidad
por fármacos, exposición ambiental y enfermedades del tejido conectivo. Pruebas de función
respiratoria que sugieran restricción. Opacidades reticulares bibasales como mínimo VIDRIO
DESPULIDO en la TAC. Biopsia transbronquial o lavado broncoalveolar no compatible con otro
diagnostico.
2) Menores: Edad > 50ª. Disnea de esfuerzo. Mas de 3 meses con la enfermedad. Estertores tipo velcro
en región subescapular bilateral.

• Cual es el manejo de la fibrosis pulmonar idiopática?


• R = Prednisona y azatriopina o ciclofosfamida.

• Cual es el pronóstico de la fibrosis pulmonar idiopática?


• R = Malo con promedio de sobrevida de 1ª.
NEUMOCONIOSIS
• Que es la neumoconiosis?
• R = Enfermedad pulmonar provocada por la acumulación de POLVOS MINERALES secundaria
a la inhalación crónica de los mismo

• Cuales son las patologías agrupadas en la neumoconiosis?


1) ASBESTOSIS por inhalación de ASBESTO
2) SILICOSIS por la inhalación de SÍLICE en su mayoría en forma de cuarzo
3) SIDEROSIS por la acumulación pulmonar de HIERRO
4) ANTRACOSIS por la exposición secundaria a CARBON

• Que caracteriza al mesotelioma, el cual es una complicación de la asbestosis?


• R = Son tumores primarios originados a partir del recubrimiento superficial de la pleura
(80%), del peritoneo (20%) y es mas común en hombres 3:1.

• Que características tiene el liquido pleural en el mesotelioma pulmonar?


• R = Exudativo y hemorrágico

• Que datos radiográficos denota la silicosis?


• R = Pequeñas opacidades redondeadas en todo el pulmón, calcificación de la periferia de los
ganglios linfáticos hiliares “CALCIFICACIÓN EN CASCARA DE HUEVO”.

• Que caracteriza al síndrome de Caplan?


• R = Se caracteriza por NÓDULOS NECROBIOTICOS en periferia de pulmón en los
TRABAJADORES DE CARBÓN con ARTRITIS REUMATOIDE
TROMBOEMBOLIA PULMONAR
• Cual es la causa hereditaria mas común de hipercoagubilidad?
• R = Resistencia a proteína C activada o FACTOR V DE LEIDEN

• Cual es la triada de Virchow identificada como factor de riesgo para desarrollo de


trombosis venosa?
1) Disminución del flujo sanguíneo o estasis venosa
2) Lesión o traumatismo vascular
3) Estado hipercoagulable o trombofilico

• Datos que sugieren TEP?


1) DISNEA SÚBITA SOSTENIDA
2) TAQUICARDIA SOSTENIDA
3) Hipotensiona rterial sistólica < 100 mm Hg
4) SINCOPE
5) DOLOR PRECORDIAL ANTERIOR QUE SIMULA IAM
6) Colapso cardiorespiratorio

• 98. Cuales son las anormalidades EKG en pacientes con TEP?


1) Taquicardia sinusal
2) Patron S1, Q3 – T3
TROMBOEMBOLIA PULMONAR
• Que datos de laboratorio te sugieren TEP?
1) Valores plasmáticos de DIMERO D, el cual es un producto de la degradación de
fibrina, mismo que se aumenta en caso de trombos 300-500 mg/ul con ELISA.
2) Hipoxemia
3) ALCALOSIS RESPIRATORIA

• Cual es el signo de Westemark en TEP?


• R = ARTERIA PULMONAR CENTRAL PROMINENTE CON OLIGOHEMIA LOCAL en la
radiografía de tórax. Área de vascularización y perfusión disminuida como
consecuencia de una embolia pulmonar, acompañada de un ensanchamiento de la
arteria pulmonar central del lado afectado. Se observa en el 2% de los casos.

• Que método sustituyo el diagnostico por imagen de TEP con gamagrama


pulmonar?
1) ANGIOGRAFÍA PULMONAR selectiva que se considera actualmente como
ESTANDAR DE ORO
2) TAC helicoidal

• Cual es el estándar de oro en el tratamiento de la TEP?


1) HEPARINA
2) El manejo con acenocumarinicos (acenocumarina y warfarina) constituye el
tratamiento a largo plazo por vía oral
TROMBOEMBOLIA PULMONAR
• Como actúa la heparina en TEP?
• R = Se fija y acelera la capacidad de la antitrombina 3 para inactivar a la trombina, factores 9 y 10.

• Como actúan los acenocumarinicos en TEP?


• R = EL mecanismo de acción depende de la inhibición de la carboxilacion de los factores de coagulación
2,7, 9 y 10 (vitamina K dependientes)

• Cual es la meta del INR en TEP?


• R = Que se encuentre entre 2-3

• Que te indica un INR <2?


• R = Aumenta el RIESGO DE TROMBOSIS

• Que te indica un INR >4?


• R = Aumenta el riesgo de HEMORRAGIA.

• Como actúa la estreptoquinasa, urocinasa y el activador tisular de plasminogeno?


• R = Aumentan los valores de plasmina, lisando directamente los trombos intravasculares

• Que medida adicional puede utilizarse en pacientes que esta contraindicado el uso de
anticoagulantes?
• R = Bloqueo de la vena cava inferior

• En pacientes quirúrgicos de riesgo tromboembolico que medida de prevención se utiliza?


• R = Heparina de bajo peso molecular
HIPERTENSION PULMONAR
• Cual es la definición hemodinámica de la hipertensión pulmonar (HP)?
• R = PRESIÓN ARTERIAL pulmonar media MAYOR A 25 MM HG EN REPOSO O DE 30 MM HG DURANTE EL
EJERCICIO.

• Cual es la etiología de la HP?


1) in situ Vasoconstricción sostenida
2) Proliferación vascular pulmonar
3) Trombosis
4) Disnea en reposo o de esfuerzo.
5) Disnea en reposo con dolor retroesternal semejante a angina de pecho.
6) Policitemia

• Cuales son los síntomas que puede incluir la hipertensión pulmonar, además de los datos de
laboratorio mas frecuentes?
• Que refleja hipoxemia crónica.

• Cuales son los datos EKG de HP?


• R = Desviación del eje a la derecha, hipertrofia ventricular derecha y crecimiento de aurícula derecha.

• Que tratamiento esta indicado tanto en la HP primaria como secundaria?


• R = Inhibidor de la fosfodiesterasa 5 (sindenafil), Anticuagulación permanente, venoclisis prolongada
continua de prostaciclina +++

• Como se maneja el Cor pulmonale como complicación de HP?


• R = Tratamiento de enfermedad de fondo, diuréticos, disminución de Na y O2
GRANULOMATOSIS DE WEGENER
• En que consiste la granulomatosis de Wegener (vasculitis pulmonar)?
• R = Es una enfermedad pulmonar ideopatica que se manifiesta por una combinación de GLOMERULONEFRITIS,
VASCULITIS GRANULOMATOSA NECROSANTE de vías respiratorias superiores e inferiores y vasculitis de
pequeños vasos.

• Cuales son los síntomas mas frecuentes en granulomatosis de Wegener?


• R = SINUSITIS CRÓNICA, artralgias, exantema y disminución de peso

• Cual es el signo pulmonar mas común en granulomatosis de Wegener?


• R = Infiltrado pulmonar nodular con cavitaciones

• Que prueba de lab te confirma granulomatosis de Wegener?


• R = ANCA 90% con patrón de tinción citoplasmática fluorecente cANCA

• Como diagnosticas granulomatosis de Wegener sin laboratorio?


• R = Con biopsia pulmonar, tejido sinusal o renal demostrando VASCULITIS GRANULOMATOSA NECROSANTE.

• Cual es el tratamiento de la granulomatosis de Wegener o vasculitis pulmonar?


• R = Corticosteroides y ciclofosfamida

• Cual es el corticosteroide de elección en granulomatosis de Wegener?


• R = PREDNISONA ORAL.

• Cual es antibiótico de elección en granulomatosis de Wegener?


• R = TMP/SMZ
CHURG- STRAUSS
• Que es el sindrome de Churg- Strauss?
• El síndrome de Churg Strauss, también llamado angeítis y granulomatosis alérgica, es un
síndrome multisistémico CARACTERIZADO POR RINITIS ALÉRGICA, ASMA Y EOSINOFILIA
importante en el extendido periférico. El órgano más comúnmente comprometido es el
pulmón, seguido de la piel.
• Sin embargo, el síndrome de Churg Strauss puede afectar cualquier órgano sistema,
incluyendo el sistema cardiovascular, el gastrointestinal, el renal y el sistema nervioso
central.

• Cuales son manifestaciones clínicas del síndrome de Churg-Strauss?


Se manifiesta en tres fases:
1) Asma y rinitis alérgica.
2) Eosinofilia periférica e infiltración a órganos.
3) Vasculitis con afectación sistémica:
• a. Pulmonar: asma grave, tos, disnea, sinusitis.
• b. Neurológica: mononeuritis, polineuropatía.
• c. Cutánea: púrpura, urticaria.
• d. Cardiovascular: pericarditis, IAM.
• e. Gastrointestinal: gastroenteritis eosinofílica, dolor abdominal, diarrea, hemorragia.

• Que laboratorio se mantiene constantemente presente en el sx de Churg-Strauss?


• R = Eosinofilia
CHURG- STRAUSS
• Para su clasificación el American College of Rheumatology (1990) publicó
los siguientes criterios diagnósticos contando con al menos cuatro de los
siguientes seis:

1) Asma: historia de dificultad respiratoria o sibilancias espiratorias difusas.


2) Eosinofilia: eosinofilia >10% en conteo diferencial de glóbulos blancos.
3) Mono o polineuropatía: desarrollo de mononeuropatía, mononeuropatía
múltiple, o polineuropatía (distribución en guante o en bota) atribuible a
vasculitis sistémica.
4) Infiltrados pulmonares migratorios: infiltrados pulmonares migratorios o
transitorios (no incluyendo infiltrados fijos). Atribuibles a vasculitis.
5) Anormalidades de los senos paranasales: historia de dolor agudo o
crónico de los senos paranasales u opacificación radiográfica de los senos
paranasales.
6) Eosinófilos extravasculares: biopsia incluyendo arterias, arteriolas o
vénulas mostrando acúmulos de eosinófilos en áreas extravasculares.
CHURG- STRAUSS
• Cual es el manejo del sindrome de Churg Strauss?
A. La combinación entre altas dosis de
corticoesteroides y ciclofosfamida son significativas
en el tratamiento de esta patología con una baja
tasa de resistencia;
B. Los corticoesteroides pueden ser usados solos en
casos del síndrome de Churg Strauss, en cambio la
ciclofosfamida se utiliza solo en casos de pacientes
con ANCA positivo.
SÍNDROME DE GOOD PASTEUR
• Menciona cuales son las causas de hemorragia alveolar difusa?
1) Enfermedad de ANTICUERPO ANTIMEMBRANA BASAL (sx de good pasteur),
2) Vasculitis y enfermedad de la colagena vascular (lupus eritematoso sistémico,
granulomatosis de Wegener).

• Cuales son las causas de hemorragia alveolar difusa de causa no inmunitaria?


• R = Coagulopatia, estenosis mitral, fármacos como la penicilamina y hemosiderosis
pulmonar idiopática.

• Que es el síndrome de Good pasteur?


• R = HEMORRAGIA ALVEOLAR recurrente idiopática y GLOMERULONEFRITIS rápidamente
progresiva que se manifiesta en varones de 30-40ª.

• Como diagnosticas el síndrome de Good pasteur?


• R = En la biopsia se observan depósitos lineales de IgG tanto en glomérulos como en
alveolos por medio de inmunofluorecencia y PRESENCIA DE AC CONTRA MEMBRANA BASAL
glomerular en suero.

• Que tratamiento a dado resultado en el síndrome de Goodpasteur?


• R = PREDNISONA, CLICLOFOSFAMIDA Y PLASMAFERESIS
SÍNDROME DE INSUFICIENCIA RESPIRATORIA
AGUDA
• Como se define el síndrome de insuficiencia respiratorio (SIRA antes SIRPA)?
• R = Pacientes con disnea, hipoxemia refractaria a tratamiento, disminución de
la complianza pulmonar e infiltrados pulmonares difusos en la radiografía de
tórax.

• Cual es la etiología del SIRA?


1) Causas de lesión pulmonar de manera indirecta como sobredosis de
fármacos
2) Causas de lesión pulmonar de manera directa como la broncoaspiracion o
politraumatismo

• Cual es el cuadro clínico del SIRA?


1) Deterioro progresivo de la función respiratoria
2) Disnea en reposo
3) Hipoxemia grave Pa O2 < 50 mm Hg refractaria a la oxigenoterapia
4) ALCALOSIS RESPIRATORIA
5) Taquipnea
6) CIANOSIS
SÍNDROME DE INSUFICIENCIA
RESPIRATORIA AGUDA
• Cuales son los criterios diagnósticos de SIRA?
R = Se diagnostica SIRA si cumple con 2 de los siguientes criterios
1) Inicio agudo o súbito de la sintomatología
2) Opacidades pulmonares alveolares o en parches difusas bilaterales
3) Hipoxemia PaO2 < 60 mm Hg y relación PaO2 / FiO2 < 200
4) Presión capilar pulmonar < 18 mm Hg o ausencia de signos de hipertensión
auricular izquierda en caso de no poder medir la presión en cuna pulmonar.

• Cual es el manejo del SIRA?


1) Ventilación mecánica invasiva de elección
2) Los fármacos no han demostrado beneficio en SIRA

• Cual es el pronostico del SIRA?


• R = Mortalidad entre 40-60%

• Que maniobras han demostrado disminuir la mortalidad den SIRA?


1) El volumen corriente bajo (<6 ml/kg de peso ideal)
2) Mantenimiento de una presión meseta o plateau < 30 cm H2O
MICOSIS BRONCOPULMONAR
ALERGICA
• Que es la micosis broncopulmonar alérgica (MBA)?
• R = Trastorno de hipersensibilidad por alergia a los antígenos micóticos.
Aspergilosis broncopulmonar alérgica.

• Agente causal de MBA?


• R = Especies de aspergilus.

• Como dx MBA?
• R = Tiene que reunir 6-7 criterios. 1) antecedente de asma 2) eosinofilia
periférica 3)reactividad cutánea a aspergilus 4)Ac vs aspergilus 5) aumento
de IgE 6)bronquiectasias 7)infiltrados pulmonares.

• Cual es el tratamiento de elección en MBA?


• R = Prednisona
BIBLIOGRAFIA
• EXARMED
• PAPADAKIS
• CTO
• HARRISON

También podría gustarte